Contracts Flashcards

You may prefer our related Brainscape-certified flashcards:
1
Q

Assignment of a unique service

A

Assignment occurs when one party delegates performance of a contract to another party, which is generally acceptable. However, a service that is unique can never be assigned without the express consent of all parties to a contract.

I.e. The band, which is well-known, provides a unique performance, which cannot be delegated.

How well did you know this?
1
Not at all
2
3
4
5
Perfectly
2
Q

Assignor v. Intended beneficiary

A

Assignment occurs when one party delegates performance of a contract to another party. In the event of an assignment, both the assignor and assignee are legally responsible for the performance of the contract. Here, there was a delegation of certain tasks to the tree trimmer, thus forming a partial assignment. However, overall responsibility for the work done still rested with the landscaper. Therefore, the mutual determination that the trimmer’s services were not needed before the trimmer performed means that the trimmer still had no rights as an assignor.

A beneficiary, in contractual terms, is a third party who sees material benefits from a contract between other parties. There are two classes of beneficiaries, intended and incidental beneficiaries. An intended beneficiary is a party where performance on a contract was specifically intended to benefit a third party. Incidental beneficiaries, or those who are not specifically named by a contract, do not have the same rights of enforcement. There is nothing to indicate that the tree trimmer was a specific beneficiary, so as an incidental beneficiary, he has no right to enforce the contract.

How well did you know this?
1
Not at all
2
3
4
5
Perfectly
3
Q

donee beneficiary’s damages

A

If a promisor’s performance is intended to benefit a donee beneficiary, the promisee will ordinarily not have suffered any economic losses, and thus will only be able to sue for nominal damages, although courts are willing to order specific performance.

How well did you know this?
1
Not at all
2
3
4
5
Perfectly
4
Q

In the event of a shipment of nonconforming goods, a valid contract exists, but …

A

…the shipping party is in immediate breach.

Importantly, under the UCC, the buyer has the right to accept nonconforming goods and still pursue the seller for breach, as long as they provide notice of nonconformity within a reasonable time.

How well did you know this?
1
Not at all
2
3
4
5
Perfectly
5
Q

Substantial performance v. Material breach

A

Substantial performance occurs when a breach is not serious enough to be considered a material breach, and the performance is considered “close enough.”

Material breach occurs if an express term of the contract is breached and the aggrieved party is damaged as a result.

An express condition is one that is expressly and materially relevant to a contract, and a breach of that condition would create a material breach.

How well did you know this?
1
Not at all
2
3
4
5
Perfectly
6
Q

Where only one of the parties to an agreement is mistaken as to a material aspect of the transaction, his mistake is…

A

…no defense to formation unless the other party knew or should have known of the mistake (or the mistake was due to a clerical error).

How well did you know this?
1
Not at all
2
3
4
5
Perfectly
7
Q

Promissory estoppel is an equitable theory that applies …

A

… in the absence of a contract.

How well did you know this?
1
Not at all
2
3
4
5
Perfectly
8
Q

Promissory estoppel is only necessary to enforce a promise where ….

A

…insufficient consideration exists.

Promissory estoppel is an equitable remedy which arises when (1) a promise is made that (2) would reasonably induce reliance, (3) actual reliance, and (4) injustice without enforcement.

Here, while the uncle made a promise, the second element is doubtful. Would someone go to college simply to receive a puppy as a gift? Probably not. Even if her reliance was genuine, and the niece actually relied on that promise, incurring debt, the question is the level of injustice that would occur if the niece was denied her puppy and the level of detrimental reliance. While incurring student loans could be considered detrimental, the niece also has a college degree, which could be quite valuable in terms of longer-term salary growth. Overall, it seems unlikely the uncle’s gratuitous promise was detrimental to the level required for promissory estoppel. Therefore, the gratuitous promise, without consideration, is not binding, and the uncle will keep the animal.

How well did you know this?
1
Not at all
2
3
4
5
Perfectly
9
Q

Where there is full performance of a contract for services that cannot be performed within a year and for which there is no writing that satisfies the Statute of Frauds, full performance will make the contract…

A

…enforceable against the paying party.

Here, the contract was for services of the consultant, and performance could not be completed within a year from formation of the contract because the agreement required the consultant to perform monthly check-ins for one year. The consultant fully performed her end of the agreement. Even though there is no writing signed by a company representative memorializing the contract, the company must pay the consultant the full contract price.

How well did you know this?
1
Not at all
2
3
4
5
Perfectly
10
Q

accepting a buyer’s offer to purchase goods he UCC by shipping nonconforming goods

A

Unlike the common law, a seller can accept a buyer’s offer to purchase goods under the UCC by shipping nonconforming goods.

i.e.

The dentist offered to purchase four Model A chairs, and the manufacturer accepted the dentist’s offer by shipping four Model B chairs (nonconforming goods). (A contract formed, but the shipment of Model B chairs is also a breach of the contract under the perfect tender rule.)

How well did you know this?
1
Not at all
2
3
4
5
Perfectly
11
Q

Under the mailbox rule, …

A

… an acceptance is effective upon dispatch if the acceptance is properly posted, and a binding contract is created when the offeree dispatches an acceptance.

The mailbox rule does not apply when the offeree first dispatches a rejection, followed by an acceptance; in that case, the first to be received is effective.

How well did you know this?
1
Not at all
2
3
4
5
Perfectly
12
Q

knockout rule

A

Where merchants come to an oral agreement and both merchants send written confirmations and the confirmations contain conflicting terms, the knockout rule applies such that neither term is in the contract.

How well did you know this?
1
Not at all
2
3
4
5
Perfectly
13
Q

An offer is a manifestation of intent by the offeror to be bounded by the contract that is communicated to the offeree with definite and certain terms.

Example?

A

Here, the owner wrote: “I will rent you the seaside cottage on the same terms as last year. I need your answer by June 1,” thereby demonstrating his intent to be bound by a contract.

How well did you know this?
1
Not at all
2
3
4
5
Perfectly
14
Q

The Statute of Frauds requires that a suretyship or guarantee for another’s debt be in writing. However, there is an exception when …

A

…the guarantor’s primary motivation in guaranteeing the debt is his own financial interest.

How well did you know this?
1
Not at all
2
3
4
5
Perfectly
15
Q

While minors may form contracts, their obligations are voidable. Can a minor who misrepresents his age to the other party may disaffirm the contract?

A

Yes.

Disaffirmance is accomplished by words or deeds that objectively signify the election to avoid liability. Disaffirmance can occur prior to performance or even afterwards. Disaffirmance is personal to the minor or his legal representative.

Until the disaffirmance, the contract is binding.

How well did you know this?
1
Not at all
2
3
4
5
Perfectly
16
Q

When a minor disaffirms after performance, he must …

A

…restore any goods or benefits still in his possession. If the goods have been damaged or have depreciated, the minor’s restitution obligation extends no further than returning the goods “as now.”

How well did you know this?
1
Not at all
2
3
4
5
Perfectly
17
Q

The “new business rule” says that …

A

…if a new business cannot calculate its expectation damages with reasonable certainty, then it is automatically barred from recovery.

How well did you know this?
1
Not at all
2
3
4
5
Perfectly
18
Q

nder UCC Section 2-706, where a buyer has made an anticipatory repudiation of a contract, a seller may recover…

A

the difference between the contract price and the resale price.

The seller, however, may not recover where the resale was a “sweetheart” deal not made in good faith or a commercially reasonable manner.

How well did you know this?
1
Not at all
2
3
4
5
Perfectly
19
Q

The court may reform the written contract to accurately reflect what the parties agreed to and will enforce those terms. The parol evidence rule does not bar evidence of the prior agreement, since such a principle would effectively preclude reformation.

Whether the source of the discrepancy between the parties’ prior agreement and the written instrument is through the mistake of one or both parties or through the fraudulent misrepresentation by one of them …

A

…does not matter; in either case, the court may reform the document to conform to the actual prior agreement.

How well did you know this?
1
Not at all
2
3
4
5
Perfectly
20
Q

The general rule is that a unilateral mistake will not justify reformation of the contract. Exception?

A

Reformation of the contract terms may be available where the other contracting party knew or reasonably should have known of the mistake and took advantage of the aggrieved party’s error.

How well did you know this?
1
Not at all
2
3
4
5
Perfectly
21
Q

When the buyer refuses to accept goods or otherwise repudiates the agreement, the measure of the seller’s damages will depend on whether or not the seller resells the goods.

A

Where, the seller resells the goods, he will be entitled to the contract-resell differential, which is the difference between the contract price and the resale price.

When the seller does not resell, he is entitled to the contract-market differential, or the difference between the contract price and the market value of the goods.

He would also be entitled to any incidental or consequential damages, in either case.

How well did you know this?
1
Not at all
2
3
4
5
Perfectly
22
Q

Under the parol evidence rule, ….

A

…evidence of any agreement made prior to or contemporaneous with the written, integrated contract is not admissible to contradict or vary the terms expressed in that contract.

However, the parol evidence rule does not bar the admission of oral agreements made after the execution of a written contract.

How well did you know this?
1
Not at all
2
3
4
5
Perfectly
23
Q

right to retract her repudiation and fulfill her obligations under the contract exists if

A

A gave notice that she would, after all, perform under the contract before the B:

  • took any action in reliance on the repudiation,
  • gave notice of acceptance of the repudiation, or
  • commenced an action against her.
How well did you know this?
1
Not at all
2
3
4
5
Perfectly
24
Q

An implied-in-fact contract is formed by

A

…manifestations of assent other than oral or written language,
i.e., by conduct.

Where a person knowingly accepts offered benefits, such conduct, viewed objectively, may be said to manifest an agreement to the conferral of such benefits, resulting in a
contract implied in fact.

While generally an acceptance must be communicated to an offeror to be effective, courts will often find an acceptance where an offeree silently accepts offered benefits.

How well did you know this?
1
Not at all
2
3
4
5
Perfectly
25
Q

A retail store ordered 100 women’s swimsuits
at $10 each, as advertised in the catalog of a
swimwear manufacturer. The manufacturer
shipped 40 swimsuits to the store, along with a
letter stating in relevant part: “We have shipped
you 40 swimsuits at $10 each in response to your
recent order. Please remit $400. Be informed
that limited inventory will prevent us from
being able to ship any additional suits at this
time or at any time during this year’s beach
season.” The store took the 40 suits and began
to sell them. The store immediately sought an
alternate supplier of swimsuits. The best price
it could obtain was $11 per suit from a different
company. The store ordered, received, accepted,
and paid for 60 suits at $11 each from the other
company. The store has refused to pay the
manufacturer the $400, and the manufacturer
has sued for payment.
What is the manufacturer entitled to recover?

A

The manufacturer is entitled to $340.

As a general rule, advertisements, catalogs, and other price
quotations are construed as invitations for offers rather than offers. If the language of the catalog could be construed as a definite promise to specific offerees, a court might construe it as an offer, but there is no evidence in this question for such an interpretation.

The store’s order is therefore an offer to purchase 100 swimsuits.

Because goods are involved, UCC Article 2 applies. Under
UCC section 2-206(1)(b), an offer to buy goods for prompt shipment invites acceptance either by a promise to ship or by prompt shipment of conforming or nonconforming goods.

In this case, the manufacturer shipped goods that did not conform to the quantity term of the store’s offer.

The Code provides that the shipment of nonconforming goods amounts to both an acceptance of the offer and a breach of the newly formed contract unless the shipper precedes or accompanies the shipment with notice that it is not accepting the offer and is offering the shipment as an accommodation. Proper accommodation notice makes the shipment a counteroffer, rather than an acceptance.

The letter sent by the manufacturer stating that it will be able to ship only 40 suits is not proper accommodation notice. Nothing indicates that the manufacturer is making a counteroffer rather than an acceptance. In fact, the demand for payment suggests that the manufacturer believes there is a contract. Without sufficient accommodation notice, a nonconforming shipment
is both an acceptance of the offer and a breach of contract, entitling the nonbreaching party to damages. Thus, in this case, the store is entitled to deduct the $60 in damages from its payment. [See UCC §2-711]

How well did you know this?
1
Not at all
2
3
4
5
Perfectly
26
Q

Impossibility requirements

A

For impossibility of performance to apply, the impossibility must be “objective”; i.e., the duties could not be performed by anyone.
Also, the impossibility must arise after the contract has been entered into.

How well did you know this?
1
Not at all
2
3
4
5
Perfectly
27
Q

If a period of acceptance is stated in an offer, the offeree must accept within that period to create a contract. Failure to timely accept terminates the power of acceptance in the offeree (i.e., a late acceptance will not be effective and will not
create a contract).

Will the mailbox rule apply?

A

Under the mailbox rule, an acceptance generally is effective upon dispatch (i.e., the acceptance creates a contract at the moment it is mailed or given to the delivery company).
However, the mailbox rule does not apply where the offer states that acceptance will not be effective until received.

***

Note that the developer will not be able to successfully argue that the acceptance was valid since the late delivery was the messenger company’s fault. This would be a valid argument if the mailbox rule applied here, because the acceptance would have been effective when the message was given to the messenger company. However, by opting out of the mailbox rule, the realtor put the burden of any negligence in delivery on the developer.

How well did you know this?
1
Not at all
2
3
4
5
Perfectly
28
Q

It will be a defense to enforcement of a contract if either the consideration or the subject matter is illegal.

If the illegality is present at the time of the offer, …

If the illegality arises after the offer but before acceptance,..

If the illegality arises after formation of a valid contract,…

A

… there is no valid offer.

… the illegality operates to revoke the offer.

…it discharges the contract because performance has become
impossible.

How well did you know this?
1
Not at all
2
3
4
5
Perfectly
29
Q

A homeowner offered a landscape gardener
$1,000 to trim and reshape the bushes on her
property if the gardener could finish the job
before her garden party on June 1. The gardener
told the homeowner that he would get back to
her after he had checked his calendar. The next
day, the gardener phoned the homeowner, who
was not at home, and left a message on her
voicemail that he had the time, but could not do
the job for less than $1,200. The gardener did
not hear from the homeowner for several days.
As June 1 drew closer, the gardener phoned
the homeowner again and left another message
on her voicemail stating that “I’ll do the job
for $1,000, this weekend, unless that would be
inconvenient.” The homeowner replayed the
second message just as she was leaving town on
a business trip and did not contact the gardener.
That weekend, unbeknownst to the homeowner,
the gardener took his tools to the homeowner’s
house and trimmed and reshaped the bushes
to the homeowner’s specifications. When the
homeowner returned from her trip several
days later, the gardener presented her with a
handwritten invoice for $1,000.
If the homeowner refuses to pay the gardener,
and the latter brings an action solely for breach
of contract to recover the $1,000 contract
amount, who will likely prevail?

A

The homeowner will likely prevail on the breach of contract claim because she did not enter into a contract with the gardener. To form a contract, there must be a valid offer and acceptance. The homeowner made an offer, but the gardener rejected the offer the next day with his first phone call. Once an offer is rejected, the offeree’s power of acceptance is destroyed. Thus, the gardener’s second call was not an acceptance, but rather a counteroffer. The homeowner did nothing to accept the gardener’s counteroffer, and this is not the type of case where silence will be deemed to be an acceptance (e.g., where the parties have so agreed or where that has been their course of dealing). Thus, there was no acceptance and no contract to breach. (

How well did you know this?
1
Not at all
2
3
4
5
Perfectly
30
Q

Under certain circumstances, an executory bilateral contract may be formed without any communication of acceptance.

What is a common example?

A

A common example is where prior dealings between the parties, or trade practices known to both, create a commercially reasonable expectation by the offeror that silence represents an acceptance.

In such a case, the offeree is under a duty to notify the offeror if it does not intend to accept.

i.e.

Here, despite the language in the invitation making leases subject to approval, the leasing company never sent any notification of approval prior to sending out the tickets and
invoice right before the season would start. This course of dealing over the past five years gave the advertising agency reason to expect that the leasing company’s silence after the invitation was returned constituted an acceptance by the leasing company, regardless of the company’s actual
intent.

How well did you know this?
1
Not at all
2
3
4
5
Perfectly
31
Q

A counteroffer serves as …..

A

…. both a rejection terminating the original offer and a new offer from the original offeree, thus reversing the former roles of the parties and giving the original offeror the right to accept or reject the new offer.

How well did you know this?
1
Not at all
2
3
4
5
Perfectly
32
Q

A sporting goods retailer whose tent stock was
running low saw a listing for the tent she wanted
priced at $90 in the catalog of a large camping
goods manufacturer. The retailer phoned the
manufacturer and placed her order for 10 tents
on May 1. The next day, the manufacturer
mailed the retailer a letter informing her that
the tents were now $92 and that they would be
shipped to her on May 16. The retailer received
the letter on May 4, but never responded. On
May 15, the retailer received a catalog from
another company showing tents similar to the
ones that she ordered, but for a cost of $70.
She immediately called the manufacturer to
cancel her order. Nevertheless, the manufacturer
shipped the tents to the retailer on May 16.
Assuming that the parties’ communications
were sufficient to form a contract, on what day
was the contract formed?

A

The contract was formed on May 2.

An offer to buy goods for shipment is generally construed as inviting acceptance either by a promise to ship or by shipment.

Here, the letter constitutes a
promise to ship and thus is an acceptance. The rule for acceptances is that they are effective as
soon as they are dispatched, which was May 2.

How well did you know this?
1
Not at all
2
3
4
5
Perfectly
33
Q

In a contract for the sale of goods, the failure to state the price does not prevent formation of a contract if the parties intended to form a contract without the price being settled. In such
a case, if the price is left to be agreed to by the parties and they fail to agree,

A

a reasonable price at
the time of delivery will be supplied by the court. [UCC §2-305]

How well did you know this?
1
Not at all
2
3
4
5
Perfectly
34
Q

On November 3, an investor who had watched
his stocks sink to unprecedented low levels sent
a fax message to a dealer in precious metals:
“Please quote your best price on 800 troy ounces
platinum bars for immediate delivery at my
bank.” At 10 a.m. the next morning (November
4) the dealer replied by fax, “My best price
is $475 per ounce.” The investor received the
dealer’s message later on that same day.
What is the best characterization of the
communications between the investor and the
dealer?

A

The investor’s communication was a request for an offer and the dealer’s response was an offer.
For a communication to be an offer, it must create a reasonable expectation in the offeree that the offeror is willing to enter into a contract on the basis of the offered terms. The investor’s communication does not pass the test because it is clear on its face that he did not want to be bound by whatever price the dealer came up with, but rather wanted to find out what the dealer would offer.
The dealer’s communication, on the other hand, passes the test. While it said nothing more than the price, it was sent in response to a request containing specific delivery terms and a specific quantity. Under the circumstances, the dealer’s response would have created a reasonable expectation
in the investor that the dealer was willing to enter into a contract under the terms of the two
communications.

How well did you know this?
1
Not at all
2
3
4
5
Perfectly
35
Q

Shortly after a series of rapes took place
within a city, the city council approved the
offering of a $25,000 reward for the arrest and
conviction of the perpetrator of the rapes. Information
concerning the reward was published in
the local newspaper.
In which of the following ways could the city’s
reward offer be effectively accepted?

A

Offers of this type normally require only that the offerees supply information leading to the arrest and conviction of the culprit.

The language of the reward offer should not be read with
total literalness, but rather in the context of what the city was seeking to obtain and the normal
duties required to accept an offer of this kind.

How well did you know this?
1
Not at all
2
3
4
5
Perfectly
36
Q

Under the divisibility doctrine,….

A

…if a party performs one of the units of a divisible contract, he is entitled to the agreed-on equivalent for that unit even if he fails to perform the other units.

It is not a condition precedent to the other party’s liability that the whole contract be performed.

For a contract to be divisible:

(i) the performance of each party must be divided
into two or more parts under the contract,

(ii) the number of parts due from each party must be
the same, and

(iii) the performance of each part by one party is agreed on as the equivalent of the
other party’s corresponding part.

How well did you know this?
1
Not at all
2
3
4
5
Perfectly
37
Q

Accord

A

An accord is an
agreement in which one party to an existing contract agrees to accept, in lieu of the performance that she is supposed to receive from the other party, some other, different performance.

The accord must be supported by consideration, but the consideration is sufficient if it is of a different type than called for under the original contract, even if the substituted consideration is of less value.

An accord suspends the right to enforce the original agreement.

Performance of the accord (i.e., satisfaction) cuts off the parties’ rights to enforce the original contract and
discharges the accord.

If a creditor breaches an accord agreement, the debtor has the option of either raising the accord agreement as an equitable defense in the creditor’s action and asking that it be dismissed,
or waiting until he is damaged (i.e., until the creditor is successful in an action on the original contract) and then bringing an action at law for damages for breach of the accord contract.

How well did you know this?
1
Not at all
2
3
4
5
Perfectly
38
Q

condition subsequent

v.

condition precedent

A

condition subsequent is one the occurrence of which cuts off an already existing absolute duty of performance

(it means other party doesn’t have to perform)

condition precedent is one that must occur before an
absolute duty of immediate performance arises in the other party (it means other party must perform)

How well did you know this?
1
Not at all
2
3
4
5
Perfectly
39
Q

In July of last summer a grape grower
contracted with a winery to deliver “500 tons
of premium quality pinot chardonnay grapes
grown on my ranch.” The price was to be $1,000
per ton and delivery was to be on or before
September 15. In August of the same year, the
grape grower entered into an identical contract
with a vineyard to sell 300 tons of premium
quality pinot chardonnay grapes.
The grape grower completed his harvest by
September 10 and had 800 tons of premium
quality grapes. On September 11, an unexpected
rain ruined 400 tons, and the grape grower
notified the winery and the vineyard on that
day that he would only be able to deliver 250
tons to the winery and 150 tons to the vineyard.
On September 14, the vineyard purchased an
additional 150 tons of premium quality pinot
chardonnay grapes from a different grape
farmer, one of several other available sources for
premium quality pinot chardonnay grapes. These
grapes along with the 150 tons from the grape
grower gave the vineyard the 300 tons it needed.
On September 15, what is the winery’s legal
position with regard to the grape grower’s failure
to deliver the 500 tons of grapes required by his
contract?

A

When crops are destroyed in the case of a farmer who has contracted to sell crops from a designated tract of land,
the loss may be governed either by UCC section 2-613 (casualty to identified goods) or

section 2-615 (failure of presupposed conditions/commercial impracticability).

The result is the same
under either section.

Under section 2-613, if goods identified when the contract is made suffer casualty without fault of either party before the risk of loss passes to the buyer, the contract maybe avoided. If the loss is partial, the buyer may treat the contract as avoided or accept the goods with allowance from the price for the deficiency. The buyer does not have any further rights
against the seller and thus cannot sue the seller for breach.

Under section 2-615, a crop failure resulting from an unexpected cause excuses a farmer’s obligation to deliver the full amount as long as he makes a fair and reasonable allocation among his buyers. The grape grower has done
this by allocating pro rata between the winery and the vineyard.

Nevertheless, under UCC section 2-616, the buyer may either accept the proposed modification or terminate the contract.

Under either provision, the winery is free to reject the 250 tons of grapes.

How well did you know this?
1
Not at all
2
3
4
5
Perfectly
40
Q

Buyers of a house with an old aboveground
pool wanted it removed, so the current
homeowners agreed that they would arrange
to have the pool moved to their new home, but
since the sale was occurring in winter, the pool
would have to be moved at a later date. The
parties therefore agreed that the buyers would
pay all but $5,000 of the home’s sale price at
closing, and then pay the final $5,000 six months
after the sale. The contract further stated:
It is understood and agreed that the
purchasers’ obligation to pay the
$5,000 six months after the sale shall
be voided if the current homeowners
have not, within three months after the
aforesaid sale, removed the existing
pool in the rear of the house.
What is the homeowners’ removal of the pool
from the backyard of the house?

A

Condition subsequent is one the occurrence of which cuts off an already existing duty of performance.

The form of the condition requiring removal of the pool is that of a condition subsequent because, under the language of the contract, failure to do so will cut off the buyers’ duty to pay
the $5,000.

How well did you know this?
1
Not at all
2
3
4
5
Perfectly
41
Q

Accomplishing accord and satisfaction by a check.

A

An accord and satisfaction may be accomplished by a good faith tender and acceptance of a check conspicuously marked “payment in full” where there is a bona fide dispute as to the amount owed.

How well did you know this?
1
Not at all
2
3
4
5
Perfectly
42
Q

A general contractor advertised in a trade
publication that she planned to bid on the
construction of a new building. The advertisement
welcomed bids from subcontractors to
perform various functions, such as plumbing,
electrical work, and masonry. The lowest
plumbing bid she received was from a plumber
for $10,000. The general contractor used that
bid in preparing her bid and submitted the bid to
the client. Shortly thereafter, the plumber called
her and explained that there was a mistake in
his bid and he could not perform the work for
less than $12,000. The general contractor was
awarded the contract and told the plumber she
was accepting his $10,000 bid, but the plumber
reiterated that he would not do the work for less
than $12,000. The general contractor hired a
different company to do the plumbing work on
the building at a cost of $12,000. She now sues
the plumber for damages.
What is she entitled to recover?

A

(B) $2,000, which represents the difference
between the plumber’s bid and the amount
she had to pay for the plumbing work.

The plumber can be held liable because his offer will be deemed irrevocable for a reasonable length of time on a theory of promissory estoppel.

Promissory estoppel renders an offer binding as an option contract even without consideration if the offeror should reasonably expect it to induce action or forbearance of a substantial character by the offeree before acceptance, and
such action or forbearance is in fact induced.

The plumber offered to do the work for $10,000 if the general contractor was awarded the contract.
The plumber should have expected that the general contractor would use this figure to prepare her bid and that if she were awarded the contract, she may be bound. This is what occurred.

The measure of the general contractor’s damages is the amount she reasonably paid for a substitute performance, less the amount saved as a consequence of the breach; i.e., the $12,000 paid to the other plumbing company minus the $10,000 not paid to the plumber, or $2,000

How well did you know this?
1
Not at all
2
3
4
5
Perfectly
43
Q

Neighbors of an apparently destitute couple
bought a month’s supply of food and gave it to
them. Later, the wife confided in the neighbors
that she and her husband did have money and
that, because they had been so kind, she was
leaving them money in her will. When the wife
died, at the neighbors’ request the husband gave
the neighbors the following signed instrument:
“In consideration of my wife’s promise to our
neighbors, and their agreement not to sue her
estate, I agree to pay them the sum of $5,000.”
When the husband died of a heart attack several
days later, the neighbors asked the administrator
of his estate to pay them the $5,000. The administrator
refused on the ground that there was no
consideration for the agreement.
On which of the following theories would it be
most likely that the neighbors would recover?

A

The most likely theory is that the husband and the neighbors entered into a valid compromise.

If the neighbors have given up a good faith claim, their agreement with the husband is a compromise supported by valid consideration. Thus, there is an enforceable contract.

How well did you know this?
1
Not at all
2
3
4
5
Perfectly
44
Q

Exclusivity is implied in output or requirements K?

A

Exclusivity is implied in a requirements contract; otherwise the
buyer’s promise would be illusory.

Consideration for a requirements
contract exists because the promisor is suffering a legal detriment by parting with the
legal right to buy goods he may need from another source. On the other side, the manufacturer
has suffered legal detriment by agreeing to sell to the retailer any amount it requires at the stated
price. The manufacturer has parted with any discretion in the amount sold to the retailer and its
price.

How well did you know this?
1
Not at all
2
3
4
5
Perfectly
45
Q

Reservation of an unqualified right to cancel or
withdraw from a contract at any time may amount to an illusory promise. However, the promise is not illusory, and there is a valid consideration, if the right to cancel is restricted in any way.

Example?

A

Here, the right of either party to cancel is restricted because it must be preceded by reasonable notice to the other party, which was provided. Therefore, the promises are not illusory, and both parties are bound.

The manufacturer finds itself confronted with circumstances that present extreme and unreasonable difficulty and expense in complying with its contractual duties. While this additional test might not be sufficient to discharge the manufacturer’s duties on grounds of impracticability, it is clearly grounds for supplying the manufacturer with a good
faith reason for canceling the contract.

How well did you know this?
1
Not at all
2
3
4
5
Perfectly
46
Q

Output contract

A

meaning that the seller has promised to sell to the buyer all of the goods the seller produces, and the buyer has agreed to buy
that amount from the seller.

How well did you know this?
1
Not at all
2
3
4
5
Perfectly
47
Q

As a general rule, a contract requires a bargained-for exchange
between the parties as consideration; “past” or “moral” consideration is usually insufficient.
Among the many exceptions to this rule is …

A

…where a technical defense such as the statute of limitations
bars enforcement of the prior obligation and a new promise is made in writing.

In such a case, courts will state that the “moral” consideration is sufficient consideration for the new agreement,
or that the existence of the prior obligation is a substitute for consideration. Regardless of how the courts characterize it, the new promise will be enforceable only according to its terms,
not the terms of the original obligation.

How well did you know this?
1
Not at all
2
3
4
5
Perfectly
48
Q

In cases where the builder breaches after partially performing, the owner of the land is entitled to ….

A

the cost of completion plus reasonable compensation for any delay in performance.

Courts generally allow the builder to offset or recover for work performed to date to avoid the unjust enrichment of the owner.

How well did you know this?
1
Not at all
2
3
4
5
Perfectly
49
Q

In a contract for the sale of goods, a seller can collect his lost profits when the buyer breaches if the seller cannot be made whole by a subsequent sale of the item contracted for. This occurs where the seller can obtain or manufacture as many goods as it can sell (e.g., a car dealership), because in such a situation, the seller would have been able to sell to the subsequent purchaser anyway. This is known as a lost volume situation, and in such a situation, the UCC allows the seller to sue for his lost profits. Generally, lost profits are measured by …

A

… the difference between the cost of goods and the contract price, less the seller’s saved expenses.

i.e.

KP - what the dealer paid to purchase the car from the manufacturer

How well did you know this?
1
Not at all
2
3
4
5
Perfectly
50
Q

A homeowner purchased a large recreational
vehicle. Local ordinances in the homeowner’s
suburb prohibited residents from parking recreational
vehicles on the street or in an open
driveway, so the homeowner contacted a local
contractor and explained his requirements for a
garage. The contractor measured the vehicle and
then entered into a written contract to build a
garage for the homeowner at a price of $5,000,
payable on completion of the job. When the
garage was finished but before the contractor
was paid, the homeowner drove the vehicle
into the garage, only to discover that the garage
was three inches too short to accommodate the
vehicle. The homeowner was told that it would
cost $4,000 to partially dismantle the garage
and rebuild it to fit the vehicle. The homeowner
refused to pay the contractor anything for the
job. The contractor consulted with several
independent real estate appraisers, and they all
agreed that the garage had enhanced the value of
the homeowner’s property by $6,000.
If the contractor sues the homeowner, what
amount will the contractor likely recover?
(A) $5,000, because a three-inch variation in
length is, at most, a minor breach.
(B) $2,000, measured by the difference
between the amount that the garage
enhanced the value of the property and the
cost to rebuild the garage to specifications.
(C) $1,000, measured by the difference between
the contract price and the amount it would
cost to rebuild the garage to specifications.
(D) Nothing, because the three-inch error was
a material breach, and the contractor will
be unable to successfully claim substantial
performance.

A

(C) The contractor will recover $1,000 in a restitution or quasi-contract action. Despite having built a garage, the contractor has breached the contract here. He was to build a garage that would fit the homeowner’s recreational vehicle and the garage that he built is too short. Moreover, the breach was material, even though the garage was a mere three inches short, because the homeowner did not receive the substantial benefit of his bargain—a place for his vehicle. Therefore, the contractor
cannot recover in contract and (A) is incorrect. Nevertheless, courts agree that where the breach is not willful, the contractor can recover on the failed contract in restitution or quasi-contract to prevent unjust enrichment. Here, it is clear that the breach was not willful. (D) is therefore incorrect.

Regarding the amount of recovery, most courts would limit the recovery to the contract price (rather than the benefit received by the nonbreaching party), offset by the damages to the nonbreaching party (here, the $4,000 cost of rebuilding the garage as bargained for). Thus, (C) is correct and (B) is incorrect.

How well did you know this?
1
Not at all
2
3
4
5
Perfectly
51
Q

A contractor gave the low bid for some
electrical repairs to a homeowner’s house. Based on this bid, the contractor and the homeowner
entered into a contract stating that the contractor
would perform the electrical repairs for $6,000. Before beginning work on the project, the
contractor notified the homeowner that he would
lose money on the job at that price, and would not proceed with the work unless the homeowner would agree to increase the price to $9,000. The homeowner thereupon, without notifying the contractor, entered into a contract with an electrician to make the repairs for $7,500,
which was the fair market cost of the work to
be done. The electrician finished the house on
schedule and then showed the homeowner that
he (the electrician) had spent $8,500 on the job.
The homeowner thereupon paid the electrician
the full balance of their contract price plus an
additional $1,000, so that the electrician would
not lose money on the job. In a contract action by the homeowner against the contractor, how much will the homeowner recover?

(C) $2,500, the difference between the contractor’s original contract price and the total
amount the homeowner paid the electrician for the repairs.
(D) $1,500, the difference between the contractor’s original contract price and the electrician’s contract price.

A

(D) The homeowner can recover $1,500, the difference between the contractor’s contract price and the contract price of the substitute performance.

Here, while the homeowner actually paid $2,500
more than the contractor’s contract price to have the house repaired, he was obligated to pay
only $1,500 more because the electrician had a legal duty to make the repairs to the house for
his contract price and no more. _The homeowner will not be able to recover the $1,000 difference
because he has a duty to mitigate damages, and paying more than he was actually obligated to pay breaches the duty.
_

How well did you know this?
1
Not at all
2
3
4
5
Perfectly
52
Q

Liquidated damages clauses are enforceable only if …

A

…damages were difficult to estimate at the time the contract was formed, and the amount agreed upon is a reasonable forecast of the damages that would result from a breach.

How well did you know this?
1
Not at all
2
3
4
5
Perfectly
53
Q

In a writing signed by both parties, a renowned architect agreed to design and supervise construction of a new house for a buyer.
The architect’s fee was to be paid on completion
of the house. When the design plans were about two-thirds complete, the architect assigned
to a newly licensed architect “all of my rights and duties under my design and constructionsupervision contract with the buyer.” The novice architect expressly promised the architect to carry out the work to the best of her ability. The buyer, on learning of the assignment, refused to allow the novice architect to proceed on the project and brought an action against the architect to compel him to resume and complete
performance of the contract. Is the buyer entitled to such relief?
(A) Yes, because the architect’s services under
the contract are unique.
(B) Yes, because the architect has personally
completed two-thirds of the design work.
(C) No, because the architect-buyer contract is
one for personal services by the architect.
(D) No, because the architect effectively
delegated his remaining duties under the
architect-buyer contract to the novice architect.

A

(C) The buyer cannot compel the architect to resume performance. Contracts for personal services are not subject to specific performance notwithstanding the fact that damages might be inadequate or difficult to assess or the services to be performed are unique. The courts reason that specific performance of personal service contracts is tantamount to involuntary servitude and would present enforcement problems. At most, the buyer would be able to obtain an injunction to prevent the architect from working on another project at the times the architect agreed to work for the buyer.

How well did you know this?
1
Not at all
2
3
4
5
Perfectly
54
Q

When a buyer breaches by repudiating its offer, the seller has a right to recover either …

A

…the difference between the contract price and the market price

or

the difference between the contract price and the resale price,

plus incidental damages.

How well did you know this?
1
Not at all
2
3
4
5
Perfectly
55
Q

Consequential damages

A

are special damages and reflect losses over and above standard expectation damages. These damages result from the nonbreaching party’s particular circumstances.
These damages may be recovered only if, at the time the contract was made, a reasonable person
would have foreseen the damages as a probable result of a breach. Therefore, to recover consequential damages, the plaintiff must show that the breaching party knew or had reason to know of the special circumstances giving rise to the damages.

How well did you know this?
1
Not at all
2
3
4
5
Perfectly
56
Q

A woman ordered several cases of rubber
bracelets from an online wholesaler. The
bracelets were sold only in bulk and they were
marketed as intended for resale. The bracelets
were stamped with a political message and
the date of an upcoming political rally in the
woman’s city. The woman, like several others
who ordered the bracelets, planned to resell the
bracelets at the event to make a profit. Due to the unexpected popularity of the event, however, the wholesaler was overwhelmed with similar orders and was late in shipping the bracelets to the woman. The woman received her order several days after the rally, thus thwarting her plans. The woman can prove that the bracelets were in such high demand at the rally that the others who received their bracelets on time quickly resold all their bracelets at twice their wholesale cost and that she could have sold even more bracelets at the rally than she ordered if she had been given the chance. Can the woman recover her lost profits from
the wholesaler?
(A) Yes, as a lost volume seller.
(B) Yes, as reliance damages.
(C) Yes, as consequential damages.
(D) No, as her profits are too speculative.

A

(C) The woman can recover her lost profits as consequential damages. In addition to the standard measure of damages, consequential damages may be awarded for further losses resulting from a breach of contract that any reasonable person would have foreseen would occur from the breach at the time of entry into the contract.

Here, the wholesaler was well aware that the bracelets were going to be resold at the rally. The wholesaler had marketed the bracelets for this very reason. The bracelets were stamped with the date of the rally. It was foreseeable that delivery of the bracelets after that date would result in a loss to the buyer.

How well did you know this?
1
Not at all
2
3
4
5
Perfectly
57
Q

When an assignment has been made, but the obligor on the contract (here, the retailer) has not been informed of the assignment, it is still obligated to pay…

A

…the party with whom it originally dealt.

How well did you know this?
1
Not at all
2
3
4
5
Perfectly
58
Q

A contract between an investor and an
inventor stated that the investor will finance all
of the inventor’s expenses for the next six months in exchange for half of the profits from any inventions that the inventor develops during that time. During those six months, the investor runs into financial problems and borrows $200,000 from a bank. The investor executes a written instrument providing that the bank “is entitled to collect the debt from my share of the profits, if any, under the inventor contract.” The bank gave prompt notice of this transaction to the inventor. If the inventor thereafter refuses to account for any profits to the bank and the bank sues the inventor for the investor’s share of profits then realized, what is the inventor’s strongest argument in defense?

(C) The bank is not an assignee of the investor’s
rights under the investor-inventor contract.

(D) The bank is not an intended third-party
beneficiary of the investor-inventor contract.

A

(C) The only theory under which the bank can recover the investor’s share of the profits is that
the bank is an assignee of the investor’s rights.

(D) would not provide a strong defense for the
inventor, because it is clear that the bank was not an intended beneficiary of the investor-inventor
contract.

How well did you know this?
1
Not at all
2
3
4
5
Perfectly
59
Q

Intended beneficiary

A

A party is an intended beneficiary of the agreement when

(i) she was expressly designated in the contract,
(ii) some performance is to be made directly to her, and
(iii) she stands in a close relationship to the promisee, suggesting that she intended for her to benefit.

However, an intended beneficiary can enforce the contract only after his rights have vested.

Vesting will occur when the beneficiary

(i) manifests assent to the promise in a manner invited or requested by the parties,
(ii) brings suit to enforce the promise, or
(iii) materially changes position in justifiable reliance on the promise.

How well did you know this?
1
Not at all
2
3
4
5
Perfectly
60
Q

A store sold office equipment and supplies to
various businesses in the area. The store entered
into a written agreement with an electronics company to purchase all of its monthly requirements of printers for a period of five years at a specified unit price. The agreement contained a clause prohibiting assignment of the contract. Shortly thereafter, the electronics company assigned the contract to a finance company as security for a loan. The store subsequently ordered the printers from the electronics company, which were delivered on time. Which of the following accurately states the
legal effect of the clause prohibiting assignment
of the contract?

(A) The clause as properly interpreted was not
breached, and the assignment was effective.
(B) The clause made the assignment to the
finance company ineffective.
(C) The electronics company’s assignment
was a breach but was nevertheless effective
to transfer to the finance company the
electronics company’s rights against the
store.
(D) The clause is effective if the parties can
establish a rational reason for including the
covenant into their agreement.

A

(A) The clause was not breached, and the assignment was effective.

Under the UCC, which governs this sale-of-goods case, “unless the circumstances indicate the contrary, a prohibition of assignment of ‘the contract’ is to be construed as barring only the delegation to the assignee of the assignor’s performance.”

Here, the electronics company assigned to the finance company the right to receive payment on its contract with the store. There was no delegation of duties to the finance company (the assignee). Therefore, when the electronics company “assigned the contract” to the
finance company, it assigned only the right to payments, and it did not breach its contract with the store.

How well did you know this?
1
Not at all
2
3
4
5
Perfectly
61
Q

On December 10, a housepainter entered
into a contract with a homeowner to paint her
house for $8,000, to be paid on completion of
the work. On December 20, before the work
was completed, the painter sent a letter to the
homeowner telling her to pay the $8,000 to
an assignee, whom he identified by name. The
painter sent a copy of the letter to the assignee,
then completed the work. The homeowner
refused to pay the assignee, and the assignee
sued the homeowner for the $8,000. Which of the following, if true, would be the homeowner’s best defense?

(B) The assignee was incapable of performing
the painter’s work.
(C) The painter had not performed his work in
a workmanlike manner.
(D) The assignment was made without consideration.

A

(C) The homeowner’s best defense is that the painter had not performed his work in a workmanlike manner. The painter has assigned his claim against the homeowner to an assignee. In the absence of an enforceable agreement to the contrary, an assignee would take subject to defenses good against his assignor. Therefore, if the painter has not properly performed the work, the homeowner could use this as a defense in a suit brought by the assignee.

(B) is wrong because the fact that the assignee is incapable of performing the painter’s work is immaterial. The painter merely assigned his right to payment; he did not delegate his painting duties to the assignee.

(D) is wrong because consideration is not required for an assignment to be valid.

How well did you know this?
1
Not at all
2
3
4
5
Perfectly
62
Q

A third-party donee beneficiary has no cause of action against the promisee, because the promisee’s act is gratuitous and he may not be held to it.

The only time a donee beneficiary has recourse against the promisee is when…

A

…the promisee himself tells the donee beneficiary about the contract and should foresee that the beneficiary would rely on it.

If the beneficiary does reasonably rely on the contract to his detriment, the beneficiary can
sue the promisee directly under a promissory estoppel, not a third-party beneficiary, theory.

How well did you know this?
1
Not at all
2
3
4
5
Perfectly
63
Q

A homeowner wanted to have his yard
landscaped. He called a number of commercial
establishments that do such work and received
a bid for $5,000, a bid for $4,500, and a bid for
$4,000. The homeowner entered into a contract
with the low bidder to have the yard landscaped
for $4,000. Shortly before the low bidder was scheduled to begin work, he called the homeowner and told him that his secretary had made a mistake in adding the figures, and he could not possibly do the work for less than $4,600. If the homeowner sues the low bidder for
breach of contract, who will prevail?

(B) The homeowner, because he did not have
reason to know of the low bidder’s mistake.
(C) The low bidder, because he had not yet
commenced performance under the
contract.

A

(B) The homeowner will prevail in his suit against the low bidder for breach of contract because the
homeowner did not know or have reason to known of the low bidder’s mistake with respect to the amount of its bid.

Where only one of the parties entering into a contract is mistaken about facts relating to the agreement, the unilateral mistake will not prevent formation of the contract unless the nonmistaken party knew or had reason to know of the mistake.

Here, the relatively small difference between the $4,000 bid and the next lowest bid of $4,500 (where there was also a $500 difference between the next lowest bid and the highest bid) probably would not have alerted the homeowner to the existence of a mistake.

(C) is incorrect because it is irrelevant that the contract is still executory.

How well did you know this?
1
Not at all
2
3
4
5
Perfectly
64
Q

A contract can be reformed to reflect the original intent of the parties where there has been a mutual mistake in the integration.

A

The plaintiff’s negligence is not a bar to reformation.

i.e. P did not read the K and didn;t notice that it was missing a clause beneficial for him.

How well did you know this?
1
Not at all
2
3
4
5
Perfectly
65
Q

A plaintiff contended that a defendant owed
him $6,000. The defendant denied that he owed
the plaintiff anything. Tired of the dispute, the
defendant eventually signed a promissory note
by which he promised to pay the plaintiff $5,000
in settlement of their dispute. In an action by the plaintiff against the defendant on the promissory note, which of the following, if true, would afford the defendant the best defense?

(A) Although the plaintiff honestly believed
that $6,000 was owed by the defendant, the
defendant knew that it was not owed.
(B) Although the plaintiff knew that the debt
was not owed, the defendant honestly was
in doubt whether it was owed.
(C) The original claim was based on an oral
agreement, which the Statute of Frauds
required to be in writing.
(D) The original claim was an action on a
contract, which was barred by the applicable
statute of limitations.

A

(B) The scenario in (B) provides the best defense because it would enable the defendant to rescind
based on unilateral mistake.

Rescission of a contract is available when one party is mistaken about material facts relating to a contract, the mistake adversely affects that party, and the other party knows of the mistake.

(A) is wrong because in this case it is the nonmistaken party who is adversely affected. (Note that under choices (A) and (B) it is unclear whether the plaintiff’s claim is based on a contract. If it is, the promissory note will be treated as a modification. The modification could be rescinded as discussed above, but there also is an issue of whether there was valid consideration for the modification under the preexisting legal duty rule.
Under the preexisting legal duty rule, past consideration is valid consideration if there is an honest dispute as to duty owed. There would be no honest dispute under (B) because the nonmistaken party is taking advantage of the mistaken party, but this is not true in (A).

(C) is wrong. The
Statute of Frauds is not a problem here because the plaintiff is seeking to enforce the promissory
note—which complies with the Statute of Frauds—and not the original agreement. Neither does the preexisting legal duty rule negate the consideration here because there is an exception to the rule for reaffirmations of voidable promises (e.g., promises unenforceable under the Statute
of Frauds, promises by infants, promises based on fraud, etc.).

(D) is wrong because the statute of limitations would run anew on the note that the plaintiff is trying to enforce, and there is no preexisting legal duty consideration problem because of the exception to the rule for “technical defense bars”—a new promise to pay a legal obligation barred by a technical defense (such as the
statute of limitations) is enforceable according to the terms of the new promise.

How well did you know this?
1
Not at all
2
3
4
5
Perfectly
66
Q

A licensed real estate broker and a homeowner
entered into a written listing agreement in which
the homeowner promised, among other things,
to pay the real estate broker a 6% commission of
the selling price of the homeowner’s home if the
real estate broker obtained a buyer ready, willing,
and able to purchase it. The homeowner’s home
was listed for $180,000 in a service made available to real estate professionals. A prospective buyer, after going to the real estate broker’s office and viewing the homeowner’s
home, submitted a written offer to purchase
the home for $180,000. The homeowner rejected
this offer by not accepting it within the stated
period. The buyer brings an action against the
homeowner for specific performance, seeking to
compel him to sell the home.
What is the probable outcome of this litigation?

(A) The homeowner will win, because no writing
or writings constitute a memorandum
sufficient to satisfy the Statute of Frauds.
(B) The homeowner will win, because the
buyer’s remedy at law is adequate.

A

(A) The buyer cannot obtain specific performance against the homeowner because the absence of a written memorandum signed by the homeowner and containing the essential terms of an agreement between the buyer and the homeowner means that there is no enforceable contract. To obtain specific performance, there must be an enforceable contract between the parties. Pursuant to the Statute of Frauds, a contract for the sale of land is not enforceable unless it is evidenced by a writing signed by the party sought to be bound. I_n addition to this signature, the writing should contain a recital of consideration, the terms and conditions of the agreement, the identity of the party sought to be charged, and an identification of the contractual subject matter._ The buyer is attempting to obtain specific performance of a contract to purchase the homeowner’s house. Thus, the buyer must show the existence of a contract through a writing sufficient to satisfy the Statute of Frauds. The only writings mentioned in the facts are the buyer’s written offer to purchase the house and the real estate listing agreement between the homeowner and the real estate broker. The
latter reflects only an agreement between the homeowner and the real estate broker as to the terms of the real estate broker’s services as a broker in connection with the sale of the homeowner’s house. This in no way constitutes written evidence of an agreement between the homeowner and the buyer. The buyer’s written offer was never signed by the homeowner and was, in fact, rejected by the homeowner in a manner consistent with its terms. Thus, this writing does not memorialize an agreement between the homeowner and the buyer for the sale of the homeowner’s house; no agreement was ever reached. Therefore, there is no enforceable contract that can be the subject of specific performance.

(B) is incorrect because a purchaser of land is almost always deemed to have an inadequate
remedy at law.
Each parcel of land is always considered unique; i.e., unlike any other that could be purchased with the same sum of money. Thus, if the buyer did have a cause of action against the homeowner, he would not have an adequate remedy at law.

(C) is incorrect because the facts do not indicate that the real estate broker and the homeowner intended that the buyer or any other third person benefit by their agreement.

Application of the factors that courts use to determine whether a third party is an intended beneficiary who can enforce a contract between the contracting parties or an incidental beneficiary who has no rights under the contract indicates that the buyer is not an intended beneficiary. The agreement did not:

(i) expressly designate a third party;
(ii) indicate that performance was to be made directly to a third party; or
(iii) state that a third party had any rights under the contract.

Also, there is no relationship between the buyer and either the homeowner or the real estate broker from which it could be inferred that either the homeowner or the real estate broker wished to make the agreement for the buyer’s benefit.

Thus, the buyer is not an intended beneficiary of the agreement between the homeowner and the real estate broker and cannot recover on that agreement.

How well did you know this?
1
Not at all
2
3
4
5
Perfectly
67
Q

A member of an out-of-town yacht club agreed
to buy a luxury boat from a local yachtsman, a
boat which the parties referred to as “the ‘Lady’
boat” throughout their preliminary negotiations.
The yachtsman owned two boats—“Lady Be
Good” and “Lady Luck.” The yachtsman’s intent
was to sell “Lady Luck,” while the club member
intended to buy “Lady Be Good,” which the
club member knew was once owned by the club
member’s favorite celebrity. The club member
was unaware of the existence of “Lady Luck.”
The written contract for the yachtsman’s “Lady”
boat included a sale price of $100,000. When the
yachtsman delivered “Lady Luck” to the club
member’s slip, the club member saw that it was
not the boat he had intended to purchase and
refused to accept delivery. If the club member sues the yachtsman for specific performance to compel him to deliver “Lady Be Good,” and the yachtsman countersues to compel the club member to accept delivery of “Lady Luck,” who should prevail?

(B) The club member, because the yachtsman
knew that he had two “Lady” boats at the
time the contract was formed.
(C) Neither party, because an ambiguity existed
at the time the contract was formed, and the
parties did not intend the same meaning.

A

(B) The club member should prevail, because the yachtsman knew that he had two “Lady” boats and the club member did not.

This is a case of ambiguous contract language, because the expression of the parties’ agreement (here, to purchase the yachtsman’s “Lady” boat) is open to two different interpretations.

In cases where one party is aware of the ambiguity and the other party is not at the time of contracting, a contract will be enforced according to the intent of the party who was unaware of the ambiguity.

In this case, the yachtsman owned two boats, each with the word “Lady” in its name, but he intended to sell the boat known as “Lady Luck.” The club member, on the other hand, intended to buy the boat known as “Lady Be Good” and was unaware that the yachtsman owned a second boat called “Lady Luck.” The club member did not know that an ambiguity existed at the time the contract was formed, while the yachtsman, of course, knew that he owned two boats with the word “Lady” in the name, and thus knew of the ambiguity. Therefore, the contract will be enforced in accordance with the club member’s intention to purchase “Lady Be Good,” because he was the party who was unaware of the ambiguity.

(C) is incorrect because an ambiguity alone does not void a contract; there are different rules as to
whether a contract containing an ambiguity is or is not voided. It depends on who knew of the
ambiguity at the time the contract was formed.

If neither party, or if both parties, are aware of the ambiguity, no contract would be formed *unless both parties intended the same meaning.*

Here, because one party knew of the ambiguity and the other did not, the contract is still enforceable.

How well did you know this?
1
Not at all
2
3
4
5
Perfectly
68
Q

A steelmaker purchased a tube rolling machine from a manufacturer of heavy machinery. The machine was sold unassembled for a price of $150,000, with $25,000 payable on delivery and the balance ($125,000) to be paid in 10 monthly installments of $12,500 each. After the machine parts were delivered, the steelmaker contacted an assembly company that specialized in assembly and installation of large and complex manufacturing machinery, and told the company that the machinery had to be up and running within 45 days, or the steelmaker would be in breach of a major contract that it relied on for much of its current revenue. The company
agreed, in a written contract with the steelmaker,
to assemble and install the tube rolling machine
within 45 days at a price of $15,000. Two weeks later, the manufacturer that sold the tube rolling machine to the steelmaker learned that the assembly company was planning to stop work, due to a strike by its labor union. The manufacturer orally offered the assembly company a $3,500 bonus if it would agree to
finish the job for the steelmaker. The company accepted the manufacturer’s promise and
completed the assembly and installation of the
tube rolling machine with supervisory personnel
within the 45-day time limit set in the agreement
between the company and the steelmaker.
However, the manufacturer refused to pay the
assembly company the $3,500 bonus, so the
company sued the manufacturer. Which of the following would be the assembly company’s strongest argument to prevail?

(A) The assembly company owed the manufacturer no preexisting duty to complete the
job for the steelmaker, and such completion was sufficient bargained-for consideration for the manufacturer’s promise to pay the additional $3,500.
(B) Because the $3,500 payment was characterized as a “bonus,” no further consideration was required and the manufacturer is bound to its promise.

(C) The assembly company would not have
completed the job for the steelmaker within the time limit except in reliance on the manufacturer’s promise to pay the additional $3,500.

A

(A) The assembly company’s best argument is that it owed the manufacturer no preexisting duty to complete the job, and such completion was sufficient bargained-for consideration.

Generally, a promise is unenforceable unless it is supported by consideration; thus, for the manufacturer’s promise to be enforceable, there must be consideration supporting it. Consideration is defined as a bargained-for exchange of something of legal value. Most courts hold that the thing exchanged will have legal value if it causes the promisee to incur a detriment. A minority of courts hold that a benefit to the promisor is also sufficient.

Thus, the company’s best argument would be one that includes the idea that it incurred a bargained-for detriment, and this is reflected by (A).

The problem with (A) is the preexisting legal duty rule. Traditionally, courts have held that performance of an existing legal duty is not sufficient consideration. However, the rule is riddled with exceptions, and one exception recognized in most jurisdictions applies when, as here, the preexisting duty is owed to someone other than the promisor. Thus, (A) is the best argument because it provides for a full contract recovery.

(B) is wrong because merely identifying a promise to pay as a “bonus” does not obviate the need for consideration. For a promise to be enforceable, there must be consideration.

(C) is wrong because mere reliance on a promise is not enough to make a contract enforceable.
For reliance to provide a substitute for consideration, under the doctrine of promissory
estoppel, the promisor must reasonably expect that its promise will induce reliance, and such reliance must reasonably be induced.
However, the promise will be enforceable only to the extent necessary to prevent injustice.

Here, because the company had a duty to complete the work even without the manufacturer’s promise, there is no indication that justice would require payment of the $3,500; there is nothing in the facts to show the company incurred more costs, etc. Thus, the recovery to the company under a promissory estoppel theory would undoubtedly be less than the
contract recovery possible under (A).

How well did you know this?
1
Not at all
2
3
4
5
Perfectly
69
Q

A man fed a lost cat and returned it to its owner. She was so happy to see her cat that she promised to leave the man money in her will.
When the cat owner died a few days later, the
man visited the owner’s daughter demanding his share of the estate. All of the cat owner’s bank accounts had been held in joint tenancy with her daughter. The daughter reluctantly signed a document stating that she would pay the man $500 in exchange for his agreement not to sue her mother’s estate. She later had second
thoughts and refused to pay the man on the
ground that there was no consideration for the
agreement. Besides the consideration stated in the daughter’s written instrument, what other fact would strengthen the man’s claim?

(A) He never would have fed and returned the
cat had he known that he would not receive
any payment for his efforts.
(B) He believed he could sue the cat owner’s
estate.
(C) The majority of the funds in the cat owner’s
bank account were royalties from a series of
television commercials starring the cat.
(D) The cat owner’s promise to him was in
writing.

A

(B) If the man had a reasonable (i.e., good faith) belief in the enforceability of his claim, his surrender of the claim is valid consideration.

(A) is wrong because his motive for feeding and returning the cat is immaterial.

(C) is wrong. The source of the cat owner’s money is irrelevant. In addition, the accounts were held in joint tenancy and any interest the cat owner may have had in the funds ended when she died.

(D) is wrong because there was no consideration given for the cat owner’s promise and the fact that it was in writing does not change the lack of consideration.

How well did you know this?
1
Not at all
2
3
4
5
Perfectly
70
Q

Two neighbors owned summer homes adjacent
to each other on the lake. After a week-long
stay by the son of one of the property owners,
the neighbor called the owner and said that his
boat dock had been badly damaged and was
told by another resident that the owner’s son and
some friends had gotten drunk and accidentally
crashed their boat into his dock. The owner was
surprised at the accusation because he was sure
that if his son had caused the damages, he would
have told him. However, he did not want to get
into a dispute with his neighbor, so he told his
neighbor that he would have the dock repaired
and pay for the repairs if the neighbor agreed not
to bring a claim against his son for the damage
to the dock. The neighbor agreed, and the owner
hired a local carpenter to do the work. Later,
however, the owner discovered that his son
did not damage the dock because the damages
occurred after his son had returned to college.
Is the owner obligated to pay for the repairs?

(A) No, because the owner never really believed
that his son caused the damage.
(B) No, because his son in fact did not cause
the damage.
(C) No, because the neighbor was wrong when
he accused his son of causing the damage
and it would be unfair to enforce an agreement
when there was a mutual mistake of
fact.
(D) Yes.

A

(D) Modern courts would hold that a promise to forbear suit on a claim that the promisor honestly and reasonably believes to be valid is good consideration to support an agreement, even if the claim ultimately turns out not to be valid.

Hence, (A) and (B) are wrong. (C) is wrong because mutual mistake is not a defense when the adversely affected party bore the risk that the parties’ assumption was mistaken. To be a defense, it must be a true mistake, not merely an uncertainty.
Here, the owner always had the right to investigate the truth of the facts before he agreed to pay for the dock.

How well did you know this?
1
Not at all
2
3
4
5
Perfectly
71
Q

A general contractor about to bid on a construction job for an office building invited a carpenter and several others to bid on the carpentry work. The carpenter agreed to bid if the general contractor would agree to give the carpenter the job provided that his bid was lowest and the general contractor was awarded the main contract. The general contractor so agreed. The carpenter, incurring time and expense in preparing his bid, submitted the lowest carpentry bid. The general contractor used the carpenter’s bid in calculating its own bid, which was successful.

Which of the following best supports the carpenter’s position that the general contractor is obligated to award the carpentry subcontract to the carpenter?

A. The carpenter detrimentally relied on the general contractor’s conditional promise in preparing his bid.

B. The carpenter gave consideration for the general contractor’s conditional promise to award the carpentry subcontract to the carpenter.

A

(B) is correct.

The carpenter’s bid was consideration for the general contractor’s promise to award the carpentry subcontract to the carpenter if his bid was the lowest and the general contractor was awarded the main contract. Thus, the general contractor and the carpenter formed a contract.

Two elements are necessary to constitute consideration.

First, there must be a bargained-for exchange between the parties; and second, that which is bargained for must constitute a benefit to the promisor or a detriment to the promisee.

The carpenter and the general contractor agreed that the carpenter would supply a bid that the general contractor could use in its own bid. The carpenter’s bid was bargained for and was a benefit to the general contractor, so it constitutes consideration sufficient to support the general contractor’s conditional promise to award the subcontract to the carpenter.

Conditional promises are enforceable, but the duty to perform does not become absolute until the condition has been met or is legally excused. The conditions in this contract were met-the carpenter’s bid was the lowest and the general contractor was awarded the main contract. Thus, the general contractor is under a duty to perform his promise to award the subcontract to the carpenter.

(A) is not the best answer. The carpenter is seeking specific performance of their agreement so that he will be awarded the carpentry subcontract. If the carpenter uses a detrimental reliance or promissory estoppel argument, he would be conceding that he gave no consideration and there is no contractual obligation, but he should be awarded damages to prevent injustice. Courts will often limit damages under this theory to reliance damages, which could be much less than the value of the subcontract.

How well did you know this?
1
Not at all
2
3
4
5
Perfectly
72
Q

If the property owner breaches the contract during construction, the builder is entitled to …

A

… any profit he would have derived from the contract plus any costs he has incurred to date.

If the builder has mitigated his damages, any losses that are avoided must be subtracted from this amount.

How well did you know this?
1
Not at all
2
3
4
5
Perfectly
73
Q

A contractor agreed to remodel a homeowner’s garage for $5,000. Just before the parties signed the one-page written contract, the homeowner called to the contractor’s attention the fact that the contract did not specify a time of completion. The parties orally agreed but did not specify in the contract that the contractor would complete the work in 60 days, and then they both signed the contract. The contract did not contain a merger clause. The contractor failed to finish the work in 60 days. The homeowner has sued the contractor for breach of contract.

Is the court likely to admit evidence concerning the parties’ oral agreement that the work would be completed in 60 days?

A. No, because the court must ascertain the meaning of the agreement from the terms of the written contract.

B. No, because the oral agreement was merely part of the parties’ negotiations.

C. Yes, because the contract is ambiguous.

D. Yes, because the time limit is an additional term that does not contradict the partially integrated written contract.

A

(D) is correct. The time limit is an additional term that does not contradict the partially integrated written contract, and evidence concerning it will be admissible.

When the parties to a contract express their agreement in a writing with the intent that it embody the full and final expression of their bargain, the writing is an “integration,” and under the parol evidence rule, admissibility of evidence seeking to vary its terms is limited.

If the writing is only a partial integration, and not a complete embodiment of the parties’ intentions, under the parol evidence rule, it cannot be contradicted, but it may be supplemented by proving up consistent additional terms.

Here, the writing likely was only a partial integration - it was only one page, it did not include a merger clause (inclusion is strong evidence the parties intended the writing to be a full and final integration), and it did not specify a time of completion. The parties did not intentionally leave out the time of completion, it just wasn’t brought to their attention until just before signing. At that point, the parties orally agreed to that supplementary term. Therefore, parol evidence would be allowed to supplement the terms of the partial integration.

(A) is incorrect. If the writing here had been a fully integrated contract, the parol evidence rule would limit the terms of the contract to the four corners of the document. However, even in that case, a court is not limited to the document itself in interpreting its terms. For example, the court can look to usage of trade and course of dealing to interpret the meaning of a contract.

(B) is incorrect. The agreement as to the time of completion was not part of the parties’ negotiations. In fact, the parties failed to discuss the matter until it was called to their attention when they had finished negotiations and were about to sign the written contract. At that time, the parties orally agreed to the 60-day deadline, which is a consistent additional term.

(C) is incorrect. The written contract was not ambiguous; rather, it was missing a time for completion.

74
Q

A toy collector had purchased 10 antique toys over the last several years and had had them restored by an expert in toy restoration. On June 1, the collector sent the 11th antique toy to the expert with a signed note that read: “Here is another toy for you to restore. As with all prior jobs, I will pay $500 for the work, but no more.” On June 4, after receipt of the collector’s June 1 note and the toy, the expert began restoring the toy. On June 6, the collector unexpectedly died. On June 7, unaware of the collector’s death, the expert sent the collector a note that stated that the restoration work had begun on June 4. The following day, the expert learned of the collector’s death.

Does a contract exist that binds the expert and the collector’s estate?

A. Yes, because the expert sent the June 7 note before learning of the collector’s death.

B. Yes, because the offer was accepted before the collector’s death.

C. No, because the collector died before the expert sent the June 7 note.

D. No, because the offer lapsed when the collector died.

A

(B) is correct. The offer was accepted before the collector’s death.

The collector’s letter was an offer. It created a reasonable expectation in the expert that the collector was willing to enter into a contract on the basis of the offered terms. Unless an offer specifically provides that it may be accepted only through performance, it will be construed as an offer to enter into a *bilateral contract* and may be accepted either *by a promise to perform or by the beginning of performance.*

Here, the expert began performance and, thus, accepted the offer when he began restoring the toy on June 4 - two days before the collector died on June 6th.

(A) is incorrect. The collector’s offer was accepted on June 4 when the expert began performance. Whether the expert knew of the collector’s death on June 7 is not relevant. Indeed, if the expert had not already accepted the offer before the collector’s death, the offer would have terminated on the collector’s death by operation of law, even if the expert did not know of the death.

(C) is incorrect. The June 7 note was not an acceptance. The acceptance had already occurred on June 4 when the expert began performance by beginning to restore the toy.

(D) is incorrect. If either of the parties to a proposed contract dies prior to acceptance, the offer is terminated. Here the collector’s offer had already been accepted at the time of the collector’s death. Thus, there was no lapse.

75
Q

A brick mason was hired by a builder under a written one-year contract, at an annual salary of $45,000, with employment to begin on March 1. Because the builder was unable to secure enough building contracts to keep all its employees busy during the season beginning March 1, it notified the brick mason on February 15 that it could not afford to employ him as a mason. At the same time, however, the builder offered to employ the mason, for the same contract period, as a night guard at an annual salary of $25,000. The mason declined the offer and remained unemployed during the year. No employment for brick masons was available in the community during the year, but the mason could have obtained other employment as a day laborer that would have paid up to $25,000 a year.

At the end of the year, in an action against the builder for breach of contract, how much, if anything, is the mason entitled to recover?

A. $20,000 (the $45,000 contract price less the $25,000 the mason could have earned in other employment).

B. $45,000 (the contract price).

C. Nothing, because the builder did not act in bad faith when it discharged the mason.

D. Nothing, because the mason did not mitigate his damages.

A

(B) is correct.

When an employer breaches an employment contract, the standard measure of the employee’s damages is the full contract price. However, the nonbreaching party cannot recover damages that could have been avoided with reasonable effort. If the breaching employer can prove that a comparable job in the same locale was available, then contract damages against that breaching employer for lost wages will be reduced by the wages that the plaintiff would have received from that comparable job.

Here, there was no comparable employment available in the community for the mason. The only jobs available, a night guard or a day laborer, were not the same type of work as a brick mason and were at a significantly lower salary.

(A) is incorrect. The mason was not obligated to accept employment in a different field at a much lower salary to mitigate his damages. (C) is incorrect. The builder is liable for damages because it breached the employment contract with the mason. The builder’s reasons for breaching, even if in good faith, do not excuse the breach. (D) is incorrect. No comparable employment was available in the community during the year and, as stated above, the mason was not obligated to accept employment in a different field at a much lower salary to mitigate his damages.

76
Q

On December 6, the owner of an electronics store sent a written request to a computer manufacturer asking for the price of a certain laptop computer. The manufacturer sent a written reply with a catalog listing the prices and descriptions of all of his available computers. The letter stated that the terms of sale were cash within 30 days of delivery. On December 14, by return letter, the store owner ordered the computer, enclosing a check for $4,000, the listed price. Immediately on receipt of the order and check, the manufacturer informed the store owner that there had been a pricing mistake in the catalog, which should have quoted the price as $4,300 for that computer. The store owner refused to pay the additional $300, arguing that his order of December 14 in which the $4,000 check was enclosed was a proper acceptance of the manufacturer’s offer.

In a suit for damages, will the manufacturer prevail?

A

The store owner’s December 14 letter was an acceptance. Whether the letter was an acceptance depends on whether the manufacturer’s letter was an offer, because an acceptance is a manifestation of assent to an offer. For a communication to be an offer, it must create a reasonable expectation in the offeree that the offeror is willing to enter into a contract on the basis of the offered terms. There must be a promise, undertaking, or commitment to enter into a contract with certain and definite terms. Courts usually hold that if a statement is made broadly, such as in an advertisement or catalog, it will not constitute an offer because it is not reasonable to expect that the sender intended to make offers to all who received the advertisement; rather, the courts usually find such advertisements to be invitations seeking offers. However, price quotations may be considered as offers if given in response to a specific inquiry. The courts will look to the surrounding circumstances, and here a court would probably determine that the catalog that the manufacturer sent was an offer because it was sent in response to the store owner’s specific inquiries about prices on a specific computer and it included delivery terms and conditions of sale. (A) is incorrect because although the letter called for payment in cash, tender by check is sufficient unless the seller demands legal tender and gives the buyer time to obtain cash. Moreover, because the contract called for payment within 30 days of delivery, even if the check was not sufficient, the store owner still had time under the contract to obtain cash. (B) is incorrect because the mistake was unilateral. Generally, a unilateral mistake will not be grounds to rescind a contract unless the nonmistaken party knew or should have known of the mistake. Here, nothing in the facts indicates that the store owner knew of the mistake, and the mistake was not so large that it could be said that he should have known of it. (C) is incorrect because, as explained above, the manufacturer’s catalog was sent in response to the store owner’s request for information and his terms for sale constituted an offer.

77
Q

A police officer spent several hours using binoculars to observe an older man loitering on a college campus. The man, who was shabbily dressed and carrying a backpack, would approach certain students as they walked by him, and after a brief conversation with them, discreetly pass the students a small envelope in exchange for cash. The officer stopped the man under suspicion that he was dealing drugs. The man was not dealing drugs, but instead had been soliciting donations for a radical political group. The man grew irate when the officer opened one of the envelopes in question and discovered that they only contained literature about the group. The officer then frisked the man and discovered an illegal weapon taped to his leg. The officer immediately arrested the man.

Which of the following best describes the situation?

A

The stop was lawful but the patdown search was unconstitutional. A police officer has the authority to briefly detain a person for investigative purposes if he has a reasonable suspicion supported by articulable facts of criminal activity. Here, the officer watched the man engage in what reasonably appeared to be drug transactions, and he was justified in stopping and detaining the man to investigate. However, a police officer may pat down a detained person only if the officer has a reasonable suspicion to believe that the detainee is armed and dangerous. Here, the man grew irate when he was stopped, and he belonged to a “radical” group, but these facts are not enough to give the officer any reason to believe that the man was armed. Therefore, the patdown was unconstitutional, and the evidence found as a result must be suppressed. (A) is incorrect. Police generally need not obtain a warrant before arresting a person in a public place, even if they have time to get a warrant. A police officer may arrest a person without a warrant when he has probable cause to believe that a felony has been committed and that the person before him committed it. (B) is incorrect. If during an investigatory detention, the officer develops probable cause for arrest, the officer can proceed on that basis. Although the officer was incorrect in his initial suspicions of drug dealing, the illegal weapon he discovered during the patdown was sufficient to establish probable cause for the man’s arrest. (D) is incorrect because, as stated above, an officer may not pat down a detainee for weapons absent a reason to believe the detainee is armed and dangerous.

78
Q

A department store buyer and a manufacturer of food processors entered into a written contract whereby the manufacturer would sell to the buyer 50 of its top-of-the-line models for $100 each. When the delivery arrived on May 15, several days early, the buyer noticed that the food processors were a different model that did not have all of the features as the top-of-the-line model that was ordered. The buyer contacted the manufacturer and told him that he was rejecting the food processors that were delivered to him and expected the manufacturer to send 50 top-of-the-line models immediately. The manufacturer replied that because of a backlog of orders that had not yet been filled, the top-of-the-line models could not be delivered until August 15. Because the department store had contracted with a restaurant to deliver three top-of-the-line models by May 31, the buyer delivered three of the nonconforming food processors along with a promise to replace them with three top-of-the-line models in mid-August. The buyer returned the remaining food processors to the manufacturer.

How much could the department store recover from the manufacturer for the three food processors that it delivered to the restaurant?

A

The department store was entitled to recover contract damages from the manufacturer for the three food processors that it accepted. If the buyer accepts goods that breach one of the seller’s warranties, the basic measure of damages is the difference between the value of the goods as delivered and the value they would have had if they had been according to the contract, which is best stated by choice (C). The department store’s acceptance of the three food processors did not waive its right to collect damages for the defect in quality. Thus, (A) and (B) are wrong. Having accepted the nonconforming food processors, the department store’s damages would be the difference between the value of the food processors as received and what they would have been worth if they had been as warranted, plus foreseeable incidental and consequential damages. (D) is wrong because the measure of damages is based on market value rather than cost. Also, the agreement with the restaurant was to accommodate the department store only and was not foreseeable by the manufacturer.

79
Q

What does term “F.O.B.” mean?

A

That term means “free on board,” and it obligates the seller to get the goods to the location indicated after the term. [UCC §2-319(1)]

i.e.

A jeweler sent a fax to a gold dealer offering to sell the dealer 100 ounces of gold at $900 per ounce. The dealer immediately responded via fax, “What are your terms of shipment?” The jeweler faxed back, “F.O.B. my store.” The dealer faxed back, “I accept.”

Who must pay the freight charge from the shop to the dealer?

The gold dealer, because of the F.O.B. term.

“F.O.B. [jeweler’s] shop,” means the jeweler is not obligated to pay for costs of shipment beyond his shop.

80
Q

A horse breeder offered to sell a colt to his neighbor and they agreed on a purchase price. The horse breeder subsequently received a letter from the neighbor thanking him for the sale and summarizing their agreement. The letter contained the neighbor’s alleged signature. When the horse breeder attempted to set up transfer of the colt, the neighbor denied that she agreed to purchase it. In a breach of contract action against the neighbor, the horse breeder offers into evidence the letter. The horse breeder testifies that he is familiar with the neighbor’s handwriting and recognizes the signature on the letter as being hers.

Assuming appropriate objection by the neighbor, who claims that she did not sign the letter, how should the trial court rule on the admissibility of the letter?

A Exclude the letter for lack of foundation because lay opinion testimony regarding handwriting identification is not admissible.

B Exclude the letter unless its authenticity is established by a preponderance of the evidence.

C Admit the letter as authentic and instruct the jury accordingly.

D Admit the letter but instruct the jury that it is up to them to decide whether the letter is authentic.

A

The court should admit the letter and instruct the jury that it is up to them to decide whether the letter is authentic. Before a writing may be received in evidence, it must be authenticated by proof showing that the writing is what the proponent claims it is. All that is necessary is proof sufficient to support a jury finding of genuineness. The authenticity of a document is a preliminary fact to be decided by the jury. Here, the horse breeder’s testimony that he is familiar with the neighbor’s handwriting and that he recognizes the signature on the letter to be that of the neighbor is sufficient to support a jury finding of genuineness. Thus, the letter should be admitted and authenticity should be left to the jury to decide.

(A) is wrong because a lay witness who has personal knowledge of the handwriting of the supposed writer may state his opinion as to whether the document is in that person’s handwriting.

(B) is wrong because authentication of documentary evidence requires only enough evidence to support a jury finding that the matter is what its proponent claims it is. It is not required that the proponent establish its genuineness by a preponderance of the evidence.

(C) is wrong because, as noted above, where there is a dispute as to the authenticity of a document, the issue of authenticity is a fact determination for the jury, not the judge, to decide.

81
Q

On February 1, a national department store chain entered into a written agreement with a canoe manufacturer providing that the manufacturer would sell the department store any quantity of 16-foot aluminum canoes that the department store desired at a price of $250 per canoe, deliveries to be made 30 days after any order. The agreement was signed by authorized agents of both parties. On March 1, the department store sent the manufacturer an order for 500 canoes to be delivered in 30 days. The manufacturer immediately e-mailed the department store a confirmation of the order. Ten days later, the department store sent the manufacturer an order for an additional 500 canoes, to be delivered in 30 days. Five days after receiving the department store’s second order, the manufacturer e-mailed the department store and explained that a large sporting goods chain was willing to purchase all of the manufacturer’s output of 16-foot canoes at $275 per canoe and that the manufacturer would be unable to fill any of the department store’s orders.

The department store found another canoe manufacturer willing to provide it with 16-foot aluminum canoes for $280 per canoe and on April 15 filed an action against the manufacturer seeking damages for the manufacturer’s failure to deliver the 1,000 canoes ordered.

How should the court rule?

A The department store is not entitled to any damages because no contract was formed by the parties’ communications.

B The department store is entitled to cover damages of $30 per canoe only for 500 canoes but is not entitled to any damages for breach of the duty of good faith.

C The department store is entitled to cover damages of $30 per canoe for 1,000 canoes but is not entitled to any damages for breach of the duty of good faith.

D The department store is entitled to punitive damages equal to the lesser of 10% of the total sale price or $500 in addition to any cover damages that are due because the manufacturer breached the duty of good faith.

A

This question is best answered by eliminating the incorrect choices first.

(A) is incorrect. A contract was formed here for 500 canoes.

The original “agreement” between the parties was nothing more than an invitation seeking offers. It did not create a contract between the department store and the manufacturer because it was illusory—an agreement to buy only what is desired is not consideration. The “agreement” probably does not even qualify as an offer. An offer must express a commitment to conclude a bargain on the offered terms. Absent some quantity limitation, a court would probably find the “agreement” here too vague to constitute an offer; otherwise, the manufacturer could be committing itself to sell more canoes than it can supply. Thus, the department store’s first order will be construed as an offer, and the manufacturer’s confirmation will be construed as an acceptance of the offer, thus creating a contract.

(C) is incorrect because there was no acceptance of the department store’s second offer. If the original agreement did not create a contract, the second order must be construed as an offer. The manufacturer did nothing to accept the department store’s second offer. The manufacturer’s failure to reject the offer until five days after it was made does not constitute an acceptance. Therefore, no contract was formed for the additional 500 canoes.

(D) is incorrect because although it is true that the Uniform Commercial Code (“UCC”) imposes a duty of good faith on all parties, and failure to deliver under a contract simply because a better price can be obtained might violate this duty, the UCC does not provide for punitive damages for breach of this duty. When a seller fails to deliver goods, one remedy available to the buyer is cover damages—the difference between the contract price and the price of substitute goods. Because the manufacturer had agreed to sell the department store 500 canoes and failed to deliver, the department store reasonably bought replacement goods for $30 more per unit and is entitled to recover the additional $30 per unit.

82
Q

In most cases, the plaintiff’s standard measure of damages will be based solely on an “expectation” measure, i.e., sufficient damages for him to buy a substitute performance.

A reliance measure of damages, on the other hand, awards the plaintiff the cost of his performance, i.e., his expenditures in performing his duties under the contract.

In certain situations, an award of compensatory damages will contain both an expectation and a reliance component.

What is the formula for construction contract?

A

In a construction contract, if the owner breaches the contract after the builder has already begun his performance, the builder will be entitled to any profit he would have derived from the contract plus any costs he has incurred to date. This formula contains an expectation component (the profit the builder would have made) and a reliance component (the cost incurred prior to the breach).

83
Q

A hardware store ordered 200 cans of wood stain in various shades. The written contract between the store and manufacturer provided that 100 cans of stain would be delivered on April 30, and the remaining 100 cans would be delivered on June 30. Payment would be due at the time of each delivery. The first shipment arrived on April 30. Sales of the stain were brisk, but 25 customers almost immediately returned their stain, complaining that it was not the color indicated on the can. The store owner called the manufacturer and informed it of the problem. The manufacturer truthfully told the owner that they had had a small problem with their labeling machine and a few cans in the store owner’s lot must have been mislabeled before they caught the problem. The manufacturer offered to replace all 100 cans from the original order. The store owner refused the offer and told the manufacturer not to deliver the second lot, because he could no longer trust the manufacturer. The owner was very sensitive to the hardware store’s good reputation, which he felt was harmed by this incident.

If the manufacturer brings a claim of breach regarding the second shipment which was due on June 30, how will the court likely rule?

A The buyer had the right to cancel the second shipment, because of legitimate fears that it would contain the same defects as the first shipment.

B The buyer had the right to cancel the second shipment, because the first delivery was defective.

C The buyer did not have the right to cancel the second shipment, because the defects in the first shipment did not substantially impair the value of the entire contract.

D The buyer did not have the right to cancel the second shipment, because he failed to make a demand upon the manufacturer for adequate assurances that the second shipment would be free of defects.

A

The buyer did not have the right to cancel the second shipment, because the defects in the first shipment did not substantially impair the value of the entire contract.

This case involves an installment contract, i.e., the contract authorizes or requires deliveries in separate lots, and the sale of goods, so Article 2 of the UCC applies. Under Article 2, a buyer may declare a total breach of an installment contract only if the defect substantially impairs the value of the entire contract. [UCC §2-612] The problem with the first shipment of the stain was discovered and corrected by the manufacturer. The manufacturer offered to cure the defect in the first shipment. In whole, the defect in the first shipment did not substantially impair the value of the entire contract.

(A) is incorrect because legitimate fears, alone, are not enough to justify anticipatorily repudiating a contract, as the buyer did here. Anticipatory repudiation occurs when a promisor, prior to the time set for performance of his promise, indicates that he will not perform when the time comes. If, as here, the promisor is unsure of whether the other party will fulfill his contract obligations, the promisor may seek adequate assurances that performance will be forthcoming. In this case, the manufacturer already provided the buyer with such assurances, which fell on deaf ears. Because the manufacturer assured the buyer and, more importantly, had already corrected the problem at its factory before the second shipment was due, the store owner’s fears would not justify canceling the second shipment.

(B) is incorrect because, as discussed above, the defects in the first delivery would not warrant cancellation of the entire installment contract unless they substantially impaired the value of the entire contract, which they did not.

(D) is incorrect because, as explained, the manufacturer had already given adequate assurances to the buyer.

84
Q

If offers stating precisely the same terms cross in the mail, they do (not) give rise to a contract despite the apparent meeting of the minds.

A

do not

i.e.

The shop owner and the antique lover each sent offers setting the price of the ensemble at $22,000. Despite the fact that these offers were identical, there is no mutual assent without at least one of the parties manifesting acceptance of the terms of the offer, and communicating that acceptance to the other. We are told that this has not yet happened even though the shop owner and the antique lover both have received the letters. Consequently, although there is an apparent meeting of the minds as to price, there has not been a sufficient objective manifestation of this agreement as to denote a mutual assent.

85
Q

A businesswoman entered into a written contract with a general contractor to build a studio and broadcast transmitter for $3 million by July 1. Among his tasks, the contractor was to install underground cables and fiberoptic lines necessary to broadcast.

When digging the deep trench necessary to lay the conduit containing the fiberoptic lines, the contractor encountered a stretch of extremely soggy soil. This was an indication that an offshoot of the nearby city’s aquifer underlay the property. This was not indicated on any of the geological survey maps available in the office of the county recorder of deeds. The contractor told the businesswoman that it would cost an additional $50,000 to lay the conduit through that stretch of soil. The businesswoman had already launched an advertising campaign indicating that the station would begin broadcasting on July 4, which was rapidly approaching. Therefore, when the contractor threatened to quit the job without the additional $50,000, the businesswoman reluctantly agreed orally to the contractor’s demand as long as he promised that all of the work would be completed by the middle of June. The contractor agreed, proceeded to lay the conduit, and completed building the studio and transmitter by June 15. The businesswoman paid the contractor $3 million, but when the contractor demanded $50,000 more, she refused to pay it. The contractor sues the businesswoman for the $50,000.

Who will prevail?

A The businesswoman, because the oral modification was not effective to alter the prior written agreement.

B The businesswoman, because no valid consideration was provided for the agreement to pay the additional $50,000.

C The contractor, because the modification was supported by consideration.

D The contractor, because he detrimentally relied on the businesswoman’s promise to pay the additional $50,000.

A

The original contract was modified by the parties, and this modification discharged the payment term of the original contract ($3 million) and replaced it with a new payment term ($3,050,000).

If a contract is subsequently modified by the parties, this will serve to discharge those terms of the original contract that are the subject of the modification. Generally, a modifying agreement must be mutually assented to and supported by consideration. In most cases, consideration is found to be present in that each party has limited her right to enforce the original contract as is. Generally, if a modification will benefit only one of the parties, it may be unenforceable without some consideration being given to the other party. If, however, a promisee has given something in addition to what he already owes in return for the promise he now seeks to enforce, or has in some way agreed to vary his preexisting duty, there is consideration.

Here, the businesswoman agreed to pay the contractor an additional $50,000 and the contractor agreed to complete the work early—by mid-June instead of July 1. Thus, there was sufficient consideration to support the modification.

Note that this modification may have been enforceable without consideration. Under the modern view, a modification is enforceable without consideration if the modification is fair and equitable in view of the unanticipated circumstances.

Here, the aquifer did not appear on city surveys, so the soggy soil was not anticipated. The $50,000 represented the electrician’s additional cost, so the modification seems fair.

(A) is incorrect because neither the Statute of Frauds nor the parol evidence rule affects the validity of the oral modification. The Statute of Frauds does not require a writing for the modification of a construction contract, and parol evidence can be offered to show subsequent modifications of a written contract. (B) is incorrect because, as explained above, the contractor’s agreement to vary his contractual duty by promising to perform all of the work by a date earlier than that originally agreed to constitutes consideration sufficient to support the businesswoman’s promise to pay the additional $50,000. (D) is incorrect because the electrician was already obligated to perform; i.e., he was already obligated to do the job he did in reliance on the businesswoman’s promise. Therefore, fulfilling his duty cannot be detrimental reliance

86
Q

A masked magician contracted with a casino
to perform magic tricks six nights a week on the
casino floor for $2,000 per week. The contract
did not contain an assignment provision. One
weekend, the magician came down with flu, so
he asked his apprentice to perform in his place.
The two signed a written assignment agreement
that the apprentice would fully take the
magician’s place under the terms of the contract
for the next week. Excited about the opportunity,
the apprentice told the casino’s manager of the
deal, but no one else knew. During that week,
the apprentice wore the magician’s costume and
mask and performed all of the magician’s tricks.
No one in the casino noticed the difference.
When the magician regained his health, he and
the apprentice had a falling out and the magician
refused to pay the apprentice. The apprentice
went to the casino with the assignment agreement and demanded his pay for the prior week’s performances. The magician claimed that he was still the one entitled to payment under the
contract. Who should the casino pay for the prior
week’s performances?

(A) The apprentice, because he told the manager
about the arrangement.

(C) The magician, because he is the original
party to the contract with the casino and no
one in the casino could tell that it was not
him performing.

A

(A) The casino should pay the apprentice for his performance because the assignment agreement
established privity of contract with the casino.

While most contractual rights may be assigned
and contractual duties delegated, duties involving personal judgment or skill typically cannot be delegated. The type of personal services contract involved here would not be delegable, but the casino waived the nondelegability by accepting the apprentice’s performance after being apprised of the delegation. The casino manager’s knowledge will be imputed to the casino. Hence, the assignment agreement between the magician and apprentice is valid, and the casino must pay the apprentice under the contract.

(C) is incorrect because
_the effect of the assignment is to establish privity of contract between the obligor (the casino) and
the assignee (the apprentice) while extinguishing privity between the obligor and the assignor
(the magician). The assignee then replaces the assignor as the real party in interest and he alone is entitled to performance under the contract._The fact that no one could tell that the magician was not on stage does not change the legal status of the parties.

87
Q

A singer entered into a contract with a
hairstylist. The stylist agreed to accompany the
singer on her six-month world tour and to fix
her hair for each performance, in exchange for
a large fee to be paid on completion of the tour.
Two months into the tour, the stylist grew tired
of life on the road and assigned “all his rights
and duties” under the contract to his assistant.
When the singer learned that the stylist had
made the assignment and left the tour, she was
distraught, but since she needed someone to style
her hair for that night’s show she allowed the
assistant to take over. The next night, without
legal excuse, the assistant abandoned the tour,
leaving the singer without anyone to style her
hair before that night’s concert.
Which of the following legal conclusions is
correct?
(A) Both the stylist and the assistant are liable
to the singer for legal damages, if any,
caused by the assistant’s default.
(B) Only the assistant is liable to the singer for
legal damages, if any, caused by the assistant’s
default, while the stylist is not liable
because the singer allowed the assistant to
style her hair after the assignment.
(C) Only the stylist is liable to the singer for
legal damages, if any, caused by the stylist’s
default, while the assistant is not liable
to the singer because he was not a party
to a contract with the singer and thus his
services were “at will.”
(D) Neither the stylist nor the assistant are
liable to the singer for legal damages, if
any, caused by either the stylist or the assistant’s
default because enforcing a contract
for personal services would be tantamount
to involuntary servitude.

A

(A) Both the stylist and the assistant are liable on the contract.

(B) is incorrect because when contractual
duties are delegated, the delegator remains liable on the contract, even if the delegate assumes the duties. The result might be different if the obligee expressly consented to the delegation of duties and released the original obligor (a novation), but that did not happen here. Here, the singer allowed the assistant to style her hair, but that does not, without more, constitute a novation.

(C) is also incorrect. The liability of a delegate turns on whether there has been a mere “delegation” or delegation plus an “assumption of duty.”

Here the delegate, the assistant, “assumed the duties” by making a promise to perform supported by consideration (i.e., the right to collect the stylist’s fees). Thus, the nondelegating party (the singer) can sue the assistant for nonperformance.

(D) is incorrect. Contracts for personal services are not subject to specific performance, but the courts will award money damages for their breach.

88
Q

On November 7, a painter agreed with a
homeowner to paint his house for $10,000,
payment to be made upon completion of the job.
On November 14, while the job was still incomplete, the painter told her paint supplier that if he would give her the paint she needed, she would have the homeowner pay to him directly the $3,000 for paint that she owed him. The paint supplier agreed, and the painter sent the homeowner a letter setting forth this agreement. On December 1, the painter had completed the job, but the homeowner refused to pay the paint supplier any money. In a suit by the paint supplier against the homeowner, what would be the homeowner’s best defense?
(A) The paint supplier had already supplied
the paint before the agreement, and therefore
he had not relied on the homeowner’s
promise.
(B) The paint supplier was not an intended
beneficiary of the agreement between the
homeowner and the painter.
(C) The painter had not painted the house in a
proper, workmanlike manner.
(D) The painter attempted to assign her rights
before completion and an assignment to
receive money before personal services are
performed is inoperable.

A

(C) The best defense is the quality of the painter’s work.

The painter assigned part of her claim against the homeowner to the paint supplier, and, as a general rule, the assignee is subject to the
same defenses that the obligor has against the assignor.
If the painter, the assignor, had not done
the work properly, a homeowner would have a defense against her, hence he can use this defense against the paint supplier.

(A) is immaterial, because this is not a third-party beneficiary agreement, but an assignment of the right to receive money. Thus, (B) must also be ruled out.

(D) is wrong because an assignment of a future claim is not inoperable.

89
Q

An aircraft manufacturer anticipating lengthy
contract negotiations with a foreign country to
sell military aircraft and parts entered into a
written agreement with a highly skilled salesperson
to act as its agent. Under the contract
between the aircraft manufacturer and the sales
agent, the agent was required to begin meetings
with the representatives of the foreign country
on August 1. However, on July 19, the sales
agent accepted a job with a competing company,
and could not thereafter represent the aircraft
manufacturer. To fulfill her contractual obligations
with the aircraft manufacturer, the sales
agent called her friend, who was reputed to be
“the best in the business,” and orally assigned
the aircraft manufacturer contract to him for a
specified sum.
If the aircraft manufacturer does not want the
sales agent’s friend to represent it and sues the
sales agent for damages for breach, who will
prevail?
(A) The aircraft manufacturer, because the assignment
was oral.
(B) The aircraft manufacturer, because the
sales agent’s performance was personal.
(C) The sales agent, because her friend is a
better negotiator than the sales agent and
so the aircraft manufacturer would not be
damaged.
(D) The sales agent, because damages are
speculative.

A

(B) The aircraft manufacturer will prevail in its suit against the sales agent for damages because
the sales agent’s performance was personal and, therefore, is nondelegable. Where a contract is assigned in total, such as the contract here, the assignment includes a delegation of duties.
Although the general rule is that all contractual duties can be delegated to a third person,
duties may not be delegated if they involve personal judgment and skill.

Here, the sales agent’s performance as a skilled negotiator involved personal judgment and skill and, thus, may not be delegated.

(A) is wrong because delegations of contractual duties can be oral.

(C) is wrong because the damage lies in taking away the sales agent that the aircraft manufacturer has chosen to be negotiator, which is not permitted when her duties involve personal judgment and skill.

(D) is wrong because damages could be measured by the cost of hiring another negotiator of the aircraft manufacturer’s choosing. Also, the question turns on whether the sales agent’s duties are delegable, and not on how damages would be calculated.

90
Q

To satisfy customer demand for ketchup from
organically grown tomatoes, a major restaurant
chain agrees to give its ketchup manufacturer a
$4 million upfront investment in exchange for
the manufacturer’s switching all of its current
tomato farming techniques to satisfy organic
standards. The contract further states that the
restaurant chain will then be able to purchase all
its needs for organic ketchup from the ketchup
manufacturer at a certain price.
Soon after entering into this contract, the
ketchup manufacturer, without the restaurant
chain’s knowledge or assent, sold all its tomato
farms, but not its manufacturing plants, to a
large agricultural corporation. Under the terms
of this sale, the agricultural corporation agreed
to sell to the tomato grower all tomatoes grown
on the land for 20 years. T_he agricultural
corporation’s employees have no experience in
organic farming, and the agricultural corporation
has no reputation in the farming industry
as a successful tomato producer._ The restaurant
chain-ketchup manufacturer contract was silent
on the matter of the ketchup manufacturer’s
selling any or all of its business assets.
If the restaurant chain seeks an appropriate
judicial remedy against the ketchup manufacturer
for selling its tomato farms, is the restaurant
chain likely to prevail?

(A) Yes, because the sale transaction created a
significant risk of diminishing the supply
of organic ketchup that the ketchup manufacturer
will be able to sell to the restaurant
chain under the contract.

(C) No, because the ketchup manufacturer
remains in a position to perform under the
restaurant chain-ketchup manufacturer
contract.

A

(A) The restaurant chain is likely to prevail.

A contractual duty may not be delegated if performance by the delegate will materially change the obligee’s expectancy under the contract.

Here, substitution of the agricultural corporation, a company with no experience in the tomato growing business, for the ketchup manufacturer, an established entity in the business, greatly decreases the probable success of the supply of organic ketchup for which the restaurant chain, the obligee, contracted. Thus, a court should rule that the attempted delegation by the ketchup manufacturer of its duty to grow organic tomatoes for the venture is invalid. It follows that (C) is incorrect.

91
Q

The owner of a large fleet of buses contracted
in writing with a mechanic that all general
maintenance and extraordinary repairs required
for the buses would be performed by the
mechanic. The fleet owner’s wife owned and
operated a small advertising agency. _The fleet
owner included a clause in the agreement that
the mechanic would place all his ads for his
repair shop through the wife’s agency during the
one-year term of the agreement._For six months,
the mechanic dutifully placed all his ads through
the wife’s agency and informed her of his agreement
with the fleet owner. During that time, the
wife turned down work from two prospective
clients because of the time that she would have
to devote to designing and disseminating ads
from the mechanic. The mechanic then discovered
that the fleet owner was having some of his
buses repaired and maintained by other shops.
The mechanic immediately ceased placing ads
through the wife and employed another agency.
Can the wife successfully bring suit against
the mechanic to enforce the agreement?

(B) Yes, because she detrimentally relied on the
mechanic’s promise to place ads when she
refused other clients.

(D) No, because the fleet owner’s exclusive use
of the mechanic’s repair shop was a condition
for the mechanic’s duty to purchase
ads through the wife, and the fleet owner’s
actions excused the mechanic’s duty to the
wife.

A

(D) The mechanic will not be liable to the wife because the mechanic’s duty to purchase ads was
excused by the fleet owner’s failure to use the mechanic’s repair shop exclusively. The facts
indicate that the fleet owner and the mechanic entered into a valid contract that required the
mechanic to render some performance to the wife.

The wife is an intended third-party beneficiary
because

(i) she was expressly designated in the contract,

(ii) some performance is to be made
directly to her, and

(iii) she stands in such a relationship to the promisee (she is the fleet owner’s wife) that an intent to benefit her can be inferred.

She can enforce the contract because her rights
have vested; she has materially changed position in justifiable reliance on the promise by turning
down work from two prospective clients because of the time the mechanic’s ads would take.
_However, when the third-party beneficiary sues the promisor on the contract, the promisor may
assert any defenses to formation or performance that he would have been able to assert against
the promisee, including failure of a condition.
_The mechanic’s promise to perform by placing all
of his ads with the wife during the term of the agreement was dependent on the condition that the
fleet owner have all of his buses repaired by the mechanic. The fleet owner’s failure to fulfill this
condition excused the mechanic’s duty to continue to place ads with the wife, and the mechanic
will be able to assert this defense to performance against the wife.

(B) is incorrect because the wife’s detrimental reliance only establishes that her rights in the contract have vested. Thus, she can bring a contract action against the mechanic, but the mechanic will have a defense.

92
Q

A debt that a brother owed to his sister had an
outstanding balance of $4,000 when the statute
of limitations ran out. Urgently needing money
but aware that she had no legal recourse, the
sister asked her brother if he would be able to
repay any part of the balance of the loan. The
brother agreed in writing to assign to his sister
a debt of $2,000 that was owed to him by his
stepson and was coming due in a week, and the
sister accepted. The brother informed his stepson
of the assignment and instructed him to pay the
sister. When the $2,000 debt became due, the
stepson refused to pay her.
If the sister brings an action against the
stepson to collect the debt, will she likely
prevail?

(C) Yes, because the stepson may not raise the
brother’s statute of limitations defense on
the original contract.
(D) Yes, because the assignment is valid as a
novation.

A

(C) The sister will likely prevail.

While an obligor (the stepson here) may raise defenses on the obligation he owes to the assignor (the brother), he may not raise defenses that the assignor might have had against the assignee (the sister-creditor) on a different obligation as a means of avoiding his own obligation. Here, there are no apparent defenses to the stepson’s liability to the brother for the $2,000 debt, and the stepson is not entitled to assert the brother’s statute of limitations defense against the sister because the debt that the brother owed to his sister was a different obligation.

(D) is incorrect because the assignment was not a novation.
A novation substitutes a new party for an original party to the contract and requires the assent of
all parties. If valid, it completely releases the original party. Here, the stepson did not assent to
anything and no one was released, so there was no novation.

93
Q

A homeowner orally agreed to sell her land to
a buyer for $79,000. As a condition of the sale,
the buyer orally agreed to pay $5,000 of the
purchase price to a creditor of the homeowner.
The buyer asked his secretary to prepare a
formal, typewritten copy of the contract and,
in so doing, the buyer’s secretary accidentally
left out the condition to pay the $5,000 to the
creditor. Both the buyer and the homeowner read
the document before signing, but neither noticed
the oversight. The buyer refuses to pay the
$5,000 to the creditor, and the creditor sues the
buyer for the $5,000.
In the creditor’s suit against the buyer for
$5,000, who will prevail?
(A) The buyer, because an agreement to pay the
debt of another must be in writing, and the
buyer’s promise to pay $5,000 of the land
sale purchase price to the homeowner’s
creditor was not included in the written
buyer-homeowner agreement.
(B) The buyer, because a promise concerning
an interest in land must be in writing, and
the buyer’s promise to pay $5,000 of the
land sale purchase price to the homeowner’s
creditor was not included in the written
buyer-homeowner agreement.
(C) The buyer, because oral statements are
inadmissible to vary the terms of an
unambiguous writing.
(D) The creditor, because his rights as a thirdparty
beneficiary have vested.

A

(D) The creditor will prevail. Once a third-party beneficiary’s rights vest, he may enforce the contract, and here the creditor’s rights vested when he brought suit. He is seeking reformation of the
contract, because the written terms do not reflect the original agreement of the parties. Here, the
original agreement called for a $5,000 payment to the creditor. Because the buyer will not be able
to successfully claim the defense of the Statute of Frauds (see below), the creditor should recover
the $5,000.

(A) is wrong because the buyer is not promising to act as a surety when he promises to pay some of the money to the debtor. Generally, contracts do not have to be in writing to be enforceable; however, under the Statute of Frauds, certain contracts will not be enforceable unless they are evidenced by a writing signed by the party to be charged. One such contract is to pay the debt of another. In this case, the buyer is not promising to pay another’s debt, if the debtor fails to repay the debt to his creditor, but is merely paying part of the purchase price to the seller’s creditor rather than to the seller himself, pursuant to the contract. Thus, this is not a surety situation, and no writing is required on that ground.

(B) is wrong because the buyer’s oral promise to
pay part of the land sale purchase price to the debtor does not itself concern an interest in land.
Under the Statute of Frauds, a promise creating an interest in land must be in writing. An agreement
that concerns only to whom payment of part of a land sale purchase price is to be made does
not create an interest in land, but rather is a promise that is collateral to the land sale and would
Contracts - Third-Party Beneficiaries and AssignmentsAnswers Set 2 5.
not fail because it was not in writing. (C) is wrong because reformation is available even if the
written document is unambiguous, as long as it fails to reflect the terms originally agreed upon.

94
Q

The owner of farmland entered into a written
contract with a prospective tenant of the property
to erect a barn with a milking station and silo on
the property in exchange for the tenant’s taking a
five-year lease on the property. After the agreement
was signed, the tenant moved himself and
his family onto the property. The farm owner,
who had worked for many years as a skilled
building contractor before retiring, began the
work of building the barn and the silo. However,
an injury prevented him from completing the
job when it was about two-thirds done. He
told the tenant that he would see to it that the
work was completed and that he had already
entered into an oral contract with his nephew, a
novice contractor, to finish the job. However, the
nephew failed to complete the job. The tenant
brought an action against the nephew for breach
of contract.
Which would be the best defense for the
nephew?
(A) His contract was with the farm owner and
not with the tenant.
(B) The tenant furnished no consideration.
(C) His agreement with the farm owner was
oral.
(D) The building duties which the nephew took
on were not delegable.

A

(D) The nephew’s best defense is that the duties were nondelegable.

Although, as a general rule, all contractual duties may be delegated to a third person, if the duties involve personal judgment and skill, they may not be delegated.

Here, the farm owner, a skilled building contractor for many years, assigned his duties to build the barn, milking station, and silo to his nephew, a novice
contractor. The nephew could argue that because the duties to which he was assigned involved
personal judgment and skill, the assignment was not proper.

(A) is incorrect because it does not matter that the nephew’s contract was with the farm owner and not with the tenant. The tenant was an intended third-party beneficiary of their contract. An intended beneficiary can enforce the contract once his rights have vested, such as by manifesting assent to the promise in a manner invited or requested by the parties. Here, the tenant’s rights would have vested when he assented to have the nephew do the work.

(B) is incorrect because the fact that the tenant furnished no consideration is immaterial, because consideration was furnished by the farm owner.

(C) is incorrect because the contract does not raise any Statute of Frauds issues; hence, it is enforceable even though it was oral.

95
Q

An entrepreneur who owned a small bed-andbreakfast
sold the establishment to a retired
couple for $500,000. The couple paid $200,000
in cash and agreed to pay the balance to the
entrepreneur in equal monthly installments over
a 10-year period. A few years later, when his son
started law school, the entrepreneur sent a letter
to the couple (with a copy to his son) instructing
them to send his son $500 a month from the
money they owed to him until he instructed
them to do otherwise. When the son finished law
school three years later, the entrepreneur sent
the couple another letter (again, with a copy to
his son) telling them to stop sending the $500
monthly payment to his son and resume sending
the full monthly installments to him. Two years
after that, the entrepreneur died, leaving his
entire estate (including the balance due on the
note from the couple) to his new bride. The
entrepreneur’s son sues the couple, claiming he
is entitled to receive $500 per month from the
time he finished law school until all sums due to
the entrepreneur had been paid.
Who will prevail?

(C) The couple, because the entrepreneur had
the right to stop making payments to his
son at any time.
(D) The couple, because the son was only a
gratuitous assignee and had no protected
rights against the couple.

A

(C) The couple will prevail, because the son was a gratuitous assignee of the contract between the
entrepreneur and the couple, and the entrepreneur had expressly reserved the right to stop the
payments at any time, which he did when his son finished law school and he told the couple to
resume making the full payments to him.

(D) is wrong because the son had rights under the contract that he could enforce against the couple
until the entrepreneur revoked the assignment.

96
Q

A grandfather told his granddaughter that she
could have his house because he was moving
to a retirement home, and entered into a valid
contract to convey it to her. He promised her
that he would have another wing added to the
house in the back before turning it over to
her, and entered into a written contract with a
builder to construct the addition for his granddaughter.
Before the grandfather had entered
into the contract with the builder, the granddaughter
had paid $5,000 for a 60-day option to
purchase another house because she was not sure
she would like the addition. However, when her
grandfather showed her the plans for his house
prepared by the builder, she liked it very much
and decided to let her option to purchase the
other house lapse. Shortly thereafter, the local
zoning authority increased the minimum lot
line setbacks, making it impracticable to put the
addition on the back of the house. The builder
offered to put an addition above the existing
floor rather than in the back, and the grandfather
agreed. After the granddaughter’s option
had lapsed, she discovered that the addition
was now going up rather than in the back. She
angrily demanded that the builder either build
the addition according to the original specifications
that she approved or pay her damages. The
builder refused and the granddaughter filed suit.
Who is more likely to prevail?

(C) The builder, because he may raise all defenses that he had against the grandfather against the granddaughter.
(D) The builder, because the granddaughter
is merely an incidental beneficiary of the contract between the grandfather and the builder and, as such, has no power to enforce the contract against the builder.

A

(C) The builder will prevail because he may raise all defenses that he had against the grandfather
against the granddaughter.

The granddaughter is an intended third-party beneficiary of the contract between the grandfather and the builder.

Generally, a third-party beneficiary has rights
under the contract as soon as she does something to vest her rights (manifests assent to the
promise, brings suit to enforce the promise, or materially changes position by justifiably relying on
the promise).

Here, the granddaughter materially changed her position by justifiably allowing her option on the other house to lapse.

Generally, once the third-party beneficiary’s rights have vested, the original contracting parties may not modify the contract without the assent of the third-party beneficiary. However, the third-party beneficiary is subject to any defenses that the promisor could have used against the original promisee, and here the builder could have used the defense of impracticability against the promisee. Therefore, he could use that defense against the granddaughter to avoid having to pay damages for not building the house as he originally agreed.

(D) is incorrect because the granddaughter is not an incidental beneficiary; rather, she is an intended beneficiary because she was specifically mentioned in the contract as the recipient of the house.

97
Q

A retailer and a wholesaler entered into an
agreement under which the wholesaler would
supply the retailer with goods and the retailer
would make payment to one of the wholesaler’s
creditors. Before the creditor became aware of
the agreement, the retailer and wholesaler agreed
that the retailer instead would pay the wholesaler
directly. The creditor sued to enforce the original
agreement between the retailer and the wholesaler.
Which of the following statements best
describes the rights of the creditor?
(A) The creditor is an incidental beneficiary
only and has no rights under the agreement
between the retailer and the wholesaler.
(B) The creditor is an intended beneficiary
and has rights against both parties under
the agreement between the retailer and the
wholesaler.
(C) The creditor is an intended beneficiary of
the agreement but has no rights against the
retailer.
(D) The creditor’s rights as a third-party beneficiary
vested when it brought suit to enforce
the agreement between the retailer and the
wholesaler.

A

(C) The creditor is an intended beneficiary but has no rights against the retailer because of the modification.
A third-party beneficiary of a contract can enforce a contract only when its rights have vested. Vesting occurs when the third party

(i) manifests assent to the promise;
(ii) brings suit to enforce the promise; or
(iii) materially changes its position in reliance on the promise.

Prior to vesting, the original parties are free to modify or rescind the third-party beneficiary’s rights.

Here, the third-party beneficiary (the creditor) did not become aware of the original agreement until
after the wholesaler and the retailer modified the agreement. Because the creditor’s rights had not
vested, the parties were free to modify the agreement, which they did. The modification rescinded
the creditor’s rights under the contract. (A) is wrong because performance was to be given to
the creditor under the agreement; hence, the creditor is an intended beneficiary rather than an
incidental beneficiary. (B) is wrong because, as discussed above, the creditor no longer has rights
against the retailer after the modification of the agreement. Note that the creditor still may sue
the wholesaler on the original debt. (D) is wrong because, as stated above, the modification of
the agreement between the retailer and the wholesaler ended the creditor’s rights as a third-party
beneficiary. Hence, bringing suit did not vest any rights for the creditor.

98
Q

On January 1, a golf pro shop mailed an order
for $1,000 worth of golf balls to a golf supply
distributor, specifying that it would pay for the
balls within 30 days of delivery. The golf balls
were delivered on February 15. Two days later,
the distributor learned that the pro shop had been
insolvent for at least a month and immediately
made a demand on the pro shop to reclaim the
golf balls, despite the pro shop’s assurances that
it would still make the payment before March 15.
May the distributor reclaim the golf balls?
(A) No, because the latest date specified for
payment of the account has not yet passed.
(B) No, because the pro shop assured the
distributor that it will make payment before
March 15.
(C) Yes, because the pro shop was insolvent
when the golf balls were delivered.
(D) Yes, because the pro shop had an affirmative
duty to tell the distributor of its insolvency
when it placed its order.

A

(C) The distributor may reclaim the golf balls because the pro shop is insolvent.

UCC section 2-702 permits a seller to reclaim goods upon demand within 10 days after the buyer receives them if the seller discovers that the buyer received the goods on credit while insolvent.

Here, the distributor made a demand to reclaim the golf balls two days after the pro shop received them, so (C) is the correct answer.

(A) is incorrect because it does not matter whether the time for payment has not yet passed. As explained above, the demand to reclaim the goods must be made within 10 days of receipt, which in this case would be February 25. Thus, if the distributor waited until the latest date for payment, it would be too late to reclaim the golf balls.

(B) is incorrect because, when the
buyer is actually insolvent, the UCC allows the seller to do more than merely demand assurances.
(D) is an incorrect statement of law.

99
Q

A psychotherapist group hired a construction
company to construct a one-story building
for $900,000 to use as the group’s offices. The
contract provided that the group would pay
one-third of the contract price (i.e., $300,000) on
completion of the foundation, another one-third
($300,000) on completion of the roof, and
the balance ($300,000) when the rooms were
carpeted. After completing work on the foundation,
the construction company abandoned the
project. The group hired another contractor to
complete the building.
How much, if anything, can the construction
company recover?
(A) $300,000, the contract price for the work
that it completed.
(B) $300,000, offset by whatever damages the
group suffered in getting the job completed
by someone else.
(C) Nothing, because the construction company
committed a material breach of the
contract.
(D) Nothing on the contract, because this
construction contract is not divisible in
nature, but it might be entitled to restitution.

A

(D) The construction company cannot recover on the contract but it may recover restitution for the
work it did. The issue here is whether this contract is divisible.

A contract is divisible if:

(i) the performance of each party is divided into two or more parts under the contract,

(ii) the number of parts due from each party is the same, and

_(iii) the performance of each part by one party is
the agreed-upon equivalent of the corresponding part from the other party.
_

The third requirement is the problem here. There is no indication that the parts the construction company is to perform are the equivalent of the payments the group is to make. Rather, these payments appear to be unrelated to the actual work and merely represent progress payments. Thus, this contract is not divisible in nature.

Because the construction company breached the contract by ceasing performance before completion of the entire job, it is not entitled to the contract price. However, it may be entitled to restitution. Modern courts permit a breaching party to recover restitution where the party has conferred a benefit on the other that will result in unjust enrichment of the other if no compensation is paid.

Here, the construction company properly completed the foundation, and this conferred a benefit on the group. Therefore, restitution may be available to the construction company.

(A) is wrong because, as mentioned, the construction company is not entitled to
the contract price, because this is not a divisible contract. Even if it were divisible, (A) would
be wrong because the group would be able to offset its damages for the construction company’s
breach and thus would not be obliged to pay the full contract price.

(B) would be the correct answer if the contract were divisible. As explained above, this contract is not divisible merely because it calls for three payments. The payments must be the equivalent of the work performed; otherwise the payments are mere progress payments and the times set for payment are simply convenient times to pay.

(C) is wrong because although the breach was material, the construction
company could recover something in restitution (see above).

100
Q

A proud grandfather who planned to take
pictures of his grandson’s graduation purchased
a camera from a camera store. He used the
camera on several occasions over the next few
weeks without incident, but when he used it
on the day before his grandson’s graduation,
it caught fire and exploded, burning him and
destroying an expensive coat he was wearing.
Although the grandfather was in a great deal
of pain because of his injuries, he insisted on
attending his grandson’s graduation. However,
because he no longer had a workable camera, the
grandfather hired a professional photographer to
take pictures of the special day.
In a breach of warranty action, which of the
following represents the most that the grandfather
may recover?

(C) The difference between the value of the
camera accepted and its value if it had been
as warranted, medical costs for treating the
grandfather’s burns, and the cost to replace
the grandfather’s coat.
(D) The difference between the value of the
camera accepted and its value if it had been
as warranted, medical costs for treating the
grandfather’s burns, the cost to replace the
grandfather’s coat, and the cost of hiring
the professional photographer.

A

(C) When a buyer accepts goods that turn out to be defective, he may recover as damages any “loss
resulting in the normal course of events from the breach,” which includes the difference between
the value of the goods accepted and the value they would have had if they had been as warranted,
plus incidental and consequential damages.

Incidental damages resulting from the seller’s breach include expenses reasonably incurred in inspection, receipt, and transportation, care, and custody of goods rightfully rejected.

In this case, the grandfather incurred no incidental damages.

Consequential damages resulting from the seller’s breach include any loss resulting from general or
particular requirements and needs of which the seller at the time of contracting had reason to
know and which could not reasonably be prevented by cover or otherwise, and injury to person or
property proximately resulting from any breach of warranty.

Here, the grandfather is entitled to breach of warranty damages for the loss of the camera—the difference between the value of the camera accepted and its value if it had been as warranted—plus damages for injury to his person (e.g., medical costs for treating the grandfather’s burns) and property (i.e., the cost to replace his coat) because they were proximately caused from the breach of warranty.

(D) is wrong because the cost of hiring the professional photographer was not foreseeable. The seller was not told of any particular requirements and needs of the grandfather at the time of contracting nor would the seller have reason to know that the grandfather planned to use the camera to take pictures of his grandson’s graduation and would hire a professional photographer if he lost the use of the camera.

101
Q

A general contractor advertised in a trade
publication that she planned to bid on the
construction of a new building to be located in
the town square. The advertisement welcomed
bids from subcontractors to perform various
functions, including plumbing. The lowest
plumbing bid was from a local plumber who
bid $10,000. The contractor used the bid in
preparing her general bid. One hour after she
submitted her general bid, the plumber called
her and told her that he made a mistake on the
bid he submitted to her, and that he could not do
the plumbing work for less than $12,000. The
contractor acknowledged that he had done good
work for her in the past and said, “I’ll just forget
you ever made that $10,000 bid.” The contractor
then hired the second lowest bidder to do the
plumbing work for $12,000.
If the contractor sues the plumber for
damages, will she prevail?

(C) No, because a rescission has taken place.
(D) No, because the contractor and the plumber
mutually agreed to a release.

A

(C) The unilateral option contract between the plumber and the contractor to keep the plumber’s offer open was effectively rescinded by the contractor’s expressed intent to make a gift of the obligation owed her.

The typical case of rescission involves a bilateral contract where neither party has yet performed; i.e., the duties of both parties are still executory.

However, no contract to do the plumbing work has been created yet, because the contractor has not communicated her acceptance of the bid to the plumber. Despite her use of the plumber’s bid to prepare her own bid, the contractor is free to award the plumbing work to someone else if she is awarded the general contract. Hence, the contract to do the plumbing cannot be rescinded because it has not been created. Another contract is present under this fact pattern, however. *Under section 87 of the Restatement (Second) of Contracts, the plumber’s offer is binding as an option contract because the contractor reasonably relied on it to submit her bid.*

The option contract here is unilateral:
The contractor’s acceptance of the option contract by using the bid also constituted performance
of her duties under the option contract.

In a unilateral contract case, a rescission promise must be supported by either

(i) an offer of new consideration,

  • *(ii) elements of promissory estoppel
    (i. e., detrimental reliance), or**

(iii) the offeree’s manifestation of an intent to make a gift of the obligation owed her.

The first two alternatives are absent in these facts, but the gift alternative is
indicated by the contractor’s statement that she will “forget” that the plumber ever made the bid.
The contractor’s response was an effective rescission of the option contract.

(A) is an incorrect
choice even though it is a true statement. A discharge of contractual duties by means of a release
requires additional consideration or some substitute, such as a signed writing or reliance by the
offeror on the discharge. Here there is no additional consideration to support a release, as choice
(A) indicates, but the contractor will not win because a rescission has taken place. (B) is incorrect
because a large dollar amount for purposes of the Statute of Frauds is irrelevant unless there is a
contract for the sale of goods, which must be in writing if the goods are priced at $500 or more.
The agreement between the contractor and the plumber involved a contract for services, which
is not within the $500 provision of the Statute. (D) is incorrect because, as discussed above, a
8. contracts - Remedies Answers Set 2
release requires additional consideration, a signed writing, or detrimental reliance by the offeror.
Because none of these is indicated by the facts, a release has not taken place.

102
Q

An electrician submitted a bid for electrical
work in response to a newspaper ad placed by
a general contractor, who was bidding on the
renovation of an office building. The electrician’s
bid was the lowest and the general contractor
used it to form his bid submitted to the owner
of the building. The general contractor was
awarded the contract, but later decided to hire
another party, who had initially submitted a
higher bid, to perform the electrical work.
If the electrician sues for damages, will he
prevail?

(C) No, because the general contractor never
communicated an acceptance of the electrician’s
offer.
(D) No, because the general contractor’s advertisement for bids did not constitute an offer.

A

(C) The general contractor did not accept the electrician’s bid even though he used it to prepare his bid. The advertisement constituted an invitation for subcontractors to make offers. The electrician’s bid constituted an offer. The general rule is that acceptance of an offer must be communicated to the offeror, and here the general contractor did not communicate any acceptance to the electrician. Although statutes in some cases may create an exception to the general rule by making acceptance of a subcontractor bid automatic on the general contractor’s being awarded the contract, no such exception is indicated by these facts.

(D) is incorrect, although it is factually a true statement. The general contractor’s advertisement was an invitation for offers, not an offer in itself. The offer on these facts was made by the electrician, and the reason no enforceable contract exists is that the general contractor never accepted the electrician’s offer.

103
Q

A pop musician of modest fame offered to sell
his first guitar, the one he played as a teenager
before he became famous, to a devoted fan for
$2,000. The fan paid the musician $50 to keep
the offer open for 60 days. Thirty days later,
the musician suddenly died. The administrator
of his estate found a copy of the agreement and
informed the fan that the offer was revoked.
Nevertheless, after the value of the musician’s
paraphernalia skyrocketed due to his death, the
fan contacted the administrator to proceed with
the sale for $2,000 just before the 60-day period
expired. At that time the value of the guitar
was estimated to be $50,000. The administrator
refused to sell the guitar to the fan for $2,000, so
the fan sued the estate for breach of contract.
Is the court likely to rule in the fan’s favor?
(A) Yes, because he had an option contract that
survived the musician’s death.

(C) No, because the option terminated by
operation of law on the musician’s death.

A

(A) The fan should win because an option contract is not terminated by operation of law on the offeror’s death. An option is a distinct contract in which the offeree gives consideration for the promise by the offeror not to revoke an outstanding offer. While ordinarily an offer is terminated if either of the parties dies prior to acceptance, an offer that was irrevocable for a period of time because of an option contract survives the offeror’s death, and the offer may be accepted at any time during the option period.

(C) is incorrect because it states the general rule for offers, and this case involves an option contract, which is an exception to the general rule that an offeror can revoke an offer at will regardless of a promise to the contrary. Here that promise was supported by $50 in consideration and, thus, could not be revoked. (

104
Q

A unilateral contract is one in which the offer requests performance rather than a promise to perform, and under the First and Second Restatements, as well as the UCC, once performance has begun the offer cannot be
revoked as to that performer.

i.e.

A

Here, the woman began performing when she purchased the deep
freezer and began buying much larger than normal quantities of food with the intent of winning
the contest. At that point in time, the offer became irrevocable as to her.

105
Q

On January 1, a car salesman offered to sell
an antique car to a collector for $35,000 cash
on delivery. The collector paid the car salesman
$100 to hold the offer open for a period of 25
days. On January 4, the collector called the car
salesman and left a message on his answering
machine, asking him whether he would consider
lowering the price to $30,000. The car salesman
played back the message the same day but did
not reply. On January 9, the collector wrote the
car salesman a letter, telling him that he could
not pay more than $30,000 for the antique car,
and that if the car salesman would not accept
that amount, he would not go through with the
deal. The car salesman received this letter on
January 10 and again did not reply. The car
salesman never heard from the collector again.
When did the offer that the car salesman
made to the collector on January 1 terminate?
(A) On January 4, when the collector made a
counteroffer.
(B) On January 9, when the collector mailed to
the car salesman what amounted to a rejection.
(C) On January 10, when the car salesman
received from the collector what amounted
to a rejection.
(D) On January 25, when the 25-day option
expired.

A

(D) The car salesman’s offer terminated on January 25, when the 25-day option expired.

An option is a distinct contract in which the offeree gives consideration for a promise by the offeror not to revoke an outstanding offer. The collector paid the car salesman $100 to hold the offer open for a period of 25 days, and the offer could not be terminated before that time, not even by the offeree (here the collector). Nor did the offer survive the option period because the option specifically identified how long the offer would be open.

106
Q

An offer to make a requirements contract
(i.e., the buyer promises to buy from a certain seller all the goods it requires and the seller agrees
to sell that amount to the buyer) is (not) sufficiently definite because the quantity is capable of being
made certain by reference to objective, extrinsic facts (i.e., the buyer’s actual requirements).

A

is

107
Q

If offers stating precisely
the same terms cross in the mail, they do not give rise to a contract.

A

An offer cannot be accepted
if there is no knowledge of it. The letters sent by the mom and the salesman constitute crossing
offers, of which there was no acceptance

108
Q

The owner of a custom jewelry supply shop
placed an order with a manufacturer for 500
pairs of sterling silver “posts” of the type that
are used to make pierced earrings. However,
when the manufacturer started to fill the order, it
had only 450 pairs of sterling silver posts available.
The manufacturer shipped the 450 pairs of
sterling silver posts to the shop owner, plus
50 pairs of higher-priced 10-karat gold posts,
without making any adjustment in price. The
manufacturer enclosed a note with the order,
explaining to the shop owner that it was sending
the last of the sterling silver posts in stock, plus
the 50 10-karat gold posts to accommodate the
buyer.
Did the manufacturer’s shipment constitute an
acceptance of the shop owner’s offer?
(A) Yes, and the shop owner may not reject
the substituted goods because they are of
comparable or greater value.
(B) Yes, although it is also a breach of the
contract under the perfect tender doctrine.
(C) No, because it is a counteroffer that the
shop owner is free to accept or reject.
(D) No, because it is a breach of the contract
under the perfect tender doctrine.

A

(C) The shipment did not constitute an acceptance.

_The manufacturer’s shipment of both conforming and nonconforming goods along with a note explaining that the nonconforming goods were being offered as an accommodation to the buyer constitutes a counteroffer, which the shop owner is
free to accept or reject.
_

If the counteroffer is accepted, a contract is thereby created. Under UCC section 2-206(1)(b), a shipment of nonconforming goods by a seller is not an acceptance of the buyer’s offer if the seller seasonably notifies the buyer that the shipment is offered only as an accommodation to the buyer. If the seller had merely shipped the nonconforming goods without the explanatory note, the shipment would have been both an acceptance of the buyer’s offer, creating a bilateral contract, and a breach of the contract. I

n that case, (B) would have been the
correct answer. Under the perfect tender rule, if goods or their tender of delivery do not conform
to the sales contract in any respect, the buyer may reject the whole, accept the whole, or accept
any commercial unit or units and reject the rest, subject to the seller’s right to cure. (A) is incorrect
for that same reason: The perfect tender rule, and not the concept of comparable worth, applies to
contracts for the sale of goods. (D) is incorrect because there first must be an acceptance before
there is a contract to breach. The perfect tender doctrine does not relate to acceptance.

109
Q

A city council passed an ordinance providing
that residents who had their car’s exhaust
pipes replaced by modern models reducing gas
emissions would receive a partial refund of the
expense incurred. Information concerning the
refund was telecast once daily for a week over
the local television station. The next year, the
city council passed a resolution repealing its
refund offer. The city council caused this resolution
to be broadcast by the city’s sole radio
station once daily for one week. The local television
station, meanwhile, had ceased operations.
If the city’s refund offer was revocable,
revocation could be effectively accomplished
only:
(A) In the same manner as made or by a comparable
medium and frequency of publicity.
(B) In the same manner as made only.
(C) By simply passing the resolution, because
the events that transpired at the city council
meeting are considered to be of public
notice.
(D) By notice mailed to all residents of the city
and all other reasonably identifiable potential
offerees.

A

(A) Thiskind of refund offer can be revoked by comparable publicity as to the revocation by the
offeror. Here, publicity through means of the city’s radio station would be comparable to the
initial offer publicized over the television station. (B) is wrong because the facts make it impossible
to use the same manner to revoke the offer.

(C) does not go far enough in communicating the
revocation effectively to anyone who heard the offer originally.

(D) is incorrect because identifying
and contacting all “reasonably identifiable” offerees would be too onerous a task.

110
Q

A manufacturer of widgets sent an offer
to a major wholesaler, offering a standard lot
(quantity well-known in the widget trade) of
widgets for $8,000. The president of the wholesaler
personally mailed back to the manufacturer
the wholesaler’s standard printed acceptance
form. However, the president wrote in large
letters in his own hand on the form, “Our
liability on this contract is limited to $200.” Two
days later, the manufacturer’s sales manager
received the communication from the wholesaler.
A week later, the manufacturer had sent no
additional communication to the wholesaler.
What is the relationship between the parties?
(A) There is no contract between the manufacturer
and the wholesaler, because the
wholesaler made a material alteration.
(B) There is a valid, enforceable contract
between the manufacturer and the wholesaler,
but it is limited to the terms of the
manufacturer’s offer.
(C) There is a valid, enforceable contract
between the manufacturer and the wholesaler,
and it contains the additional term
because the manufacturer raised no objection.
(D) The wholesaler has sent a valid counteroffer
to the manufacturer, which the manufacturer
can accept or reject.

A

(B) An enforceable contract exists between the manufacturer and the wholesaler, but it does not
include the limitation of liability added by the wholesaler’s president.

At common law, any different or additional terms in an acceptance of an offer made the response a rejection and counteroffer.

In contracts for the sale of goods, however, UCC section 2-207 substantially alters the common law rule. The proposal of additional terms by the offeree in a definite and timely acceptance does not constitute a rejection and counteroffer. Rather, the acceptance is effective unless it is expressly made conditional on assent to the additional terms. If both parties to the sale of goods contract are merchants, the additional terms will become part of the contract unless

(i) they materially alter the contract,
(ii) the offer expressly limits acceptance to the terms of the offer, or

(iii) the offeror has already objected to the particular terms, or he objects within
a reasonable time after notice of them is received.

Here, a sale of goods contract is involved and
both parties are merchants. The wholesaler accepted the manufacturer’s offer by mailing back a standard printed acceptance form, even though the form contained an additional limitation of liability term added by the wholesaler’s president. This limitation of liability provision did not become part of the contract, however, because it would have substantially changed the allocation of economic risks and benefits and impaired an otherwise available remedy. Hence, a valid contract exists between the parties but it is limited to the terms of the manufacturer’s offer.

(A)
is incorrect because the material alteration by the wholesaler does not prevent formation of the
contract under section 2-207, but the term that produces the material alteration will not be a part
of the contract. (C) is incorrect. The additional term does not become part of the contract, even
though the manufacturer raised no objection to it, because it materially altered the contract. An
additional term will only become part of the contract if it is not a material alteration and if the
offeror does not object to it within a reasonable time. Hence, the manufacturer’s failure to object
did not cause the term to be included in the contract. (D) is incorrect because under section 2-207,
the wholesaler’s response to the manufacturer was an acceptance rather than a counteroffer. For
the response to have been an offer rather than an acceptance, it would have had to expressly
condition acceptance on the manufacturer’s assent to the additional term In that case, it would
have been a rejection and new offer.

111
Q

An accord is an agreement
in which one party to an existing contract agrees to accept, in lieu of the performance that
he is supposed to receive from the other party, some other, different performance. Generally, an
accord must be supported by consideration, but the consideration may be of a lesser value than the
originally bargained-for consideration in the prior contract, as long as …

A

it is of a different type or the claim is to be paid to a third party.

Here, the homeowner’s obligation to provide the contractor’s mother with a new washer and dryer was of a lesser value to the homeowner than the originally bargained-for consideration in the prior contract,

but it was of a different type and the claim
was to be paid to a third party (the contractor’s mother). Thus, it was sufficient new consideration
to form a valid accord.

112
Q

On April 4, a homeowner hired an electrician
to evaluate his home’s wiring. The electrician
found the wiring to be faulty, called it a
fire hazard, and urged the homeowner to fix it
soon. That day, the homeowner entered into a
written contract with the electrician, whereby
the electrician agreed to repair the wiring for
$1,600. The contract provided that the repair
would take place prior to April 30, on a day
when the homeowner would be home. On
April 26, the work had yet to be done so the
homeowner called the electrician to pin down
a date. The electrician suggested April 29, and
the homeowner agreed. On the morning of April
29, the electrician arrived ready to do the repair
work. However, he discovered that no one was
home, and he could not get inside. Earlier that
morning, the homeowner had been rushed to
the hospital for an emergency appendectomy,
and the homeowner did not return home from
the hospital until May 1. On that day, the faulty
wiring caused a fire that severely damaged the
house.
Will the electrician succeed in a breach of
contract action?
(A) Yes, because the homeowner made and
breached an implied-in-fact promise to
make the house available to the electrician
before April 30.
(B) Yes, because any recovery by the electrician
will be subject to a setoff by the
homeowner on account of his damage from
the fire.
(C) No, because the electrician did nothing
about the agreement between April 4 and
April 26.
(D) No, because the homeowner’s obligations
under the contract were subject to
an express condition precedent that failed
because of supervening impossibility.

A

(D) The electrician will not succeed because there was an express condition precedent that the repairs had to be performed on a day when the homeowner would be home, and the homeowner was unable to honor that commitment due to an unforeseeable supervening event.

(C) is not the best answer. Although the electrician did nothing between April 4 and April 26, he did confirm the repair date of April 29 with the homeowner. The electrician would have honored the contract but for the supervening impossibility of the homeowner being unable to stay home.

(A) is a false statement.
The promise to make the house available was an express condition, not an implied-in-fact
contract.

(B) is wrong because impossibility discharges duties on both sides.

113
Q

A standard commercial unit for widgets is one
gross, i.e., 144 widgets. A buyer ordered from a
seller 50 gross of widgets at $100 per gross, with
the widgets to be delivered on or before October
3. On September 15, 50 boxes arrived from the
seller. Each box was labeled, “Widgets—one
gross.”
On finding that one of the boxes contains 143
widgets, what can the buyer do?
(A) Seek cover.
(B) Immediately sue for breach.
(C) Reject the entire shipment of widgets.
(D) Do nothing because one less widget is not
material.

A

(C) Under the “perfect tender rule,” the buyer may reject the entire shipment. This power to reject
is tempered by the UCC ’s provisions allowing a seller to “cure” defects. Although “cure” is not
mentioned as an option, (C) remains a better answer than the others because (A) and (B) are not permissible options for the buyer at this time.

*With regard to (A), the contract’s delivery date
has not yet arrived and the seller must be given an opportunity to “cure” before the buyer seeks
“cover.”
*

_(B) is not an option because the delivery date has not yet arrived and there is no reason to
anticipate that the seller will not make good before the date in question. There is no breach until delivery date, and an anticipatory breach situation does not exist.
_
Hence, (A) and (B) are incorrect.
(D) is incorrect because 143 widgets is not a gross and thus is a breach of the contract.

114
Q

A builder of racing cars entered into a contract
with a buyer to sell him a hand-built car for
$25,000. The price was to be paid and the car
was to be delivered one week later. Unbeknownst
to the buyer, the builder’s wife had a one-half
interest in the car. The day after the contract was
signed, the builder called the buyer and told him
about his wife’s half interest in the car, and that
she would not go along with the sale at $25,000
but would agree to a sale for $40,000.
If the buyer sues the builder for breach of
contract, will the court find that there is an
enforceable contract for $25,000?
(A) No, because the car cannot be sold unless
both owners convey title.
(B) Yes, because the buyer was unaware of the
interest of the builder’s wife when he signed
with the builder.
(C) Yes, regardless of the fact that the buyer
was unaware of the interest of the builder’s
wife at the time he signed.
(D) Yes, but the contract is discharged by
prospective inability of performance.

A

(C) The builder’s unconditional promise to sell created a contract even if the buyer knew of the
builder’s wife’s interest.

*When a promise is unconditional, the failure to perform according to its terms is a breach of contract.* By not making his promise conditional on his wife’s consent to convey her interest, the builder impliedly undertook to obtain her consent. Therefore, the contract is enforceable. (Note that this does not necessarily mean that the buyer will be able to get the car; he may have to settle for damages because of the builder’s wife’s interest.)

(A)
is incorrect although partially true. It is true that the builder cannot sell his wife’s half of the
car without her consent; however, that does not make the contract here unenforceable. As
stated above, by making his promise unconditional, the builder undertook a duty to obtain his
wife’s consent to sell the car. His failure to do so is a breach of contract, but a breach does not
negate a contract; it merely gives the nonbreaching party a right to certain remedies. Therefore,
the contract is enforceable, even if the builder’s wife refuses to sell her interest. (B) is incorrect
because the buyer’s knowledge of the builder’s wife’s interest is irrelevant to the issue of
the contract’s enforceability. As discussed above, the builder’s unconditional promise implied
that the builder would obtain his wife’s consent to convey her interest in the car. Therefore,
the contract is enforceable regardless of whether the buyer was aware of the builder’s wife’s
interest at the time he signed. (D) is incorrect because “prospective inability” is not a ground
for discharge. Prospective failure of consideration occurs when a party has reasonable grounds
to believe that the other party will be unable or unwilling to perform when due. The prospective
inability of performance does not discharge the contract; rather, it allows the innocent party
to suspend further performance until he receives adequate assurances that performance will be
forthcoming. Therefore, the contract between the builder and the buyer is not discharged because
of prospective inability of performance.

115
Q

An owner of land who was also a home
contractor agreed in writing with a buyer to
build a house to the buyer’s specifications on his
(the contractor’s) land and then sell the house
and lot to the buyer. The contract provided that
the house was to be completed by March 1, with
full payment due at that time. On March 1, the
house was nearly complete, but due to delays in
the delivery of materials, the contractor would
need 20 more days to finish construction. On
March 5, after discovering that the house had
not yet been completed, the buyer notified the
contractor in writing of her election to cancel
the contract because of the contractor’s failure
to deliver the house by March 1. The contractor
responded that, due to an unanticipated strike at
his supplier’s company, performance had been
unforeseeably delayed and that the house would
be ready by March 20. The buyer responded that
she would no longer accept delivery of the house
and land. The contractor then brought an action
to recover damages for breach of contract.
Who will prevail?
(A) The buyer, because the express date listed
in the contract indicates that time was of
the essence.
(B) The buyer, because delivery by March 1
was a condition precedent to the buyer’s
performance.
(C) The contractor, because the strike was truly
an unforeseeable intervening event.
(D) The contractor, because the buyer received
the substantial benefit of the bargain.

A

(D) The contractor will prevail because the buyer received the substantial benefit of her bargain. The failure to perform on time is a breach of contract, but in this case, it was a minor breach. Unless the nature of the contract is such as to make performance on the exact day agreed upon of vital importance, or the contract provides that time is of the essence, failure to perform at the stated time is not a material breach.

Here, the home was nearly complete, and the delay was relatively short. The contract did not specify that time was of the essence; thus, the breach was minor. The remedy for a minor breach is damages; the aggrieved party is not relieved of her duty to perform.

(A) is incorrect because merely stating a date for performance does not indicate that time is of
the essence. There must be some explicit statement indicating that time is of the essence.

(B) is incorrect. Although the delivery on March 1 is a condition precedent to the buyer’s duty to pay, the condition is excused by substantial performance. The test for whether a party has substantially performed is the same as the one for assessing whether a breach is minor or material. Here the breach is minor, the contractor substantially performed, and the condition is excused.

(C) is incorrect because an unforeseeable event does not discharge a party’s duty to perform. A strike at the contractor’s supplier does not rise to the level of impossibility or impracticability, which would discharge his duty to perform. The contractor could have procured the supplies elsewhere.

116
Q

A builder contracted to build a house for a
newly married couple. Terms of the contract
provided that the builder would receive the
contract price when the building was fully
completed. Just when the builder had completed
one-half of the structure, a tornado struck the
area and demolished the building.
What is the builder entitled to recover from
the couple under the contract?
(A) Nothing.
(B) One-half of the contract price.
(C) One-half of the fair market value of what
remains of the house.
(D) Cost of materials and reasonable labor
costs.

A

(A) The builder will not be able to recover anything from the couple under the contract because he has not performed his duty. Under the parties’ contract, the builder’s completion of the house was a condition precedent to the couple’s duty to pay. The condition precedent was not discharged by the destruction of the work in progress because construction has not been made impossible, but rather merely more costly—the builder can rebuild. Thus, he is not entitled to any recovery.
Note, however, that a number of courts will excuse timely performance because the destruction was not the builder’s fault.

(B) is incorrect because the contract is not divisible (i.e., it is not divided into an equal number of parts for each side, each part being the quid pro quo of the other);
thus, completion of one-half of the house did not entitle the builder to one-half of the price.

(C) is incorrect because it is not a correct measure of recovery. As stated above, the builder cannot recover under the contract. However, he could recover restitution if he determined that he could not perform under the contract by rebuilding. Restitution is a remedy that prevents unjust enrichment by imposing on a recipient of requested goods or services a duty to pay for the benefit received when there is a failed contract or no contractual relationship between the parties. The measure of recovery here would be the fair market value of what remains of the house because that is the benefit conferred—it would not be cut in half merely because the house was only half completed.

(D) is an incorrect contract recovery because the builder has not fulfilled the condition
precedent to the couple’s duty to pay. The only way the builder could recover anything would
be in an action for restitution. Sometimes, in cases where there is little or no benefit to the other party, the measure of restitutionary recovery is the detriment suffered by the plaintiff. However, when the plaintiff is in breach, the courts that permit recovery limit it to the contract price less damages caused by the breach. Because the builder only did half of the work, he is in breach by not rebuilding the house. Hence, the couple’s damages, which involve building an entirely new house, outweigh any possible restitution to the builder.

117
Q

A consortium of actors decided to open a
new theater and almost immediately raised
$130,000 of the $250,000 needed to build it.
The contractor whom the consortium hired to
build the theater agrees to be paid $50,000 in
cash on commencement of construction, another
$80,000 when construction was completed, and
the remaining $120,000 in monthly payments
of $1,000 principal plus 12% annual interest on
the outstanding balance, once the theater started
earning a profit. Their agreement was reduced to
a writing that the parties signed.
The first installment was paid, construction
was completed on time, and the second installment was paid. The theater opened, but business was not good, and the theater did not make any profit. The consortium found a buyer who paid them $260,000 for the theater. The contractor sues the consortium for the remaining $120,000 owing on the contract.
Will the contractor recover?
(A) No, because the consortium never earned a
profit from its operation of the theater.
(B) No, because the failure to earn profits from
the operation of the theater was an unforeseeable
intervening event.
(C) Yes, because the provision governing
payment of the outstanding balance of the
construction cost merely established the
time frame in which payment was to be
made.
(D) Yes, because all of the conditions precedent
to the consortium’s duty to pay had
occurred.

A

(C) The contractor will recover the balance owing on the contract because he did not intend to assume the risk of the theater not making a profit. Once again, distinguishing between a promise and a condition is critical. If the provision regarding payment once the theater became profitable is a condition, its failure to occur will extinguish the consortium’s duty to pay the contractor what it promised under the contract. The basic test of whether a contractual provision is a promise or a condition is the intent of the parties. Courts will examine the words of the agreement, prior practices of the parties, custom, and whether fulfillment of the provision is to be rendered by the promisee (in which case it is more likely to be a condition) or by the promisor. Under a provision that a duty is to be performed “once” an event occurs, if the event is not within the control of the
promisee, it is less likely that he will have assumed the risk of its nonoccurrence and therefore less likely to be a condition of the promisor’s duty to perform.
In doubtful situations, courts will more likely hold that the provision is a promise rather than a condition because it supports the contract and preserves the reasonable expectations of the parties.

Here, there is no evidence that the
contractor was agreeing to a joint venture with the consortium whereby he would not get paid if
the consortium did not do well but would not receive any extra if it was very successful. Whether the theater made a profit was more within the control of the consortium than of the contractor; he was assuming nothing more than a risk of delay in receiving payment to allow the consortium a chance to begin making a profit. His reasonable expectation was that ultimately he would be paid the balance owing on the contract, and the sale of the theater by the consortium gives it the ability to pay him.

(A) is incorrect because the provision governing payment is not a condition
excusing the absolute duty of the consortium to perform under the contract. The court will imply
a promise to pay the balance after a reasonable time if the timing provision in the contract is not
triggered. (B) incorrectly suggests that the contractual duty of the consortium was discharged by
frustration of the contract’s purpose. Frustration exists if: (i) some supervening act or event occurs
that was not reasonably foreseeable at the time of entering into the contract; (ii) the act or event
has completely or almost completely destroyed the purpose of the contract; and (iii) both parties
realized the purpose of the contract at the time of making it. Here, the failure to earn a profit
from a new theater was certainly not unforeseeable; in fact, the timing provision indicates that
the parties realized at least some uncertainty as to the theater’s profitability. (D) is not as good
a choice as (C) even though it may be a true statement. While the contractor’s completion of the
theater is not only a promise by him but also a condition precedent to the consortium’s duty to
pay (and this condition has occurred), the issue in this question is whether the payment provision
is a condition or merely a promise as to the timing of the monthly payments, and (C) specifically
addresses that issue.

118
Q

A proud grandfather who planned to take pictures of his grandson’s graduation purchased a camera from a camera store. He used the camera on several occasions over the next few weeks without incident, but when he used it on the day before his grandson’s graduation, it caught fire and exploded, burning him and destroying an expensive coat he was wearing. Although the grandfather was in a great deal of pain because of his injuries, he insisted on attending his grandson’s graduation. However, because he no longer had a workable camera, the grandfather hired a professional photographer to take pictures of the special day.

In a breach of warranty action, which of the following represents the most that the grandfather may recover?

A The difference between the value of the camera accepted and its value if it had been as warranted.

B The difference between the value of the camera accepted and its value if it had been as warranted, plus medical costs for treating the grandfather’s burns.

C The difference between the value of the camera accepted and its value if it had been as warranted, medical costs for treating the grandfather’s burns, and the cost to replace the grandfather’s coat.

D The difference between the value of the camera accepted and its value if it had been as warranted, medical costs for treating the grandfather’s burns, the cost to replace the grandfather’s coat, and the cost of hiring the professional photographer.

A

When a buyer accepts goods that turn out to be defective, he may recover as damages any “loss resulting in the normal course of events from the breach,” which includes the difference between the value of the goods accepted and the value they would have had if they had been as warranted, plus incidental and consequential damages.

Incidental damages resulting from the seller’s breach include expenses reasonably incurred in inspection, receipt, and transportation, care, and custody of goods rightfully rejected. In this case, the grandfather incurred no incidental damages.

Consequential damages resulting from the seller’s breach include any loss resulting from general or particular requirements and needs of which the seller at the time of contracting had reason to know and which could not reasonably be prevented by cover or otherwise, and injury to person or property proximately resulting from any breach of warranty.

Here, the grandfather is entitled to breach of warranty damages for the loss of the camera—the difference between the value of the camera accepted and its value if it had been as warranted—plus damages for injury to his person (e.g., medical costs for treating the grandfather’s burns) and property (i.e., the cost to replace his coat) because they were proximately caused from the breach of warranty. Thus, (C) is correct, and (A) and (B) are wrong. (D) is wrong because the cost of hiring the professional photographer was not foreseeable. The seller was not told of any particular requirements and needs of the grandfather at the time of contracting nor would the seller have reason to know that the grandfather planned to use the camera to take pictures of his grandson’s graduation and would hire a professional photographer if he lost the use of the camera.

119
Q

A manufacturer of down coats and jackets entered into a written agreement with a distributor, whereby the distributor agreed to distribute the manufacturer’s products statewide for a one-year period to begin on June 1. Before the manufacturer signed the distribution contract with the distributor, the distributor told the manufacturer that their deal was exclusive, but nothing to that effect was in the written agreement. However, in the outerwear industry it has been a custom for many years for distributors to distribute only one brand of outerwear.

On September 1, the distributor began distributing coats and jackets manufactured by one of the manufacturer’s chief competitors. These coats and jackets were sewn with man-made fabrics, were as warm as the manufacturer’s jackets, and were less bulky. The competitor’s advertising campaign throughout the state emphasizes that “you don’t have to look fat to stay warm.” Seasonally adjusted sales figures showed that the manufacturer’s sales in the state dropped 6% after its competitor’s products were introduced.

The manufacturer of the down coats and jackets complained to the distributor, demanding that it stop distributing the man-made coats and jackets made by the manufacturer’s competitor. The distributor refused, and the manufacturer of the down coats and jackets brought suit against the distributor.

Which of the following facts would provide a basis for the manufacturer’s best case against the distributor?

A The competitor’s advertising campaign throughout the state alluding to the unattractive bulkiness of the manufacturer’s coats and jackets.

B The 6% drop in seasonally adjusted sales figures in the state after the competitor’s products were introduced.

C The distributor’s oral statement to the manufacturer about their deal being exclusive.

D The long-standing custom in the outerwear industry for distributors to distribute only one brand of outerwear.

A

Of all the alternatives listed, (D) is the only one that presents any real basis for supporting the manufacturer’s case.

One of the general rules of contract construction, including contracts for goods under the UCC, is that courts will look to see what custom and usage are in the particular business and in the particular locale where the contract is either made or to be performed. The manufacturer could claim that when he and the distributor entered into the distribution contract, both parties implicitly understood that the custom of distributing only one brand of outerwear would be followed in their transaction. Under such circumstances, the manufacturer may be able to successfully assert that the distributor’s distribution of the competitor’s outerwear constitutes a breach of contract. The fact that the competitor’s advertising campaign at least impliedly denigrates the appearance of the manufacturer’s outerwear (as in (A)), or that the manufacturer’s sales have dropped since the introduction of the competitor’s products (as in (B)), establishes no cause of action against the distributor. Absent some provision in the contract, or some reference to custom and usage as mentioned in (D), there is no basis for holding that the distributor was prohibited from distributing other companies’ products, or that the distributor can be held liable for a decline in the manufacturer’s sales figures due to sales or advertisements made by a company whose products are being distributed by the distributor. (C) is incorrect because the written agreement between the distributor and the manufacturer would probably be deemed to be a final expression of the bargain, so that evidence of the distributor’s prior expression would be inadmissible to vary or supplement the writing under the parol evidence rule. Under UCC section 2-202, a party cannot offer consistent additional terms if the writing was intended as a complete statement of the terms of the agreement. (In contrast, evidence of custom in the trade can be offered regardless of the completeness of the written agreement.) Also, the distributor’s statement about their deal being exclusive is not specific enough on its face to establish whether he meant that the distributor would distribute only the manufacturer’s products, or that the manufacturer would have its products distributed in the state only by the distributor, or perhaps some other meaning. The statement, even if admissible, is not definite enough to form a basis for a cause of action against the distributor.

120
Q

A grandfather told his granddaughter that she could have his house because he was moving to a retirement home, and entered into a valid contract to convey it to her. He promised her that he would have another wing added to the house in the back before turning it over to her, and entered into a written contract with a builder to construct the addition for his granddaughter. Before the grandfather had entered into the contract with the builder, the granddaughter had paid $5,000 for a 60-day option to purchase another house because she was not sure she would like the addition. However, when her grandfather showed her the plans for his house prepared by the builder, she liked it very much and decided to let her option to purchase the other house lapse. Shortly thereafter, the local zoning authority increased the minimum lot line setbacks, making it impracticable to put the addition on the back of the house. The builder offered to put an addition above the existing floor rather than in the back, and the grandfather agreed. After the granddaughter’s option had lapsed, she discovered that the addition was now going up rather than in the back. She angrily demanded that the builder either build the addition according to the original specifications that she approved or pay her damages. The builder refused and the granddaughter filed suit.

Who is more likely to prevail?

A The granddaughter, because she was an intended beneficiary of the contract whose rights had vested.

B The granddaughter, because the subsequent agreement between her grandfather and the builder to modify the construction was unsupported by consideration.

C The builder, because he may raise all defenses that he had against the grandfather against the granddaughter.

D The builder, because the granddaughter is merely an incidental beneficiary of the contract between the grandfather and the builder and, as such, has no power to enforce the contract against the builder.

A

The builder will prevail because he may raise all defenses that he had against the grandfather against the granddaughter. The granddaughter is an intended third-party beneficiary of the contract between the grandfather and the builder. Generally, a third-party beneficiary has rights under the contract as soon as she does something to vest her rights (manifests assent to the promise, brings suit to enforce the promise, or materially changes position by justifiably relying on the promise). Here, the granddaughter materially changed her position by justifiably allowing her option on the other house to lapse. Generally, once the third-party beneficiary’s rights have vested, the original contracting parties may not modify the contract without the assent of the third-party beneficiary. However, the third-party beneficiary is subject to any defenses that the promisor could have used against the original promisee, and here the builder could have used the defense of impracticability against the promisee. Therefore, he could use that defense against the granddaughter to avoid having to pay damages for not building the house as he originally agreed. (A) is incorrect because although it is true that the granddaughter’s rights had vested, the answer fails to take into account the defenses available to the builder. (B) is incorrect because there was consideration for the modification. Because the original contract was impracticable to perform, the builder would have been discharged. By agreeing to build the addition above, he undertook something that he was not otherwise bound to do. Likewise, because of the impracticability, the grandfather would have been discharged from his original contract to pay. (D) is incorrect because the granddaughter is not an incidental beneficiary; rather, she is an intended beneficiary because she was specifically mentioned in the contract as the recipient of the house.

121
Q

Pursuant to a contract, a landscaper performed $30,000 of landscape work for a homeowner. By coincidence, the homeowner and the landscaper were involved in an automobile accident that was unrelated to the landscape work. The homeowner was injured in the accident and sued the landscaper in federal district court for negligence, seeking $100,000 in damages. The homeowner and the landscaper are citizens of different states.

May the landscaper assert and maintain a counterclaim against the homeowner for breach of contract, seeking the $30,000 due under the landscape contract?

A No, because the Federal Rules of Civil Procedure permit a counterclaim only if the counterclaim arises from the same transaction or occurrence as the plaintiff’s initial claim.

B No, because, while the court has diversity of citizenship jurisdiction over the homeowner’s negligence claim and the Federal Rules of Civil Procedure permit the counterclaim, the federal court does not have subject matter jurisdiction over the landscaper’s contract claim.

C Yes, because the Federal Rules of Civil Procedure permit the counterclaim, and the court has diversity of citizenship jurisdiction over both claims.

D Yes, because the court has diversity of citizenship jurisdiction over the homeowner’s negligence claim, and it has supplemental jurisdiction over the landscaper’s contract claim because it is a compulsory counterclaim.

A

The landscaper cannot assert and maintain a counterclaim against the homeowner for breach of contract because the federal court does not have subject matter jurisdiction over the contract claim.

The court has diversity of citizenship jurisdiction over the negligence claim because there is complete diversity and the amount in controversy exceeds $75,000. A compulsory counterclaim (a claim that arises out of the same transaction or occurrence as the plaintiff’s claim) does not need to meet the jurisdictional amount requirement for diversity jurisdiction. However, a permissive counterclaim (a claim that arises out of an unrelated transaction) must meet the jurisdictional amount requirement. Here, the landscaper’s contract claim would be a permissive counterclaim because it is unrelated to the negligence claim arising from the accident. Therefore, the claim’s amount in controversy would have to exceed $75,000. The claim is only for $30,000; therefore, the court does not have subject matter jurisdiction over the contract claim. (A) is incorrect because a defendant may assert claims against a plaintiff that are unrelated to the plaintiff’s claims, provided there is subject matter jurisdiction. (C) is incorrect because the court does not have diversity jurisdiction over the contract claim. (D) is incorrect because the contract claim is not a compulsory counterclaim.

122
Q

The owner of an apartment building contracted with a painter to paint the porches of the apartments for $5,000. The contract was specifically made subject to the owner’s good faith approval of the work. The painter finished painting the porches. The owner inspected the porches and believed in good faith that the painter had done a bad job. The painter demanded payment, but the owner told him that the paint job was poor and refused to pay. The painter pleaded that he was desperately in need of money. The owner told the painter that she would pay him $4,500, provided he repainted the porches. The painter reluctantly agreed, and the owner gave the painter a check in the amount of $4,500. The painter went to his bank, indorsed the check “under protest” and signed his name, then deposited the check in his account. He never returned to repaint the porches.

The painter sues the owner for $500, which he believes is still owed to him on his contract to paint the porches. Will he prevail?

A Yes, because he indorsed the check “under protest.”

B Yes, but only if he repaints the porches.

C Yes, because he performed the contract by painting the porches the first time.

D No, even if he repaints the porches.

A

The painter will be unable to recover the $500 because he did not satisfy the condition precedent to payment under the contract. A party does not have a duty to perform if a condition precedent to that performance has not been met.

Here, the parties made the owner’s satisfaction with the painter’s paint job a condition precedent to the owner’s duty to pay the $5,000. Because the owner was not satisfied with the paint job, her duty to pay the painter never arose. The fact that the owner offered to give the painter $4,500 if he repainted the porches has no effect on this analysis, because the offer constituted a new contract, the owner having been excused from the old one.

(A) is wrong because it does not matter whether the painter indorsed under protest. The indorsement will not change the result here because the new contract did not seek to discharge any contractual duty—the owner was already excused from her duties because the condition precedent was never met.

(B) is wrong because the old contract, which provided for payment of $5,000, is considered to be at an end. Under the terms of the new contract, the painter is entitled to only $4,500, provided he repaints the porches.

(C) is wrong because the condition precedent to the payment of $5,000, the owner’s satisfaction, was not met. The courts have held such conditions to be valid—not illusory promises—because of the promisor’s duty to exercise good faith in assessing satisfaction. Here, the facts state that the owner believed in good faith that the painter had done a bad job; thus, the painter is not entitled to payment under the original contract. Note that since he has not performed under the new contract, he is in breach and not entitled to the $4,500 already paid

123
Q

On April 10, the owner of a small farm mailed a letter to a new resident of the area who had expressed an interest in buying the farm. In this letter, the farm owner offered to sell the farm to the resident for $100,000. The offer expressly stated that the offer expires on June 1, “if acceptance by the offeree has not been received by the offeror on or before that date.”

On the morning of June 1, the resident sent a written acceptance to the farm owner by messenger. However, through negligence of the messenger company, the acceptance was not delivered to the farm owner until June 2. On June 4, the farm owner entered into a contract to sell the farm to another buyer for more money but did not inform the resident of the transaction. When the resident followed up by phone on June 10, the farm owner told him that he had sold the farm to another buyer.

Which of the following is the most correct statement?

A No contract between the farm owner and the resident arose on June 2.

B An enforceable contract arose on June 1.

C The farm owner’s silence constituted an acceptance of the resident’s message on June 2.

D A voidable contract arose on June 1.

A

No contract arose on June 2 because the farm owner’s offer expired on June 1, when the farm owner did not receive the resident’s acceptance. If a period of acceptance is stated in an offer, the offeree must accept within that period to create a contract. Failure to timely accept terminates the power of acceptance in the offeree (i.e., a late acceptance will not be effective and will not create a contract). Under the mailbox rule, an acceptance generally is effective upon dispatch (i.e., the acceptance creates a contract at the moment it is mailed or given to the delivery company). However, the mailbox rule does not apply where the offer states that acceptance will not be effective until received. In the latter case, acceptance is effective only upon receipt. Here, the farm owner’s offer specifically stated that the acceptance must be received by June 1 to be effective. Thus, the farm owner opted out of the mailbox rule, and no contract was created by delivery of the acceptance on June 2. Note that the resident will not be able to successfully argue that the acceptance was valid because the late delivery was the messenger company’s fault. This would be a valid argument if the mailbox rule applied here, because the acceptance would have been effective on June 1, when the message was given to the messenger company. However, by opting out of the mailbox rule, the farm owner put the burden of any negligence in delivery on the resident. Thus, there was no valid acceptance.

124
Q

A smoothie retailer and a fruit processor entered into an oral agreement that provided that the processor would deliver to the retailer 100 barrels of fruit each month at a price of $10 per barrel, with delivery on the first of the month and payment of the $1,000 to a creditor of the fruit processor on the 15th of each month. However, when the agreement was reduced to a writing, the fruit processor’s manager inadvertently wrote $20 per barrel, and neither party noticed before signing. The creditor first learned of the agreement between the parties when he received a copy of it the day after it was signed, showing that he was to receive $2,000 per month. One day later, the retailer discovered the mistake and alerted the food processor. The parties prepared a revised writing reflecting the correct contract price of $10 per barrel, and also agreed in writing that the retailer would receive a $2 per barrel discount the first month because it discovered the mistake by the fruit processor. The first delivery under the contract was made two days late, on the third of the month. On the 15th of the month, the creditor demanded payment of $2,000 from the retailer according to the terms of the original writing.

If the retailer contends that it is not liable to pay the full $2,000, which of the following would NOT be relevant to its defense?

A The parties had modified the contract to provide for a $2 per barrel discount the first month.

B The parties had originally agreed that the price per barrel would be $10, and neither party noticed before signing that the manager of the fruit processor had inadvertently written $20 in the contract.

C The fruit processor owed its creditor only $1,600.

D The fruit processor was late with its first delivery.

A

Any defense that the fruit processor might have with respect to the money it owed to the creditor would not provide the retailer with a defense.

If the promisor has made an absolute promise to pay the third-party beneficiary (and not simply a promise to pay whatever the promisee owed him), the promisor cannot assert the promisee’s defenses.

Hence, the fact that the processor owed only $1,600, even if it could be asserted as a defense by the processor against the creditor, cannot be asserted as a defense by the retailer.

(A) is incorrect because the promisee and promisor in a third-party beneficiary contract are free to modify their contract until the third party’s rights have vested. While the creditor learned of the agreement before the processor and the retailer modified it, he did not (i) manifest assent, (ii) bring suit, or (iii) materially rely on the agreement before it was modified; thus, his rights did not vest. Assuming the processor has performed and the retailer’s duty to perform is now absolute, the retailer would be liable to the creditor for only $800 under the contract as modified. Therefore, (A) is a partial defense. (B) is also a partial defense. When a third-party beneficiary sues the promisor on the contract, the promisor may raise any defense he would have had against the promisee. Under the doctrine of reformation, either of the parties to the contract may ask a court in equity to modify the terms of the contract where the writing, through mistake or misrepresentation, does not incorporate the terms orally agreed upon. Here, the parties’ mistake in memorializing the contract permits the retailer to have the contract reformed to show the parties’ original agreement. This provides a partial defense that the retailer can use against the creditor to show that it is not liable for the full $2,000. (D) is incorrect for a similar reason. The processor’s failure to perform according to the terms of the contract may be asserted as a defense by the retailer. The retailer’s liability to the creditor would be offset by whatever lost sales the retailer incurred as a result of the fruit processor’s late delivery.

125
Q

A homeowner contracted with a local heating company to install two baseboard heaters in an addition to his home for a total cost of $3,500. This figure included the heaters and labor costs for installation. Upon completion of the installation, the heating company sent an invoice to the homeowner for the $3,500. The homeowner did not immediately pay the bill because the heaters were too noisy. The heating company sent a repair worker to the home to service the heaters, but after several attempts to fix the problem, the heaters were still too loud.

The homeowner contacted a qualified repairman to find out how to fix the problem and was told it would cost an additional $300 for new blowers and $150 in labor costs to replace the faulty blowers. The homeowner mailed the heating company a copy of the repair estimate and a check for $3,050—the contract price less the cost of new blowers and labor to install them—and wrote prominently on the check “Payment in full for installation of two baseboard heaters.” The heating company cashed the check upon receipt. The heating company then sued the homeowner for $450, the difference between the agreed contract price and the amount paid.

Is the heating company likely to prevail in its suit seeking the $450 from the homeowner?

A No, because the heating company failed to fix the problem after several attempts.

B No, because the heating company cashed the check.

C Yes, because the check did not represent payment in full (i.e., the contract price) for the work done.

D Yes, because the heating company should have been given more time to cure the defect.

A

The heating company is not likely to prevail in its suit seeking to recoup the $450 from the homeowner.

An accord is an agreement in which one party to an existing contract agrees to accept, in lieu of the performance that she is supposed to receive from the other party to the existing contract, some other, different performance.

Satisfaction is the performance of the accord agreement. Satisfaction discharges not only the original contract but also the accord contract.

If a monetary claim is uncertain or is subject to a bona fide dispute, an accord and satisfaction may be accomplished by a good faith tender and acceptance of a check when that check (or an accompanying document) conspicuously states that the check is tendered in full satisfaction of the debt.

Here, there was a good faith dispute as to the amount owed. The check the homeowner tendered had a prominent notation that it was payment in full for the two baseboard heaters. By cashing the check, the heating company accepted it, and the accord and satisfaction was complete. The homeowner is discharged from any further performance under the contract. (A) and (D) are wrong because it was the existence of an accord and satisfaction, and not the heating company’s failure to adequately fix the problem, which foreclosed its options to recoup further monies. (C) is wrong because the check represented an accord and thus did not have to be for the contract price.

126
Q

Disclaimer of warranty

A

In every sale of goods, unless expressly disclaimed, there arises a warranty that the goods will be merchantable, which means that they will be fit for the ordinary purposes for which such goods are used.

A television that explodes after five weeks of use likely breaches this warranty.

The warranty will not be disclaimed because, to be effective, a disclaimer must be part of the offer and acceptance process or must be agreed to by the buyer as a modification.

Here, the “disclaimer” was in the box, and the consumer did nothing to accept the disclaimer.

127
Q

A feed store owner agreed to purchase several tons of grain products at a specified price from a large supplier of cattle feeds. The supplier later failed to deliver the promised grains, and the owner was forced to cover by purchasing from local producers at a higher price. The owner contacted a large law firm in the city and obtained their agreement to represent him in connection with his possible claims against the supplier. Due to error, the applicable statute of limitations period passed without the filing of any action on the owner’s behalf. The owner retained another lawyer and sued the large law firm for malpractice. The jurisdiction retains traditional contributory negligence.

In addition to the firm’s negligence, what else does the owner have to establish as part of his prima facie case?

A He had a good faith claim against the supplier that was lost by the law firm’s dilatoriness.

B He would have recovered from the supplier if an action had been timely filed.

C He did not contribute to the failure to timely file an action through his own negligence.

D The losses resulting from breach of the sales agreement by the supplier severely harmed his financial situation.

A

The owner will have to show that he would have recovered damages in his lawsuit.

The following elements must be proved for a prima facie case of negligence: (i) the existence of a duty on the part of the defendant to conform to a specific standard of conduct for the protection of the plaintiff against unreasonable risk of injury, (ii) breach of that duty by the defendant, (iii) that the breach of duty was the actual and proximate cause of the plaintiff’s injury, and (iv) damage to the plaintiff’s person or property. Here, the owner can establish that the law firm breached its professional duty of care by failing to file a claim within the statute of limitations. He must also establish that this breach was an actual and proximate cause of his damages, which here would be the loss of the contract damages that he could have recovered from the breach by the supplier. (A) is incorrect because merely having a good faith claim that was lost because of the firm’s negligence is not sufficient. The owner has to show by a preponderance of the evidence that he suffered damages because of the firm’s negligence. (C) is incorrect because it states a defense rather than part of the prima facie case; any contributory negligence on the owner’s part must be pleaded and proved by the law firm to either defeat or reduce his recovery. (D) is incorrect because it is irrelevant whether the breach by the supplier severely harmed the owner’s financial situation. The only issue is whether he would have been able to recover any of his losses had he timely filed a breach of contract action. If he establishes that he would have recovered, then the law firm’s negligence was an actual and proximate cause of his suffering damages.

128
Q

An owner and a builder executed a contract providing that the builder was to construct a residence on a specified lot according to plans and specifications. The total contract price was $800,000. No date was included in the contract for completion of the home. After the builder completed 60% of the residence, a flash flood from a nearby river partially eroded the lot but left the construction undamaged. The builder determined that it would cost an additional $1.7 million to repair the lot so that the residence can be constructed according to the plans. Without the additional lot repair work, the residence cannot be constructed at all.

Which of the following states the probable legal consequences of the lot erosion?

C The builder is discharged from his duties under the contract because of impracticability.

D The builder remains obligated to perform under the contract, but he may bring an action against the owner for the increased costs of construction.

A

The builder will be discharged from his duties under the contract. Modern courts recognize that impracticability due to excessive and unreasonable difficulty or expense is a defense to breach of contract for nonperformance. Since the cost to the builder to perform under the original contract would exceed more than double what he would be paid, he likely would be excused from performance by commercial impracticability.

(D) is therefore incorrect. Unlike destruction of the building itself before completion, which will not discharge a contractor’s duty, the erosion of the lot, which destroys the means of performing the contract, will generally not be one of the risks that a builder will be deemed to have assumed. (A) is wrong because the builder’s bankruptcy is irrelevant for this purpose. (B) is wrong because there is no mutual mistake here.

129
Q

promise to forbear suit

A

Modern courts would hold that a promise to forbear suit on a claim that the promisor honestly and reasonably believes to be valid is good consideration to support an agreement, even if the claim ultimately turns out not to be valid.

130
Q

A young woman went to her local shoe shop and selected a pair of shoes. She gave the salesperson cash for the shoes. As the salesperson was putting the shoes into a bag, a robber brandishing a gun entered the store, forced the salesperson to put all of the money in the register into the bag with the shoes, and fled with the bag, the money, and the shoes. After the police had come, the young woman asked the salesperson to get her another pair of shoes. He told the young woman that she would have to pay for them again. The young woman refused.

If the young woman sues the shoe shop for another pair of shoes, who will prevail?

A The young woman, because she did not yet have possession of the shoes.

C The shoe shop, because title to the shoes had already passed to the young woman.

D The shoe shop, because the contract goods had already been identified.

A

The young woman will prevail.

Where the seller is a merchant, the risk of loss does not pass to the buyer until the buyer takes physical possession of the goods.

(C) is wrong because passage of title does not shift the risk of loss in this case. (D) is wrong because while a buyer gains some rights once the goods are identified, identification does not shift the risk of loss.

131
Q

third-party beneficiaries have (no) rights against promisees

A

no

meaning, third party beneficiary can’t sue promisee if the promisor doesn’t perform

i.e.

father gave an agency money in return for a promise that the agency will pay son’s tuition but the agency didn’t

132
Q

The handyman was hired to do a job, and did so to the best of his ability. However, the man who hired him failed to inform him that he had moved. That caused the tiles to be fixed on the wrong house.

A

This question involves a misunderstanding under Restatement (Second) of Contracts Section 20.

When both parties communicate a term to a contract, but both mean different things and neither has any reason to know the other’s meaning, there is no contract.

However, as here, where one party has no reason to know a different meaning, but the other party does have reason to know, the contract is operative in accordance with the meaning of the unaware party.

Here, the handyman had no reason to know the man meant anything different by “in my house” other than the house the man had lived at for several years, whereas the man should have known that the handyman would have this misunderstanding if he didn’t give his updated address. As such, the contract is valid, and is for the repair of the bathroom at the man’s old address.

133
Q

There are several kinds of power of attorney.

A

A general power of attorney gives broad authority to the agent. The agent may thereafter make medical decisions, legal choices, or financial or business decisions for the principal.

A special power of attorney pertains to a limited authority to be exercised by the agent. In the latter, the agent can only perform the specific tasks as stated in the power of attorney.

Typically, a power of attorney becomes invalid when the principal becomes mentally incompetent or dies. However, a durable power of attorney pertains to an authority which subsists even if the principal becomes incapacitated. As compared to a general power of attorney and a special power of attorney, a durable power of attorney is not extinguished by the incapacity of the principal.

134
Q

Under the doctrine of impossibility,

A

both parties to a contract may be excused from their obligations under the contract if performance has been rendered impossible by events occurring after the contract was formed. This doctrine requires that performance be objectively impossible and that the occurrence of the contingency be unknown to the parties at the time of the contract. Contracts may be objectively impossible or subjectively impossible. Only objectively impossible contracts will allow for performance to be excused. Here, performance on the contract is subjectively impossible because of some failure or fault on the part of the performing party. The seller chose not to purchase the jade from the out-of-state seller and as a result of that failure, the seller caused the impossibility. In such circumstances, the performance obligation is not excused and will be considered a breach of the contract.

135
Q

common law option contract

v.

UCC firm offer

A

Question concerns a sale of goods by a merchant to a non-merchant. If this were a UCC firm offer, it would not need a payment to be effective. Rather, a written offer made by a merchant that expressly states that it will be held open is effective, but becomes revocable after three months. By adding the consideration, as the parties did here, they are making a common law option contract, which can be irrevocable for more than three months.!!!!!!!!!!!!!!!

UCC firm offer (a written offer made by a merchant that expressly states that it will be held open) is only irrevocable for three months. However, with the added element of consideration, the parties have a common law option contract, which can remain irrevocable longer.

136
Q

Under the UCC, the preexisting duty rule no longer applies.

A

However, even if the parties agree to contract modifications that involve new consideration, they must do so in good faith.

i.e.

Here, the event planner needed to delay the delivery for two hours, and this caused the ice cream chef to need additional supplies to keep the ice cream cold. There is no evidence that either of them acted other than in good faith. This was therefore a valid modification.

137
Q

Debts that are barred by the statute of limitations can still be enforceable without new consideration and are binding to the extent of the new promise if

A

there is a subsequent promise to pay.

i.e.

The grandfather is entitled to $2,500, because the grandson’s moral obligation to pay his grandfather $5,000 became the consideration for the grandson’s agreement to pay the grandfather $2,500.

138
Q

Unilateral K

A

A unilateral contract is created when the offeror requests acceptance of the offer by the performance of an act rather than by a promise to perform the act.

The offer is accepted only when the offeree performs the requested act.

An offer is revoked by operation of law upon the death of the offeror.

i.e.

Here, given that the businessman’s offer requested acceptance by performance rather than by a promise to perform, the dealership accepted the offer when it performed the act that the businessman had requested (i.e., extending his son credit). Therefore, the businessman’s offer created a unilateral contract, which can be enforced against his estate. The fact that the dealership notified the businessman of its acceptance of the offer after his death is not the controlling factor; what is controlling is that by performing the requested act, the dealership validly accepted the offer before he died.

139
Q

If the man says that he is not happy with the work and his dissatisfaction is in good faith, he will not have to pay the contract price, because the condition of satisfaction has not been met.

A

However, if his dissatisfaction is not in good faith, the court will consider the satisfaction condition fulfilled and the man will have to pay the artist the contract price.

140
Q

UCC Section 1-303(b) defines a course of dealing as “a sequence of conduct concerning previous transactions between the parties to a particular transaction . . . .” In this case, on several past occasions, the baker had taken a 10% discount without objection from the manufacturer, thereby establishing a course of dealing.

A

Under the UCC’s parol evidence rule, course-of-dealing evidence is admissible to explain or supplement a final written agreement even if the parties intended the agreement to be complete and exclusive.

141
Q

The doctrine of impossibility excuses both parties from their obligations under a contract if the performance has been rendered impossible by events occurring after the contract was formed. In order to apply, the doctrine of impossibility requires:

A

(1) objectively impossible performance; and
(2) that the occurrence of the contingency must not be known to the parties at the time of contracting.

Objective impossibility occurs when the performance under contract becomes literally impossible because of circumstances beyond the control of the parties.

Here, neither party was aware of the asbestos before entering into the contract. As a result, it has become impossible for the contractor to perform in time, because now there are no available appointments for the city inspector before December 10. As such, the contractor should prevail, because his performance has become impossible due to circumstances beyond his control.

142
Q

SoF and emails!!!

The Statute of Frauds requires a contract for the sale of land to

identify the parties,

contain a description of the land,

evidence an intent to buy and sell,

recite (usually) a price term, and

be signed by the party against whom enforcement is sought.

A

The email messages here fulfill those requirements. Courts are liberal regarding the nature of a signature; it need only reflect an intent to authenticate the writing. Both the tenant’s and the realtor’s names were placed below the word “Signed,” which adequately reflected their desire to be bound.

Note that the other requirements of description of the property and consideration are satisfied in addition to the signature of the seller.

143
Q

It is well settled that in a situation where emergency medical services are performed, …

Where the patient is unable to request or accept treatment, the law…

A

… the party rendering the services must have a reasonable expectation of payment.

….implies acceptance based on the theory that any person in such circumstances has both the desire and the right to recover.

***

Courts have determined that any medical treatment properly performed is a benefit in that it increases the chances of survival.

144
Q

When a sales contract provides that a seller may retain the buyer’s earnest money as liquidated damages,

A

courts routinely uphold the seller’s retention of the money upon breach if the amount appears reasonable in light of the seller’s anticipated and actual damages.

Many courts uphold retention of earnest money of up to 10% of the sales price without inquiry into its reasonableness.

145
Q

A customer selected a new wallet at a local department store that the salesperson said was made of the finest calfskin and was stitched by hand. The customer bought the wallet and left the store. A few moments later, he took out the wallet to transfer his cash and credit cards into it. On close inspection, he noticed a small nick in the leather. He immediately went back to the department store and demanded a refund. The salesperson refused.

If the customer sues for a refund, who will prevail?

A The customer, because there was a breach of contract.

B The customer, because he had a reasonable time after purchase in which to inspect.

C The department store, because the customer accepted the goods.

D The department store, because the customer did not give written notice of the breach.

A

Once a buyer has accepted goods, his right to reject for nonconformity generally lapses and his only remedy is a suit for damages. Acceptance usually occurs when the buyer takes possession of the goods. In some cases, the buyer can revoke acceptance, but the breach must be substantial and the buyer must have a good reason for accepting the goods (i.e., something more than not taking the time to carefully inspect). Here, the customer accepted the goods and the breach appears minor. (A) entitles the customer only to damages, not a full refund. (B) is a misstatement of the law. (D) is wrong because written notice is not required; oral notice is acceptable.

146
Q

A wealthy sportsman purchased a large old wooden sailing ship for $200,000. Although the boat was a classic, the sportsman wanted it to be modernized and made more comfortable. To that end, the sportsman entered into a written contract with a marine architect-engineer to draw up and then execute the modernization plans, for $7,500.

At the time the parties entered into the agreement, the sportsman told the architect-engineer that his modernization plan would be subject to the approval of the sportsman’s sister, that they would, in fact, have no deal unless the plans meet with her approval. The architect-engineer agreed to this. He finished his drawings and submitted them to the sportsman, who was enthusiastic about the designs. The sportsman’s sister, a famous yachtswoman, was engaged in a trans-Pacific yacht race at the time and was not expected home for a number of weeks. Cheered by the sportsman’s enthusiasm, the architect-engineer went ahead and modernized the ship according to his designs. When he finished the work, he submitted a bill to the sportsman, who refused to pay, pointing out that his sister had never approved the designs.

If the architect-engineer sues the sportsman, which of the following issues of contract law is most likely to be decisive in determining the outcome of the case?

A Statute of Frauds.

B Parol evidence rule.

C Rules of construction.

D Conditions precedent.

A

Where there is an oral condition precedent, evidence of the condition falls outside the parol evidence rule.

The parol evidence rule provides that where the parties to a contract express their agreement in a writing with the intent that it embody the final expression of their bargain, any other expressions, written or oral, made prior to the writing, as well as any oral expressions contemporaneous with the writing, are inadmissible to vary the terms of the writing. Certain forms of extrinsic evidence are deemed to fall outside the scope of the parol evidence rule. For instance, a party to a written contract can attack the validity of the agreement. One way of doing so is by asserting that there was an oral agreement that the written contract would not become effective until the occurrence of a condition. Such a condition would be deemed a condition precedent to the effectiveness of the agreement, and evidence of the condition will be freely offered and received.

i.e.

Here, the sportsman and the architect-engineer have entered into a written agreement that apparently embodies the full and final expression of their bargain. However, the sportsman’s statement at the time of entering into the agreement indicates quite clearly that the parties had no agreement absent the approval of his sister, and the architect-engineer agreed with this statement. Thus, there is an oral agreement that the written contract would not become effective until the occurrence of a condition precedent. As discussed above, evidence of this oral condition does not come within the purview of the parol evidence rule and is therefore admissible. Consequently, the sportsman can assert the nonoccurrence of a condition precedent as a way to avoid liability on the contract.

147
Q

. When one of the original parties to a valid contract assigns his rights under the contract to a third party, the assignee may

A

enforce his rights against the obligor directly but is generally subject to any defenses that the obligor had against the assignor.

As long as the defense is inherent in the contract, such as failure of a condition, it is always available against an assignee because it was in existence when the contract was made (even if whether the obligor would be able to utilize it was uncertain).

148
Q

The owner of a television agreed to sell it to a neighbor for $250. The neighbor made a down payment of $70, took possession of the television and agreed to pay the outstanding balance in nine equal $20 installments, beginning on June 5, with subsequent installments due on the fifth of each month until the balance was paid in full.

The neighbor’s friend owed her $200. On May 20, the neighbor and her friend entered into an oral agreement whereby the friend agreed to make the nine $20 installment payments to the seller in exchange for the neighbor’s promise to forgive the friend’s $200 debt. On June 7, the seller called the neighbor to ask her where his first $20 installment payment was, and she told him at that time of her agreement with her friend. The friend has made none of the installment payments.

If the seller files suit against the friend demanding payment, who will prevail?

A The seller, because he was a third-party beneficiary of the agreement between the neighbor and her friend.

B The seller, because he is an assignee of the neighbor’s rights against her friend.

C The friend, because there was no consideration for her promise to the neighbor.

D The friend, because the surety provision of the Statute of Frauds prevents the seller from enforcing the friend’s promise.

A

The neighbor has delegated her duties under the agreement with the seller to her friend, and the friend has agreed to assume the duties by agreeing to make the installment payments to the seller. Where a delegate’s promise to perform the delegated duty is supported by consideration, there results a third-party beneficiary situation, so that the nondelegating party to the contract can compel performance or bring suit for nonperformance. The friend’s promise to make the payments to the seller, totaling $180, was given in exchange for the neighbor’s promise to forgive the $200 debt owed by the friend to her. The neighbor thus relinquished her right to take action against her friend for the full amount owed, thereby incurring legal detriment. Consequently, the promise of the friend was supported by consideration, and a situation arose in which the seller became a third-party beneficiary of the agreement between the neighbor and her friend, and able to enforce performance of the friend’s promise to pay.

149
Q

The owner of a semi-pro baseball team offered a former player a position as the team’s manager. During negotiations, the owner agreed to pay the manager $1,000 per week, but insisted that if the manager quit during the season, he would have to pay a “penalty” of $500 per week for each week that he did not manage the team, because it would cost the owner several thousand dollars to replace the manager during the season. The parties agreed in writing to those terms. The manager managed the team for 14 weeks with only mixed success. Nevertheless, with 10 weeks left in the season, he was offered and accepted a job as manager of a professional minor league baseball team. Fortunately for the owner of the semi-pro team, the manager’s replacement had great success with the team, causing attendance to skyrocket.

If the owner of the semi-pro team brings suit against the manager to recover $5,000, the amount due under the “penalty” provision, is the owner likely to prevail?

A No, because “penalty” clauses in contracts are not enforceable.

B No, because the owner of the semi-pro team was not harmed by his breach.

C Yes, because the manager can be penalized for a willful breach.

D Yes, because the “penalty” provision is enforceable.

A

The owner of the semi-pro team is likely to prevail. Although the $500 per week damages was denominated a “penalty” by the parties, in fact it operates as a reasonable liquidated damages clause.

The parties to a contract may stipulate what damages are to be paid in the event of a breach if

(i) damages are difficult to ascertain at the time the contract is formed, and
(ii) the amount agreed on is a reasonable forecast of compensatory damages in the case of a breach.

These conditions have been met here. The owner stated that if he had to replace the manager in the middle of the season, it would cost several thousand dollars, which seems to be a reasonable forecast of the compensatory damages.

150
Q

A new college graduate entered into an oral agreement with a freshman to lease the freshman her mini-refrigerator for a term of four years. The freshman was to pay the graduate $20 a month, of which $10 of the monthly charge was to be paid directly to the graduate’s parents, in satisfaction of a debt the graduate owed her parents. While the graduate was putting the agreement into writing she accidentally failed to include the agreement to pay her parents directly. The freshman also failed to notice that the direct payment provision was missing before she signed the contract, which the graduate signed.

If the parents bring an action against the freshman, which of the following will have the greatest effect on the outcome?

A Whether the parents were a party to the agreement between the graduate and the student.

B Whether the agreement between the graduate and the freshman was completely integrated.

C Whether the graduate was negligent in not discovering that the agreement omitted mention of the payment of money directly to her parents.

D Whether the freshman was negligent in not discovering that the agreement omitted mention of the payment of money directly to the parents.

A

The most critical factor will be whether the agreement was completely integrated.

The effect of a completely integrated agreement, meaning that the writing embodies the entire agreement of the parties, is that evidence could not be introduced to show a prior or collateral oral agreement.

(A) is incorrect because the parents need not be a party to the contract; they have rights as third-party beneficiaries.

(C) and (D) are incorrect because the negligence of either of the parties has no bearing on whether the collateral oral agreement can be proved or enforced.

151
Q

Upon graduation from high school, a young man wanted to enroll in a nine-month program at a community college to study to be an electrician, but he could not afford tuition and the costs of being unemployed for that time period. His uncle told him that if he enrolled and participated in the program, he would pay his tuition and living expenses for the time involved, and that he would also pay him a $1,000 bonus for each “A” he earned as a final grade in a class. The young man told his uncle that he would enroll in the program. The next day, the young man’s grandfather called and told him that he had learned of the uncle’s offer and that if the uncle failed to pay the young man as promised, he (the grandfather) would. The young man attended the program and earned “A’s” as final grades in three classes. Shortly thereafter, the uncle died, and the executor of the uncle’s estate refused to pay the young man the bonus for each of the three “A’s.” When the young man told his grandfather that the uncle’s estate refused to pay, his grandfather sympathized but said he no longer thought it was a good idea to pay for grades. He too refused to pay.

If the young man brings suit against his grandfather for breach of contract, which of the following represents his grandfather’s best defense?

A The contract was illusory.

B The contract was oral.

C There was no consideration flowing to the grandfather.

D The fact that the young man received nine months’ worth of free education and living expenses was sufficient compensation for his efforts in earning the three “A’s.”

A

The young man will not succeed in trying to enforce his grandfather’s promise because the promise was not in writing, as is required under the Statute of Frauds.

Generally, contracts do not have to be in writing to be enforceable; however, under the Statute of Frauds, certain contracts will not be enforceable unless they are evidenced by a writing signed by the party to be charged. One such contract is to pay the debt of another, such as the grandfather’s promise here to pay the uncle’s debt if he does not pay.

(A) is wrong because the contract clearly is not illusory. Each party is bound by his promise to the other party, so the requisite mutuality exists. The young man agreed to (and did) attend the nine-month program, and he worked hard and earned three “A’s,” by which he incurred a legal detriment because he was doing something he was under no legal obligation to do. The grandfather agreed to pay the young man’s tuition and living expenses plus a $1,000 bonus for each “A” he earned as a final grade in a class if the uncle failed to pay the young man as promised. Detriment to the promisee in performing an act or making a promise is valid consideration. Thus, the contract was not illusory. (C) is wrong because, as discussed above, consideration flowed to the grandfather when the young man participated in the program and earned the three “A’s,” neither of which he was legally obligated to do. (D) is wrong because nine months’ worth of free education and living expenses was not the full consideration for which the parties bargained.

152
Q

A publisher entered into a contract with a paper manufacturer who used very fine materials, whereby the publisher was given the right to purchase all paper refined by the paper manufacturer for the next five years at a price set at 95% of the domestic market price at the time of delivery. The publisher agreed to purchase no less than 1,000 pounds of paper a week. At the time this contract was signed, the publisher gave written notice to the paper manufacturer that it intended to buy all paper produced by the paper manufacturer until further notice. The paper manufacturer then sold its business to a lumber-processing company.

What is the effect of this sale on the paper manufacturer’s obligation to the publisher?

A The sale discharges the paper manufacturer’s obligation to the publisher because there has been a full performance.

B The paper manufacturer is liable for damages if the lumber processing plant fails to deliver paper to the publisher.

C The paper manufacturer is excused from further performance because it no longer has a factory to produce paper.

D The paper manufacturer breached its contract with the publisher.

A

The paper manufacturer is liable for damages if the lumber processing plant fails to deliver paper to the publisher. Because delivery of paper is not personal in nature, that duty can be delegated. The quantity will be measured by the paper manufacturer’s original output. However, when a duty is delegated to a delegate, the delegator remains liable should the delegate fail to perform. (A) is incorrect because the contract was for five years, and five years have not yet elapsed. (C) is wrong because a delegator remains liable. (C) would be correct only if the paper manufacturer went out of business without delegating its duties to another by selling the other the business, not the case here. (D) is wrong because, as indicated above, such a delegation is proper.

153
Q

A man loaned $10,000 to his good friend who was heavily in debt. He told his friend that he could have until the following June to repay the loan and to make repayment directly to the man’s grandson. The man knew that his grandson, who would be graduating from law school in June, was hoping to buy a new car upon graduation and could use the money as a down payment on the car. Neither the man nor his friend told the grandson of this agreement. Before the friend repaid the $10,000 loan, the grandson went out and bought his dream car, cashing some savings bonds to make the down payment. A week later, the grandfather suffered a reversal of fortune and told his friend to pay the $10,000 to him after all, which the friend did. A short time later, the grandson learned of the aborted plan to pay the $10,000 to him.

If the grandson sues the friend for the $10,000, will he prevail?

A Yes, because he was the intended beneficiary of the contract between his grandfather and the friend.

B Yes, because he detrimentally relied on the promise between his grandfather and the friend by purchasing the car.

C No, because his rights had not yet vested when the contract was modified.

D No, because the man intended the repayment of the $10,000 to the grandson to be a gift.

A

The grandson will not prevail in his suit. He was an intended beneficiary whose right to collect the $10,000 had not vested at the time the contract was modified. Since the grandson’s rights had not vested, the man and his friend were free to modify the contract, and the grandson could not enforce the original agreement between his grandfather and the friend.

The grandson was clearly an intended third-party beneficiary; he was expressly designated in the contract and performance was to be made directly to him. A court must then decide whether his rights have vested, which is necessary to enforce the contract. Vesting occurs when the beneficiary:

(i) manifests assent to the promise in a manner invited or requested by the parties;

(ii) brings suit to enforce the promise; or

(iii) materially changes position in justifiable reliance on the promise.

Here, the grandson’s rights had not vested. Neither his grandfather nor the friend told the grandson of the arrangement, so he could not have manifested assent to it. The grandson did not bring suit to enforce the promise prior to the modification of the contract, and he did not materially change his position in justifiable reliance on the promise because he did not know about the agreement when he bought the car. (B) is incorrect because there was no detrimental reliance. The facts do not indicate any action taken by the grandson in reliance on the contract. He did not know about it. (A) is incorrect because although the grandson was indeed an intended beneficiary of the contract, that alone would not be enough to enforce the contract where his rights had not vested prior to the contract’s modification. (D) is incorrect because the general rule regarding the vesting of rights in a third party applies to a donee beneficiary (the beneficiary of a contract benefit intended as a gift) as well as to a creditor beneficiary, so long as he is an intended beneficiary to the contract. Thus, the fact that the grandson was a donee beneficiary would not prevent him from prevailing in a suit had his rights been vested.

154
Q

The owner of an art gallery entered into a written contract with an avid art collector whereby the art collector agreed to buy and the gallery owner agreed to sell for $7,500 any painting in the gallery by artist Alpha. The contract was to be executed on July 6 according to its written terms.

The art collector went to the gallery on July 6 with a certified check in the amount of $7,500. The art collector pointed out a painting by a different artist hanging on the wall, and told the gallery owner that that was the painting he wanted, and that he would also take its old-fashioned $250 gilt frame to go with it. The gallery owner responded that the painting was by the artist Beta, but that the art collector could have it with the frame if he was willing to pay $250 extra for it.

This enraged the art collector, and he filed suit against the gallery owner, asserting in his pleading that he remains able and willing to tender $7,500 to the gallery owner.

He also asserts that prior to signing the contract, the parties agreed orally that the art collector could have a painting by Beta for the same price in lieu of one by Alpha, and that the gallery owner would throw in the frame for whatever painting he chose. The gallery owner denied that any such conversation took place. There are no other witnesses.

About which agreements should the court allow the art collector to testify?

A The oral agreement for the painting, but not the oral agreement for the frame.

B The oral agreement for the frame, but not the oral agreement for the painting.

C Both the oral agreement for the painting and the oral agreement for the frame.

D Neither the oral agreement for the painting nor the oral agreement for the frame.

A

The court should allow the art collector to testify regarding just the oral agreement for the frame.!!!!!!!!!!!!!!!!!!!!!!!!

_Contractual terms *that are set forth* in a writing intended as a final expression of the parties’ agreement cannot be contradicted by evidence of any prior agreement or contemporaneous oral agreement._ Although this parol evidence rule prohibits contradicting the writing, the terms of the writing may be explained or supplemented by consistent additional terms, unless the court finds from all the circumstances that the writing was intended as a complete and exclusive statement of the parties’ agreement. To determine whether the parties intended the writing to be the complete and exclusive statement of their agreement, it must be determined whether parties situated as were the parties to this contract would naturally and normally include the extrinsic matter in the writing.

Here, the writing at issue states clearly that the painting subject to sale is any painting by Alpha. The art collector’s assertion of a prior agreement allowing him to buy a painting by Beta clearly contradicts the terms of the writing. Consequently, the parol evidence rule will render inadmissible testimony as to such an alleged agreement. (A) and (C) are therefore incorrect. The assertion that the parties agreed prior to signing the writing that the art collector could have a $250 frame at no additional cost does not contradict any of the terms of the writing.

However, it does supplement those terms.

As noted above, such supplementation is permitted unless there is a finding that the writing was intended by the parties as a complete and exclusive statement of the terms of their agreement.

Under the UCC, which applies here because a sale of goods is involved, a writing is presumed not to be the complete and exclusive integration of all of the terms of the agreement. While the presumption may be overcome if the parties actually intended a total integration or it is certain that similarly situated parties would have included that term, there is no indication of that in these facts. Given that the subject of the contract was a $7,500 painting, a promise to throw in a frame priced at a fraction of the cost of the painting is likely to be found to be a consistent additional term. Given this finding, evidence of the alleged agreement regarding the frame will be admissible for the purpose of supplementing the terms of the writing. Thus, (D) is incorrect.

155
Q

A woman purchased a bottle of hair dye at a drugstore. The bottle was labeled “ash blonde,” but due to a packaging mistake at the factory, the bottle actually contained a “rose gold” shade of dye. The two colors appear identical while still in the bottle, but are noticeably different once applied to hair. The woman discovered the mistake later that week when her hair turned to a perfect shade of rose gold after applying the dye.

Can the woman recover damages from the drugstore for breach of the implied warranty of merchantability?

A Yes, because the woman had a particular purpose in mind when selecting that shade of dye and relied on the packaging when she selected it.

B Yes, because the bottle was mislabeled.

C No, because the hair dye worked properly.

D No, because the factory, not the drugstore, was responsible for the mistake.

A

The drugstore breached the implied warranty of merchantability because the bottle was mislabeled.

Implied in every contract for sale by a merchant who deals in goods of the kind sold, there is a warranty that the goods are merchantable.

To be merchantable, goods must be adequately contained, packaged, or labeled according to the contract and must conform to any promises or affirmations of fact made on the label.

The bottle the woman purchased was labeled “ash blonde” when it actually contained “rose gold” dye. The product did not conform to its label, thus the drugstore breached the warranty of merchantability with the sale.

(A) is not the correct answer because it mistakenly attempts to argue that there was a breach of the warranty of fitness for a particular purpose, rather than addressing the warranty of merchantability. The implied warranty of fitness for a particular purpose is implicated in a sale of goods whenever the seller has reason to know the particular purpose for which the goods are to be used and that the buyer is relying on the seller’s skill and judgment to select suitable goods and the buyer in fact relies on the seller’s skill or judgment. A particular purpose differs from the ordinary purpose for which goods are used in that it envisages a specific use by the buyer which is peculiar to the nature of her use, whereas the ordinary purposes for which goods are used are those envisaged in the concept of merchantability. (C) is also incorrect. While the implied warranty of merchantability requires that goods are fit for the ordinary purpose for which such goods are used, this does not mean that mislabeled goods, that are not otherwise defective, meet this standard. As discussed above, to be merchantable goods must conform to any promises or affirmations of fact made on the label. (D) is incorrect because it makes no difference that the factory was responsible for the mistake and that the drugstore did not know of the defect or could not have discovered it. Implied warranties are not based on negligence but rather on absolute liability that is imposed on sellers who deal in goods of the kind sold, such as the drugstore here.

156
Q

A landowner orally agreed to sell 100 acres of land to a buyer for $10,000. As a condition of the sale, the buyer agreed to pay $5,000 of the purchase price to the landowner’s creditor. The buyer’s attorney drafted the contract, which both the landowner and the buyer read before signing. The signed document made no reference to the payment to the landowner’s creditor, and neither party noticed the oversight.

In an action by the creditor against the buyer for $5,000, which of the following facts, if proved, would be most important?

A The buyer was negligent in not having carefully read the written agreement.

B The landowner-buyer agreement was completely integrated.

C The terms of the signed document are unambiguous.

D The omission of any reference to the creditor from the written document was accidental.

A

In making a determination as to who would prevail in an action by the creditor against the buyer, the most important fact would be that the omission of any reference to the creditor in the written document was accidental.

If there is an agreement between the parties, the agreement is put into writing, and there is a variance between the original agreement and the writing, the writing can be reformed to reflect the intent of the parties.

A plaintiff who wants to obtain reformation of a contract must show that there was an antecedent agreement that is not correctly reflected in the writing (e.g., by mistake).

These requirements are met here because the writing fails to include a provision that was included in the earlier oral agreement concerning payment of part of the purchase price to the landowner’s creditor.

(A) is wrong because the negligence of either the buyer or the landowner should not adversely affect the creditor’s claim where the facts do not suggest that either party assumed the risk of the mistake.

(B) is wrong because, even if the landowner-buyer agreement was completely integrated, the contract could still be reformed for mutual mistake if the omission of any reference to the creditor from the written document was accidental.

Under the parol evidence rule, completely integrated writings cannot be contradicted or supplemented by either written or oral expressions made prior to the writing or oral expressions made contemporaneously with the writing, but the parol evidence rule does not apply if a party to a written agreement alleges facts that entitle him to reformation of the agreement.

(C) is wrong because, while it is true that the terms of the writing are unambiguous, the answer turns on a mutual mistake involving omission of a material term and not on an ambiguity in the writing, and, as explained above, such mutual mistake allows for reformation of the contract.

157
Q

A car owner was having problems with her car’s brakes. The brake shop to which she brought her car told her that the entire hydraulic brake system needed to be replaced at a cost of $1,800. The car owner agreed to have the work performed, and the shop replaced the brake system. When the car owner picked up the car, she insisted on testing the brakes before paying for the work. During a test drive, the brakes squeaked when compressed but otherwise worked perfectly. The mechanic told the car owner that the squeak would disappear on its own in a few days. The car owner stated that she would pay the shop when the squeak disappeared and left with her car. A month later, the brakes still squeaked and the car owner refused to pay for the work. The shop files suit to collect the $1,800 contract price.

What is the likely outcome of the suit?

A The car owner must pay the shop the $1,800, because she got the substantial benefit of her bargain.

B The car owner must pay the shop the $1,800, but is entitled to set off the amount necessary to fix the squeak.

C The car owner does not have to pay the shop anything, because the squeak amounted to a breach of contract that relieved the car owner of her duty to perform.

D The shop must “cure” by providing the car owner with a new set of brakes that do not squeak, after which the car owner must pay the shop the $1,800.

A

Because the shop’s breach of contract was minor, the car owner must pay the brake shop the $1,800, but she would be entitled to damages to remedy the squeak.

A breach of contract is minor if the nonbreaching party gets the substantial benefit of her bargain, despite the other party’s defective performance. A minor breach does not relieve the nonbreaching party of her duty to perform under the contract, although she will be entitled to remedial damages for the breach.

Here, the brakes worked perfectly except for the squeak and, most importantly, stopped the car when compressed, as brakes are supposed to do. Thus, the breach was minor and would not relieve the car owner of her own duty to perform (i.e., to pay the bill), although she could get damages, if the court so decided, for the squeak.

(A) is wrong because it does not take into account the fact that the car owner could be entitled to damages for the shop’s breach. (C) is wrong because, as noted above, the breach is minor and does not relieve the car owner of her duty to perform. (D) is wrong because a seller’s right to cure when goods or their tender are not perfect may not apply here, because the facts do not establish whether the defect causing the brakes to squeak is in the product itself or in its installation. The UCC rules regarding the right to cure do not apply to service contracts (e.g., faulty installation of brakes). Moreover, the right to cure arises when the buyer has rejected goods because of defects, and here the car owner did not reject the brake system—she took the car home with her knowing of the squeaking brakes. Thus, it would be too late for a “cure.”

158
Q

A retailer and a wholesaler entered into an agreement under which the wholesaler would supply the retailer with goods and the retailer would make payment to one of the wholesaler’s creditors. Before the creditor became aware of the agreement, the retailer and wholesaler agreed that the retailer instead would pay the wholesaler directly. The creditor sued to enforce the original agreement between the retailer and the wholesaler.

Which of the following statements best describes the rights of the creditor?

A The creditor is an incidental beneficiary only and has no rights under the agreement between the retailer and the wholesaler.

B The creditor is an intended beneficiary and has rights against both parties under the agreement between the retailer and the wholesaler.

C The creditor is an intended beneficiary of the agreement but has no rights against the retailer.

D The creditor’s rights as a third-party beneficiary vested when it brought suit to enforce the agreement between the retailer and the wholesaler.

A

The creditor is an intended beneficiary but has no rights against the retailer because of the modification. A third-party beneficiary of a contract can enforce a contract only when its rights have vested. Vesting occurs when the third party (i) manifests assent to the promise; (ii) brings suit to enforce the promise; or (iii) materially changes its position in reliance on the promise. Prior to vesting, the original parties are free to modify or rescind the third-party beneficiary’s rights. Here, the third-party beneficiary (the creditor) did not become aware of the original agreement until after the wholesaler and the retailer modified the agreement. Because the creditor’s rights had not vested, the parties were free to modify the agreement, which they did. The modification rescinded the creditor’s rights under the contract. (A) is wrong because performance was to be given to the creditor under the agreement; hence, the creditor is an intended beneficiary rather than an incidental beneficiary. (B) is wrong because, as discussed above, the creditor no longer has rights against the retailer after the modification of the agreement. Note that the creditor still may sue the wholesaler on the original debt. (D) is wrong because, as stated above, the modification of the agreement between the retailer and the wholesaler ended the creditor’s rights as a third-party beneficiary. Hence, bringing suit did not vest any rights for the creditor.

159
Q

A bulk retailer of accessories for musical instruments placed an advertisement in a trade magazine popular with those in the music business, offering for sale 50-count boxes of a particular type of mouthpiece for use with the French horn, minimum purchase 10 boxes, at $100 per box. In response to the advertisement, the owner of a large store that sold brass and woodwind instruments in its shop and over the Internet sent a written order to the bulk retailer for 12 boxes (50-count) of the mouthpiece. In his letter that accompanied the order, the store owner stated that he would send the bulk retailer his payment of $1,200 upon delivery. The letter also said that the mouthpieces must fit onto three specified models of French horn.

The day after receiving the written order and letter from the store owner, the bulk retailer shipped 12 boxes (50-count) of the mouthpiece to him. Accompanying the invoice on the boxes was a letter from the bulk retailer stating that the mouthpieces are compatible with two of the models of French horn, but that the retailer makes no warranties as to the compatibility of the mouthpieces with any other model of French horn. Shortly after accepting shipment of the boxes, the store owner realized that the mouthpieces did not fit onto the third model of French horn that it had specified and instituted an action against the bulk retailer.

Which of the following statements would offer the strongest support in favor of the store owner’s position?

A The store owner’s letter was an offer, and shipment of the units was an acceptance.

B The store owner’s letter was an offer, and the bulk retailer’s letter accompanying the invoice was an acceptance.

C The bulk retailer’s letter was an offer, and acceptance of the units by the store owner was an acceptance of the offer.

D Shipment of the units was a counteroffer, and acceptance of the units by the store owner was an acceptance of the counteroffer.

A

The store owner’s best position is that his letter was an offer, and shipment of the mouthpieces was an acceptance; thus, the shipment of nonconforming goods both created a contract and a breach of that contract, affording the store owner an immediate cause of action. The contract at issue involves the sale of goods, and is thus governed by Article 2 of the UCC. Under UCC section 2-206, an offer to buy goods for current or prompt shipment is construed as inviting acceptance either by a promise to ship or by current or prompt shipment of conforming or nonconforming goods. The shipment of goods, even though they are nonconforming, is an acceptance creating a bilateral contract and a breach unless the seller reasonably notifies the buyer that the nonconforming goods are offered only as an accommodation to the buyer. Here, the store owner’s letter was an offer that invited the bulk retailer to accept by either a promise to ship or a prompt shipment. The bulk retailer’s shipment of the mouthpieces is an acceptance of the store owner’s offer, because the bulk retailer’s letter accompanying the shipment was probably not sufficient as an accommodation notice (which would have made the shipment a counteroffer rather than an acceptance). The shipment contains nonconforming goods, because the mouthpieces are not compatible with the third type of French horn specified in the store owner’s offer. The shipment of nonconforming goods as an acceptance both created a bilateral contract between the parties and constituted a breach of that contract by the bulk retailer, thus allowing the store owner to sue for any appropriate damages for breach of contract. (B) is incorrect because, since the bulk retailer’s letter was an acceptance, there is a chance that the store owner will be bound by the terms of the letter. Between merchants, additional proposed terms in an acceptance become part of the contract unless: (i) they materially alter the original contract; (ii) the offer expressly limits acceptance to its terms; or (iii) the offeror has already objected to the particular terms, or objects within a reasonable time after receiving notice of them. [UCC §2-207] The bulk retailer and the store owner are both merchants, in that they deal in goods of the kind involved in the contract. Insofar as the bulk retailer’s letter is an acceptance of the store owner’s offer, the sentence disclaiming compatibility of the mouthpieces with other models of French horns is an additional proposed term. The store owner’s offer did not expressly limit acceptance to its terms, nor did the store owner object to the additional terms. Thus, the additional terms will become part of the contract unless the court finds that they materially alter the original contract. If these terms are held to be part of the contract, the store owner will have no cause of action for the incompatibility of the mouthpieces with one of the French horn models. Because (B) creates a framework in which the store owner might lose (i.e., if it is found that the additional terms do not materially alter the original contract), (B) is not as good an answer as (A). (C) and (D) represent the worst scenarios for the store owner. If the bulk retailer’s letter was an offer as stated in (C) and acceptance of the mouthpieces by the store owner was an acceptance of the offer, there is a contract based on the terms in the bulk retailer’s letter; i.e., no warranties as to the compatibility of the mouthpieces with any other model of French horn. In that event, the store owner would have no cause of action against the bulk retailer based on the mouthpieces’ incompatibility with one of the French horn models. The same result would arise if shipment of the mouthpieces were treated as a counteroffer as stated in (D) (this would be the result if the bulk retailer’s letter were held to be a sufficient accommodation notice under UCC section 2-206(1)(b)). Acceptance of the mouthpieces would be acceptance of the counteroffer according to the terms of the letter accompanying the shipment, so that the store owner could not bring an action based on the mouthpieces’ incompatibility with one of the French horn models.

160
Q

implied warranty of merchantability and an express warranty

A

An express warranty will arise from any statement of fact or promise.

Here, the salesperson said that the racket was made of titanium. This is a statement of fact that will give rise to a warranty.

An implied warranty of merchantability will arise in every sale by a merchant unless disclaimed. To be merchantable, goods must be fit for ordinary purposes, and arguably a racket that breaks right away because it is made of plastic is not fit for ordinary purposes.

an implied warranty of fitness for a particular purpose arises only when:

(i) a seller has reason to know the particular purpose for which the goods are to be used and that the buyer is relying on the seller’s skill and judgment to select suitable goods; and

(ii) the buyer in fact relies on the seller’s skill or judgment.

Here, the buyer did not convey a particular purpose. He merely stated he wanted a high end racket. There was no indication that the man was looking to the seller’s skill or judgment in selecting the racket. Thus, the implied warranty of fitness did not arise.

161
Q

A buyer who receives nonconforming goods generally has the right to accept all, reject all, or accept any commercial units and reject the rest.

Here, 20% of the phones shipped were defective, so the retailer had a right to reject.

To properly reject, the rejecting party must,

A

within a reasonable time after delivery and before acceptance, reject the goods or notify the seller of the rejection.

Here, the retailer failed to reject the goods or notify the seller of the defects. He merely transferred payment for less than the contract price. Thus, the retailer cannot rely on the defect in claiming a breach, and his rejection was improper; i.e., he has accepted the goods.

162
Q

The Article 2 battle of the forms provision provides that

A

the proposal of additional or different terms by the offeree in a definite and timely acceptance is effective as an acceptance, unless the acceptance is expressly made conditional on assent to the additional or different terms.

Whether the additional or different terms become part of the contract depends on whether or not both parties are merchants.

  • If any party to the contract is not a merchant, the additional or different terms are considered to be mere proposals to modify the contract. They do not become part of the contract unless the offeror expressly agrees.
  • If both parties are merchants, additional terms in the acceptance become part of the contract unless:
  1. they materially alter the terms of the offer,
  2. the offer expressly limits acceptance to the terms of the offer, or
  3. the offeror has already objected to the terms (or objects within a reasonable time after notice of them is received).

Between merchants, some courts treat different terms in an acceptance the same as additional terms; other courts apply the knockout rule (i.e., conflicting terms are knocked out and replaced by gap-filling terms under the UCC).

163
Q

Under the knockout rule,

A

conflicting terms are knocked out of the contract and gaps are filled by the UCC gap-filler provisions.

Conflicting terms are subject to the knockout rule; additional terms are never subject to the knockout rule.

164
Q

On September 15, a manufacturer of office furniture received an email purchase-order form from a retailer of office furniture. The order was for 100 executive leather swivel chairs and specified a delivery date no later than November 1, at a total cost of $10,000, as quoted on the manufacturer’s website. Two days later, the manufacturer emailed its own purchase-order acceptance form to the retailer, who was a new customer and had never seen the form before. The purchase-order acceptance form stated that it was an acceptance of the specified order, was signed by the manufacturer’s sales manager, and contained all of the terms of the retailer’s form, but it also contained an express warranty and a clause disclaiming all implied warranties such as the implied warranty of merchantability.

Assuming that there were no further communications between the parties, what is the status of the relationship between the parties?

A

The manufacturer and the retailer have a contract without the disclaimer.

In contracts for the sale of goods, a definite expression of acceptance operates as an acceptance even if it states additional terms.

Between merchants, additional terms proposed by the offeree in an acceptance automatically become part of the contract unless

(i) they materially alter the original terms of the offer (e.g., they change a party’s risk or the remedies available);
(ii) the offer expressly limits acceptance to the terms of the offer; or
(iii) the offeror had already objected to the additional terms or objects within a reasonable time.

Here, a clause was added by the manufacturer (the offeree) providing for an express warranty and a disclaimer of all implied warranties, including the warranty of merchantability. The disclaimer materially altered the original terms of the offer. Therefore, the disclaimer would not become part of the contract.

165
Q

Under the mailbox rule, acceptance of an offer by mail creates a contract at the moment

A

the acceptance is posted, properly stamped, and addressed.

If the offeree sends both an acceptance and a rejection, whether the mailbox rule will apply depends on which the offeree sent first, the acceptance or the rejection.

If the offeree first sends an acceptance and later sends her rejection, the mailbox rule applies. Thus, even if the rejection arrives first, the acceptance is effective upon mailing (and so a contract is formed) unless the offeror changes his position in reliance on the rejection. Here, the classmate first sent an acceptance, then called with her rejection. The mailbox rule applies, and because there is nothing in the facts to show that the nurse relied on the rejection, a contract was formed.

The mailbox rule does not apply when the offeree sends a rejection, followed by an acceptance. In such a case, whichever is received first controls.

166
Q

An offer normally can be revoked at will by the offeror. An option contract is a distinct contract in which an offeree gives consideration for a promise by the offeror not to revoke an outstanding offer.

Even if an offeror gives written assurances regarding a promise not to revoke for a certain period, the revocation-at-will rule applies unless the offeror’s power to terminate the offer has been limited in some way, such as by

A

the creation of an option contract. Note that if the offeror could reasonably expect that the offeree would rely to her detriment on the written assurances and the offeree does so rely, the offer could be held irrevocable as an option contract, but the assurances alone are generally not enough to make an offer irrevocable.

Detrimental reliance can limit an offeror’s power to revoke where the offeror could reasonably expect that the offeree would rely to his detriment on the offer, and the offeree does so rely. However, this usually is limited to those situations in which the offeror would reasonably contemplate reliance by the offeree in using the offer before it is accepted; e.g., when a general contractor uses a subcontractor’s bid in making its own offer

167
Q

Under the Merchant’s Firm Offer rule in Article 2, a promise to keep an offer open will be enforceable without the payment of consideration when

A

a merchant offers to buy or sell goods in a signed writing that gives assurances that the contract will be held open for a specified period of time. This is not the same as an option contract, which is a distinct contract and generally requires that the offeree give some consideration for the promise not to revoke.

A merchant’s firm offer does not apply to any offer by a merchant; it applies only to an offer under the UCC for the sale of goods where a signed writing gives assurances that the offer will be held open.

168
Q

A contractor’s duty to construct a building is not discharged by destruction of the work in progress. However, if the destruction was not caused by the contractor, such as by an act of nature,

A

most courts will extend the date of performance beyond the original deadline.

The general rule is that a contractor is responsible for destruction of the premises under construction prior to completion. Once the residence is completed, risk of loss shifts to the owner.

Contractual duties are discharged where it has become impossible to perform them. The occurrence of an unanticipated or extraordinary event may make contractual duties impossible to perform. If the nonoccurrence of the event was a basic assumption of the parties in making the contract, and neither party has assumed the risk of the event’s occurrence, duties under the contract may be discharged.

If there is impossibility, each party is excused from duties that are yet to be performed.

If either party has partially performed prior to the existence of facts resulting in impossibility, that party has a right to recover in quasi-contract for the reasonable value of his performance.

While that value is usually based on the benefit received by the defendant (unjust enrichment), it also may be measured by the detriment suffered by the plaintiff (the reasonable value of the work performed).

A contractual duty to perform may be discharged by objective impossibility (i.e., no one could have performed), but subjective impossibility (defendant could not perform) is insufficient.

169
Q

When the amount due is undisputed, payment of a smaller sum than due will not be sufficient consideration for a promise by the creditor to discharge the debt. Neither a legal detriment nor a benefit would be present. In contrast, if the consideration is in any way new or different, such as

A

payment before maturity or

to one other than the creditor; or

payment in a different medium (e.g., stock instead of cash), then sufficient consideration may be found.

When the proposed consideration is in any way new or different (e.g., an alternative method of payment), there is usually sufficient consideration to change a preexisting duty, such as discharging an existing debt.

170
Q

A professional baseball player visited a sick boy in the hospital. The player told the boy that in consideration of the boy’s courage, he would hit a home run for him in his next game. As the player was leaving the hospital, the boy’s father stopped the player and told him how important the home run could be in improving his son’s spirits and health. The father told the player he would pay him $5,000 if he did hit a home run in his next game. The player agreed and took extra batting practice before his next game to improve his chances. In his next game, the player hit two home runs. The player’s contract with his ball club does not forbid him from accepting money from fans for good performance. The player has now asked the father for the $5,000.

If the father refuses to pay and the baseball player brings an action against him for damages, which of the following is correct under the prevailing modern rule in contract law?

A

Generally, contracts must be supported by consideration. A promise to perform is valid consideration, but if a person already owes a duty to perform, traditionally that performance cannot be used as consideration for another promise. Thus, under the traditional rule, the player could not enforce the father’s promise to pay the player $5,000 if he hit a home run because the player gave no valid consideration in exchange for the father’s promise, since the player owed a preexisting duty to his ball club to exert his best efforts to hit home runs.

However, under the modern view as formulated in Restatement (Second) of Contracts, section 73, and followed by a majority of courts, a duty is a preexisting duty only if it is owed to the promisee. Thus, a promise to perform a duty is valid consideration as long as the duty of performance is not already owed to the promisee. In other words, if the duty is owed to a third party, a promise to perform given to another is valid consideration as long as it was bargained for.

(B) is incorrect because there is no exception to the preexisting duty rule—modern or otherwise—that allows the promisor to recover merely because his performance benefited a third party. The player can recover under the modern approach because his promise to the father was bargained for. Conversely, the player does not have to prove that the value of his home run to the boy was at least $5,000, because courts generally will not inquire into the adequacy of consideration.

Under the modern trend, the promise here is valid consideration because the duty to hit home runs was owed to a third party (the ball club) rather than to the promisee (the father).

171
Q

In a contract for a sale of goods priced at $500 or more, if goods are either received and accepted or paid for, the contract is enforceable without a writing.

A

However, the contract is not enforceable beyond the quantity of goods accepted or paid for. Thus, if only some of the goods called for in the oral contract are accepted or paid for, the contract is only partially enforceable.

A contract for specially manufactured goods, i.e., goods that are to be specially manufactured for the buyer and are not suitable for sale to others by the seller in the ordinary course of his business, can sometimes be enforceable without a writing, but only under circumstances where the seller has reasonably indicated that the goods are for the buyer and made a substantial beginning in their manufacture or committed for their purchase before notice of a repudiation was received. There is no exception to the Statute of Frauds for unique goods.

There is no exception to the Statute of Frauds for contracts in which the goods have been shipped. There is an exception once the goods have been received and accepted or paid for.

172
Q

Generally, under the Statute of Frauds, a promise creating an interest in land must be evidenced by a writing to be enforceable.

A

A seller can enforce an oral land sale contract only if the seller conveys the property to the buyer. A buyer can specifically enforce a seller’s oral promise to sell land if the part performance doctrine applies. Under the doctrine of part performance, conduct that unequivocally indicates that the parties have contracted for the sale of the land will take the contract out of the Statute of Frauds. Most jurisdictions require at least two of the following three actions by the buyer to find part performance: payment, possession, and/or valuable improvements.

173
Q

In a single delivery contract, if the buyer has rejected goods because of defects, the seller may, within the time originally provided for performance,

A

cure by giving reasonable notice of her intention to do so and making a new tender of conforming goods, which the buyer must then accept. If the new tender of conforming goods is made within the time originally provided for performance, the buyer does not have the option to accept or reject the goods. The buyer must accept the conforming goods.

The UCC provides that in cases where a buyer rejects a tender of nonconforming goods that the seller reasonably believed would be acceptable “with or without money allowance,” the seller, upon a reasonable notification to the buyer, has a further reasonable time beyond the original contract time within which to make a conforming tender.

A seller will probably be found to have had reasonable cause to believe that the tender would be acceptable if the seller can show that trade practices or prior dealings with the buyer led the seller to believe that the goods would be acceptable, or the seller could not have known of the defect despite proper business conduct.

174
Q

In an installment contract, an installment can be rejected only if

A

the nonconformity substantially impairs *the value of that installment.*

Moreover, the installment contract as a whole is deemed to be breached only if the nonconformity substantially impairs *the value of the entire contract.*

175
Q

Specific performance is granted when: (i) there is a valid contract; (ii) the legal remedy is inadequate; (iii) enforcement is feasible; and (iv) mutuality of remedy is present.

The gallery owner and her friend had a contract, pursuant to which the gallery owner promised to sell her friend the painting for $2,700. Although this was an oral contract for the sale of goods for a price exceeding $500 and thus subject to the Statute of Frauds, the contract is removed from the Statute by the fact that the friend

A

tendered full payment for the painting. Thus, the oral nature of the agreement is no hindrance to its validity.

Moreover, a painting, by its nature, is unique, rendering the legal remedy (damages) inadequate.

However, feasibility of specific performance against the gallery owner is lacking here. The salesperson sold the painting to a customer who paid value for it and was unaware that the gallery owner had already agreed to sell it to her friend. The salesperson also was unaware of the gallery owner’s agreement with her friend. With the subject matter of the contract having been transferred in good faith to a third party, there is no feasible means to enforce against the gallery owner her agreement to sell the painting to her friend. Thus, the right to specific performance is cut off.

176
Q

A builder contracted to build a house for a newly married couple. Terms of the contract provided that the builder would receive the contract price when the building was fully completed. Just when the builder had completed one-half of the structure, a tornado struck the area and demolished the building.

What is the builder entitled to recover from the couple under the contract?

A

The builder will not be able to recover anything from the couple under the contract because he has not performed his duty. Under the parties’ contract, the builder’s completion of the house was a condition precedent to the couple’s duty to pay. The condition precedent was not discharged by the destruction of the work in progress because construction has not been made impossible, but rather merely more costly—the builder can rebuild. Thus, he is not entitled to any recovery. Note, however, that a number of courts will excuse timely performance because the destruction was not the builder’s fault.

177
Q

The owner of an old car parked it in front of his house with a “for sale” sign in the windshield. In response to an inquiry from his neighbor, the car owner said that he would take $400 for the car. The neighbor responded, “You’ve got a deal.” Because it was a Sunday, and the banks were closed, the neighbor told the car owner that he would come to his house with the $400 the next day at about 6 p.m. The car owner said that was fine. At 9:15 the next morning, the car owner called his neighbor and told him that when they had talked the previous day, he forgot that he had just put two new tires on that car and that he would need an extra $50 to cover their cost. The neighbor agreed to bring $450 in cash to the car owner’s house at about six o’clock.

Is the neighbor legally bound to pay the car owner the additional $50?

A

The neighbor must pay the car owner the additional $50 because the parties have an enforceable contract.

A contract for the sale of goods (the car) was formed when the neighbor said, “You’ve got a deal.” The parties then orally agreed to a modification of the contract when the car owner called his neighbor the next morning. Under the Statute of Frauds provision in the UCC, which applies to all contracts for the sale of goods, a promise requires a writing signed by the party to be charged to be enforceable if it is for the sale of goods of $500 or more. Here, the contract as modified is under $500, so it is enforceable even though it is not in writing.

(C) is incorrect because under UCC section 2-209, no consideration is needed for a good faith modification of a contract for the sale of goods.

178
Q

A retailer entered into an oral contract with an office supply wholesaler to buy 100 file boxes for an upcoming back to school sale at the retailer‘s store. The wholesaler agreed to deliver the file boxes in two weeks at a cost of $4 per file box. A week later, the retailer phoned the wholesaler and asked if she could increase her order to 200 file boxes. The wholesaler agreed. The wholesaler delivered the 200 file boxes as promised, but the retailer accepted only 150 upon discovering that she lacked storage space for all 200.

May the wholesaler recover damages with respect to the 50 file boxes that were not accepted?

A

The wholesaler may not recover damages. Under the UCC Statute of Frauds, a contract for the sale of goods for $500 or more is unenforceable unless evidenced by a writing signed by the party sought to be held liable. The original contract was for $400 and, thus, was not within the Statute. Whether a modification must be in writing to be enforceable depends on whether the entire contract price as modified is within the Statute. Here, the retailer and the wholesaler modified their original contract to 200 file boxes, bringing the total price to $800. Thus, the modification was unenforceable under the Statute of Frauds, and the wholesaler cannot collect damages with respect to the 50 unaccepted file boxes.

(A) is incorrect. Acceptance is an exception to the Statute of Frauds—but only to the extent of the goods accepted. That is, an oral contract for the sale of goods for $500 or more is enforceable to the extent the goods are accepted. Here, the original contract for 100 boxes is enforceable, but the modification is not. However, since the retailer accepted 50 additional boxes, the modified contract is enforceable to the extent of the additional 50 boxes accepted. The fact that the accepted amount meets the Statute of Frauds $500 threshold does not make the contract enforceable for all 200.

179
Q

A small processor of specialized steel agreed in writing with a small manufacturer of children’s toys that it would supply, and the manufacturer would buy, all of the manufacturer’s specialized steel requirements over a period of years at a set price per ton of steel. Their contract did not include a nonassignment clause. Recently, the toy manufacturer decided to abandon its line of steel toys, so it made an assignment of its rights and delegation of its duties under the contract to a toymaker many times larger. The large toymaker notified the steel processor of the assignment and relayed to the processor its good faith belief that its requirements will approximate those of the assignor.

Must the steel processor supply the requirements of the large toymaker?

A

Because the large toymaker acts in good faith in setting its requirements to approximately those of the small manufacturer into whose shoes it stepped, the contract may be assigned. The contract in this question is a “requirements” contract: The steel processor must sell the small manufacturer of children’s toys all the specialized steel it requires for its toys. Generally, the right to receive goods under a requirements contract is not assignable because the obligor’s duties could change significantly. In fact, here, a significant change would seem possible because the large toymaker is a larger company than the small manufacturer and its needs could be greater. However, the UCC allows the assignment of requirements contracts if the assignee acts in good faith not to alter the terms of the contract. [UCC §2-306] (The UCC applies here because goods are involved.) Thus, assuming the large toymaker’s requirements remain about the same as the small manufacturer’s requirements, the steel processor would be required to honor its contract, now assigned to the large toymaker.

180
Q

A landowner entered into a written agreement with a real estate broker whereby the broker would receive a commission of 10% of the sale price if he procured a “ready, willing, and able buyer” for the landowner’s property and if the sale actually proceeded through closing. The broker found a buyer who agreed in writing to buy the property from the landowner for $100,000, the landowner’s asking price. The buyer put up $6,000 as a down payment. The agreement between the landowner and the buyer contained a liquidated damages clause providing that, if the buyer defaulted by failing to tender the balance due of $94,000 at the closing date, damages would be 10% of the purchase price. The landowner included that clause because she was counting on using the proceeds of the sale for a business venture that would likely net her at least $10,000.

The buyer became seriously ill and defaulted. When he recovered, he demanded that the landowner return his $6,000, and the landowner refused. The broker also demanded the $6,000 from the landowner and was refused. The broker and the buyer filed separate suits against the landowner, with the buyer pleading impossibility of performance. The two cases are consolidated into a single case.

How should the court rule as to the disposition of the $6,000?

A

The landowner may keep the $6,000 as liquidated damages. A liquidated damages clause is enforceable if: (i) damages are difficult to ascertain at the time of the making of the contract, and (ii) the damages are a reasonable forecast of compensatory damages. Here, the landowner was unsure what her damages would be if she did not receive the sales proceeds from the property, but $10,000 seemed a reasonable amount. Thus, both criteria for valid liquidated damages clauses are met.

(D) is incorrect because the broker was to receive proceeds from the sale of the property; the $6,000 was damages and not sale proceeds.

181
Q

A man shopping for a leather jacket at a clothing store could not decide between two jackets, so the proprietor, who knew the man and his family well, let him take one of the jackets on approval. No mention was made by the proprietor of the method of payment he expected. The man wore the jacket on a visit to his grandfather, who liked it so much that when the man told him what the jacket cost and that he had taken it on approval, the grandfather said he would buy it for him if he promised to give some of his old clothes to a favorite charity for the poor at Christmastime. The man wholeheartedly agreed to donate the clothes to the charity at Christmas. Very pleased, the grandfather called the shop and told the proprietor to send the bill for the jacket to him, which he did. Before the bill was paid and before the Christmas season arrived, the grandfather fell ill and died. The grandfather’s executor has refused to pay the bill, and the man has not yet given any old clothing to the charity.

Will the proprietor be able to recover the price of the jacket from the estate?

A

The proprietor can recover the cost of the jacket from the grandfather’s estate because the proprietor is an intended third-party beneficiary and his right to enforce the contract has vested. The rights of an intended third-party beneficiary vest when the beneficiary

(i) manifests assent to the promise in a manner invited or requested by the parties;

(ii) brings suit to enforce the promise; or

(iii) materially changes his position in justifiable reliance on the promise.

Here, the proprietor qualifies as an intended beneficiary of the agreement between the man and his grandfather because the proprietor was expressly designated in the contract, he was to receive performance directly from the grandfather, and he stood in an existing contractual relationship with the man that required the man to either pay for the jacket or return it, making it likely that the young man’s purpose in making the arrangement with his grandfather was to satisfy the obligation to the proprietor. The proprietor can enforce the contract because his rights vested when he sent the bill to the grandfather at the grandfather’s request. Thus, the proprietor will prevail against the grandfather’s estate.

(B) is wrong because the man does have a duty to give the clothes to the charity; if he does not do so, he will be in breach of his contract with his grandfather, and this would give his grandfather’s estate a defense to payment. However, the man’s time for performance (Christmastime) has not yet occurred, and so he is not in breach. Nevertheless, this fact is not the reason the proprietor will recover; he will recover due to his status as an intended beneficiary, not because this possible defense has been negated.

(D) is wrong because the man’s giving the clothes to the charity is not a condition that must be fulfilled before the grandfather’s estate must pay. The grandfather promised to pay for the jacket if the man promised to donate the clothes; i.e., the consideration for the grandfather’s promise was the man’s promise, not his actually donating the clothes. As soon as the man made the promise, the grandfather’s duty to pay became absolute. (If the man does not donate the clothes, he will be in breach of his contract with his grandfather, but the grandfather’s performance was not conditioned on the man’s donating the clothes.)

182
Q
A